RRB JE - SPLessons

RRB JE 2019 Reasoning Ability Quiz

Home > > Tutorial
SPLessons 5 Steps, 3 Clicks
5 Steps - 3 Clicks

RRB JE 2019 Reasoning Ability Quiz

shape Introduction

What is reasoning and its types? Reasoning is the process of using existing knowledge to draw conclusions, make predictions, or construct explanations. Three methods of reasoning are the deductive, inductive, and abductive approaches. Deductive reasoning: conclusion guaranteed.
The article RRB JE 2019 Reasoning Ability Quiz provides Reasoning Ability questions with answers useful to the candidates preparing for Competitive exams, Entrance exams, Interviews etc. Railway Recruitment Board (RRB) has released RRB JE 2019 Official Notification to recruit eligible candidates for the post of it Railway JE Recruitment 2018-19 will be held to recruit candidates for the posts of Junior Engineer(JE), Junior Engineer(Information Technology), Depot Material Superintendent(DMS) and Chemical & Metallurgical Assistant(CMA).Reasoning Ability plays major role to qualify examination. The article RRB JE 2019 Reasoning Ability Quiz will assist the students understand the type of questions expected from the topic Reasoning Ability .
Click Here for RRB JE official website

shape Quiz

1. Direction: In the following question, one/two statements are given followed by two conclusions (I) and (II). You have to consider the two statements to be true even if they seem to be at variance from commonly known facts. You have to decide which of the given conclusions, if any, follow from the given statements. Statements : Some books are Magazines. Some Magazines are Novels. Conclusions : (I) Some books are Novels. (II) Some Novels are Magazines.
    A. Only (I) follows B. Only (II) follows C. Either (I) or (II) follows D. Neither (I) nor (II) follows

Answer - Option B
Explanation :

Conclusion I doesn't follow.
Conclusion II follows.
Hence option B is the right answer. 2. Direction: In the following question, some statements followed by some conclusions are given. Taking the given statements to be true even if they seem to be at variance from commonly known facts, read all the conclusions and then decide which of the given conclusions logically follows the given statements. Statements: 1. No girl is a parrot. 2. Sowmya is a girl. Conclusions: I. Sowmya is not a parrot. II. All girls are not Sowmya.
    A. Only conclusions I follows B. Only conclusions II follow C. Either conclusions I or II follows D. Neither conclusions I nor II follows

Answer - Option A
Explanation :
Only conclusion I can be inferred from the Statement as no girl is parrot and Saumya is a girl. Thus Saumya cannot be a parrot.
Conclusion II cannot be inferred from the given Statements.
Hence Option A is correct
3. In the following question, some statements are given followed by two conclusions/inferences I and II. You have to consider the statements to be true even if they seem to be at variance from commonly, known facts. You are to decide which of the given conclusions. If any, follow from the given statements. Indicate your answer. Statements : All huts are painted blue. Some blue are Bungalows. Inferences : I. All Bungalows are blue. II. Some blue are not Huts.
    A. Only inference I follows B. Only inference II follows C. Both inferences I and II follow D. Neither of the two inferences follows

Answer - Option D
4. Direction: Two statements are given followed by four conclusions I, II and IV. You have to consider the statements to be true even if they seem to be at variance from commonly known facts. You are to decide which of the given conclusions, if any, follow from the given statements. Indicate your answer. Statements : 1. All dogs are books 2. All books are pictures. Conclusion : I. All dogs are pictures. II. All books are dogs. III. All pictures are dogs. IV. Some pictures are books.
    A. Only I and IV follow B. Only II and III follow C. Either II or III follows D. Either I or IV follows

Answer - Option A
Explanation :
Conclusion :
I. All dogs are pictures. (correct; as the region of dogs is fully contained in the region of books)
II. All books are dogs. (incorrect; as the region of dogs does not contain the whole region of books)
III. All pictures are dogs. (incorrect; as the region of dogs does not contain the whole region of pictures)
IV. Some pictures are books. (correct; as the region of books contains some of the region of pictures).
5. Directions: Two statements are given followed by four conclusions I, II, III and IV. You have to consider the statements to be true even if they seem to be at variance from commonly known facts. You are to decide which of the given conclusions, if any, follow from the given statements. Indicate you’re answer. Statements : 1. All goats are tigers 2. All tigers are lions. Conclusions : I. All tigers are goats. II. All lions are tigers III. No goat is a lion. IV. No lion is a goat
    A. Either II or III follows B. Either II or IV follows C. Either I or III follows D. None of the conclusions follow

Answer - Option D
Explanation :

Conclusions :
I. All tigers are goats. (Not true)
II. All lions are tigers. (Not true)
III. No goat is a lion. (not true)
IV. No lion is a goat. (not true)
Therefore None of the conclusions follow.
6. Directions: In each of the following questions one/two statements are given followed by two conclusions (I) and (II). You have to consider the two statements to be true even if they seem to be at variance from commonly known facts. You have to decide which of the given conclusions, if any, follow from the given statements. Statements : (1) All students like excursions. (2) Some students like Laboratory experiments. Conclusions : (I) Students who like Laboratory experiments also like excursions. (II) Some students do not like Laboratory experiments but likes excursion.
    A. Only (I) follows B. Only (II) follows C. Conclusions (I) and (II) follow D. Neither (I) nor (II) follows

Answer - Option A
Explanation :

Conclusions :
(I) Students who like Laboratory experiments also like excursions. (True) - It is clear from the diagram.
(II) Some students do not like Laboratory experiments but likes excursion. (False) - It cannot be determined from the given statement.
Hence, option A is the correct response.
7. Directions: Two statements are given followed by four conclusions I, II, III and IV. You have to consider the statements to be true even if they seem to be at variance from commonly known facts. You are to decide which of the given conclusions, if any, follow from the given statements. Indicate your answer. Statements : (i) All Blue is Black and some Black is red. (ii) All red is Green but not Yellow
Conclusions : I. Some Green is Black. II. No Black is Yellow. III. Some Black is not Yellow. IV. No Black is Green.
    A. Only I and II follow B. Only II and IV follow C. Only I and III follow D. Only I, II and IV follow

Answer - Option C
Explanation :

Conclusion 1:Some Green is Black is correct.
Conclusion 2: Some black is not yellow follows, but no black is yellow does not follow.
Some Black is not Yellow is also correct because some part of Red and Black is not Yellow.
Thus only Conclusions I and III follow.
8. Directions: One statement is given, followed by two conclusions I and II. You have to consider the statement to be true, even if it seems to be at variance from commonly known facts. You have to decide which of the given conclusions, if any, follow from the given statement.Indicate your answer. Statement: I. All medals are awards. II. Some awards are win. Conclusions : I. All medals are not awards. II. Some win are not medals.
    A. Both conclusions I and II follow B. Neither conclusion I nor II follows C. Only conclusion I follows D. Only conclusion II follows

Answer - Option C
Explanation :

All awards are not medals.( not follow) as all medal are awards.
Some win are not medals. (not follow) as there is no direct relationship between win and medal.
9. Directions: In each of the following questions two/three statements are given followed by two/four conclusions I, II, III and IV. You have to consider the statements to be true even if they seem to be at variance from commonly known facts. You have to decide which of the given conclusions, if any, follow from the given statements Statements: 1) Some humane creatures are angels. 2) All doctors are angels. Conclusions: I. Some humane creatures are doctors. II. Some doctors are humane creatures.
    A. Only I follows B. Only II follows C. Either I or II follows D. Neither I nor II follows

Answer - Option D
Explanation :

10. Directions: In the following question, two/three statements are given followed by two/four conclusions I, II, III and IV. You have to consider the statements to be true even if they seem to be at variance from commonly known facts. You have to decide which of the given conclusions, if any, follow from the given statements. Statements: 1. Some books are mobiles. 2. Some calculators are mobiles. Conclusions: I. Some mobiles are calculators. II. Some mobiles are books
    A. Only conclusion I follows B. Only conclusion II follows C. Both conclusions I and II follow D. Neither conclusion I nor II follows

Answer - Option C
Explanation :

I. Some mobiles are calculators. (follows). As some calculators are mobiles.
II. Some mobiles are books. (follows). As some books are mobiles.
1. Direction: In the question below are given three statements followed by three conclusions number I, II and III. You have to take the given statements to be true even if they seem to be at variance from commonly known facts. Read the entire conclusion and then decide which of the given conclusions logically follows from the given statements disregarding commonly known facts. Statements: Some marker are sketch. Some sketch are sharpener. No sharpener is eraser. Conclusions: I. Some eraser is sketch. II. Some eraser is sharpener is a possibility. III. All sketch can be eraser
    A. None follows B. only II follows C. only III follows D. only I follows

Answer - Option A
Explanation :
2. Direction: In the question below are given three statements followed by three conclusions number I, II and III. You have to take the given statements to be true even if they seem to be at variance from commonly known facts. Read the entire conclusion and then decide which of the given conclusions logically follows from the given statements disregarding commonly known facts. Statements: All ruby are diamond. Some ruby are gold. No diamond is silver. Conclusions: I. Some ruby are not silver. II. All gold can be silver is a possibility. III. Some silver can be gold is a possibility.
    A. only I and II follow B. only I and III follow C. only II and III follow D. All I, II and III follow

Answer - Option B
Explanation :
3. Direction: In the question below are given three statements followed by three conclusions number I, II and III. You have to take the given statements to be true even if they seem to be at variance from commonly known facts. Read the entire conclusion and then decide which of the given conclusions logically follows from the given statements disregarding commonly known facts. Statements: All elephants are cat. All cat are lion. No dog is are elephant. Conclusions: I. Some lion is dog. II. Some cat are dog is a possibility. III. Some dog are elephant.
    A. Only I follows B. Only II follows C. Only III follows D. All follow

Answer - Option B
Explanation :
4. Direction: Based on the given statement and few conclusions, choose the right statement which gives the correct conclusions. Statements: 1) Some drinks are slice. 2) Some slice are mangoes. 3) Some mangoes are fruits. Conclusions: A) Some mangoes are definitely not drink. B) Some fruits are slice.
    A. Only conclusion A follows. B. Only conclusion B follows. C. Either conclusion A or B follows. D. Neither conclusion A nor B follows.

Answer - Option D
5. Direction: In the following question, some statements are followed by some conclusions. Assuming the given statements to be true, find which of the following conclusions follow the given statements and choose appropriate answer choice. Statements: All Clerks are PO No Mangers is clerk. All PO are Cashier. Conclusions: I. All Clerks are Cashier II. Some Cashiers are not PO’s III. Some PO’s are Mangers
    A. Only I and II follow B. Only III follows C. Only I follows D. None follows

Answer - Option C
6. Direction: In the following question, some statements are followed by some conclusions. Assuming the given statements to be true, find which of the following conclusions follow the given statements and choose appropriate answer choice. Statements: Some CM’s are MLA’s. All MLA’s are Leaders No Leader is a PM. Conclusions: I. Some PM’s are MLA’s II. Some MLA’s are CM’s
    A. Only I follow B. Only II follow C. Neither I nor II follows D. All follow

Answer - Option B
7. Direction: In the following question, some statements are followed by some conclusions. Assuming the given statements to be true, find which of the following conclusions follow the given statements and choose appropriate answer choice. Statements: No eagle are rabbits. All cheetah are eagle. All deer are cheetah. Conclusions: I. Some eagle are not deers II. No cheetah is rabbit
    A. Only I follow B. Only II follow C. Neither I nor II follows D. All follow

Answer - Option B
8. Direction: In the following question, some statements are followed by some conclusions. Assuming the given statements to be true, find which of the following conclusions follow the given statements and choose appropriate answer choice. Statements: All IPL’s are ICC. No ICC is a FIFA. Some ICC’s are PSL. Conclusions: I. Some IPL’s are not FIFA II. Some PSL’s are IPL is a possibility.
    A. Only I follow B. Only II follow C. Neither I nor II follows D. All follow

Answer - Option D
Explanation :
9. Directions: In each of the following questions two statements are given and these statements are followed by two conclusions numbered (1) and (2). You have to take the given two statements to be true even if they seem to be at variance from commonly known facts. Read the conclusions and then decide which of the given conclusions logically follows from the two given statements, disregarding commonly known facts. Statements: All sweet are sour. No sour is tasty. All tasty are food. Conclusions: I. All sweet being food is a possibility. II. No sweet is tasty.
    A. Only I follows B. Only II follows C. Either I or II follows D. Both I and II follow

Answer - Option D
10. Directions: Study the following information and answer the questions. In each of the questions below are given two or three statements followed by two conclusions numbered I and II. You have to take the given statements to be true even if they seem to be at variance with commonly known facts. Read all the conclusions and then decide which of the given conclusions logically follows from the given statements, disregarding commonly known facts. Give answer. Statements: Some handles are doors All keys are doors No lock is a handle Conclusions: I. At least some handles are not keys II. No door is a key
    A. if only conclusion I follows B. if only conclusion II follows C. if either conclusion I or conclusion II follows. D. if neither conclusion I nor conclusion II follows

Answer - Option D
Explanation :
1. Direction: In each of the following questions, two statements are given followed by two/four conclusions I, II, III and IV. You have to consider the two statements to be true even if they seem to be at variance from commonly known facts. You have to decide which of the given conclusions, if any, follow from the given statements. Statement: 1. All peacocks are lions. 2. Some tigers are peacocks. Conclusions : I. Some lions are not tigers. II. All tigers are lions. III. Some tigers are lions. IV. All peacocks are tigers.
    A. Only conclusion III follows B. Only conclusion IV follows C. Only conclusion I follows D. Only conclusion II follows

Answer - Option A
Explanation :

From the given diagram, conclusion III is the correct response.
2. Direction: In the following question, two statements are followed by four conclusions I, II, III and IV. You have to consider the statements to be true even if they seem to be at variance from commonly known facts. You are to decide which of the given conclusions, if any, follow from the given statements. Statements: (A) All clerks are typists. (B) Some typists are stenos. Conclusions: I. Some stenos are clerks. II. No steno is a clerk. III. All typists are clerks. IV. All clerks are stenos.
    A. All the conclusions follow B. None of conclusions follow C. Either I or II follows D. Only IV follows

Answer - Option C
Explanation :
All clerks are typists. (A-type)
Some typists are stenos. (A-types)
Therefore,
All clerks are typists [latex]\longleftrightarrow[/latex] Some typists are stenos
A + A types conclusion A- type conclusion
Conclusion I follows.
All typists are clerks[latex]\longleftrightarrow[/latex] All typists are stenos.
A + E types conclusion E -type conclusion
Conclusion II also follow.
Thus, Either I or II follows.
Hence, option C is correct.
3. Mother was three times the age of her daughter five years ago. After five years, mother will be twice as old as her daughter. How old is the daughter today?
    A. 5 years B. 10 years C. 15 years D. 20 years

Answer - Option C
Explanation :
Suppose the present age of daughter is x years and the present age of mother is y years.
According to the question;
3(x - 5) = (y - 5)
3x - 15 = y - 5
3x - y = 10 ……………..(i)
2(x + 5) = (y + 5)
2x + 10 = y + 5
2x - y = -5……………..(ii)
From the equations (i) and (ii);
x = 15 years
Hence, option C is the correct response.
4. Ramesh and Suresh are brothers. Dharmendra is the father of Ramesh and Sunita is the wife of Suresh's only brother. Sapna is the daughter of Sunita. How is Sapna related to Dharmendra?
    A. Daughter B. Sister-in-law C. Niece D. Granddaughter

Answer - Option D
Explanation :

Thus Sapna is the Granddaughter of Dharmendra.
5. In a certain code language, "MATERIAL" is written as "RIALMATE". How is "REMEMBER" written in that code language?
    A. REMEREBM B. MBEREMER C. MBERREME D. MBERREEM

Answer - Option C
Explanation :
Since the word ‘MATERIAL’ has been divided in two equal part and in coded language second
part is written first followed by first part likewise,
First part of ‘REMEMBER’ is ‘REME’ and second part is ‘MBER’
Therefore, coded word will be ‘MBERREME’
Hence, the correct option is C.
6. Vineet travels 16 km towards west, turns right and travels another 12 km, and then takes two successive left turns covering 8 km and 12 km in each turn respectively. Finally, he takes a right turn and travels 10km further. How far is he now from his original position?
    A. 46 km B. 34 km C. 40 km D. 38 km

Answer - Option B
7. In the following question, select the missing number from the given series.
    A. 19 B. 1 C. 21 D. 13

Answer - Option D
Explanation :
Elements in 3rd row is the sum of elements in 1st row and 2nd row respectively:
9 = 5 + 4
11 = 6 + 5
13 = 7 + 6
Hence, the correct option is D.
8. Direction: Unscramble the following letters to frame a meaningful word and find out the correct numerical sequence of the letters.
    A. 10 2 3 5 16 4 7 8 9 B. 3 1 2 4 5 7 6 9 8 10 C. 1 3 5 2 9 4 8 6 7 10 D. 9 1 3 6 2 7 54 8 10

Answer - Option B
Explanation :
Let we form a meaningful word from given alphabets:
R E S T A U R A N T
3 1 2 4 5 7 6 9 8 10
Hence correct option is (3 1 2 4 5 7 6 9 8 10)
9. A series is given with one term missing. Select the correct alternative from the given ones that will complete the series. AT, CR, FO, JK, ?
    A. PE B. QD C. NM D. OF

Answer - Option D
Explanation :

A + 2 = C, C + 3 = F, F + 4 = J and J + 5 = O
T - 2 = R, R - 3 = O, O - 4 = K and K - 5 = F
Hence, the correct option is D
10. Identify the diagram that best represents the relationship among the given classes. Animals, Lion, Tiger
    A. B. C. D.

Answer - Option A
Explanation :
Lion and Tiger both belong to the Animal category.
Hence A is the correct answer.
1. In a certain code language, “TRUMP” is written as “46321” and “GRAIN” is written as “76598”. How is “GRUNT” written in that code language?
    A. 23684 B. 23847 C. 67834 D. 76384

Answer - Option D
Explanation :

GRUNT is 76384
Hence, the correct option is D
2. In a certain code language, ”RIVER” is written as “12351” and “RED” is written as “156”. How is “DRIVER” written in that code language ?
    A. 612311 B. 612531 C. 456789 D. 612351

Answer - Option D
Explanation :
‘D’ is coded as ‘6’
‘R’ is coded as ‘1’
‘I’ is coded as ‘2’
‘V’ is coded as ‘3’
‘E’ is coded as ‘5’
Therefore, DRIVER will be: 612351
Hence, the correct option is D.
3. In a certain code language, "TEACHER" is written as "6427341" and "MAN" is written as "825". How is "MERCHANT" written in that code language ?
    A. 84172563 B. 84172564 C. 84173256 D. 84172566

Answer - Option C
Explanation :

Therefore, MERCHANT will be coded as: 84173256
Hence, the correct option is C.
4. If PARK is coded as 5394. SHIRT is coded as 17698 and PANDIT is coded as 532068, how would you code NISHAR in that code language ?
    A. 261739 B. 266734 C. 231954 D. 201739

Answer - Option A
5. In a certain code MISCHIEF is written as NKVGMOLN, then how is RELIEVED written in that code?
    A. SFMJFMFE B. SEOIJVLD C. SGOMJBLL D. SGOMJVED

Answer - Option C
6. In a certain code language "GASTRIC" is written as "HCVXWOJ". How will "DECEIVE" be written in that code language?
    A. ACBCCTG B. ACACCTG C. EGFINBI D. BFCCTGL

Answer - Option C
Explanation :
As,
G + 1 = H
A + 2 = C
S + 3 = V
T + 4 = X
R + 5 = W
I + 6 = O
C + 7 = J
Similarly,
D + 1 = E
E + 2 = G
C + 3 = F
E + 4 = I
I + 5 = N
V + 6 = B
E + 7 = L
Thus, "DECEIVE" will be written in that code language as EGFINBL.
Hence, option C is the correct response.
7. In a certain code language, "ROYALTY" is written as "ZUMBZPS". How is "LINGER" written in that code language?
    A. SFHJOM B. SFHOJM C. SHFOJM D. SHFOMJ

Answer - Option B
Explanation : As,
Similarly,
Thus LINGER is coded as SFHOJM.
8. In a certain language "BROTHER" is written as "CSPUIFS". How is "SISTER" written in that code language?
    A. TJTFUS B. TJTUFS C. SFUTJT D. TJTSFU

Answer - Option B
Explanation :
There are 26 alphabets in English and if we assign numbers to each and every alphabet starting from ‘A’, ‘B’, ‘C etc., it will appear to be:
A = 1, B = 2, C = 3, D = 4……. likewise, till Z = 26

Hence, the correct option is B.
9. In a certain code language, "READ" is written as "18514". How is "RATE" written in that code language?
    A. 181520 B. 181025 C. 181205 D. 181914

Answer - Option C
Explanation :

10. In a certain code language, "DOMINOS" is written as "5981796" and "MONEY” is written as "89742". How is "MOMOS" written in that code language?
    A. 89872 B. 89896 C. 89895 D. 89897

Answer - Option B
Explanation :

Therefore, MOMOS is coded as: 89896
Hence, the correct option is B.
1. Direction: Select the related word/letters/ number from the given alternatives. AEG: GKM :: MQS : ?
    A. SWY B. SWZ C. ZWQ D. SWB

Answer - Option A
2. Five boys A, B, C, D and E are standing in a row. D is on the right of E. B is on the left of E, but on the right of A. D is on the left of C, who is standing on the extreme right. Who is standing in the middle ?
    A. E B. B C. C D. D

Answer - Option A
3. Directions: In the following question, which one of the given responses would be a meaningful order of the following words in ascending order? 1) Point 2) Triangle 3) Square 4) Angle 5) Line
    A. 4, 1, 5, 2, 3 B. 3, 2, 1, 5, 4 C. 2, 1, 4, 5, 3 D. 1, 5, 4, 2, 3

Answer- Option D
4. If police is called teacher, teacher is called politician, politician is called doctor, doctor is called lawyer and lawyer is called surgeon, who will arrest the criminals?
    A. Teacher B. Doctor C. Police D. Lawyer

Answer Option A
Explanation :
Police arrests the criminals. Here police has been called as Teacher.
5. In a certain code COURSE is written as ESRUOC. How can BREATH be written in that code?
    A. HATEBR B. HTAERB C. HTBREA D. HEATRB

Answer - Option A
6. The sum of the ages of a father and a son presently is 70 years. After 10 years the son's age is exactly half that of the father's. What are their ages now ?
    A. 45 years, 25 years B. 50 years, 20 years C. 47 years, 23 years D. 50 years, 25 years

Answer - Option B
7. Find the odd one
    A. KJML B. GFIH C. TSVU D. ZABY

Answer - Option B
8. In the following question, select the related word from the given alternatives. Poverty : Unemployment : Anaemia : ?
    A. Malnutrition B. Illiteracy C. Drought D. Death

Answer - Option A
9. In the following question, select the odd number pair from the given alternatives.
    A. 122 - 1331 B. 173 - 2197 C. 197 - 2744 D. 290 - 4913

Answer - Option B
Explanation :
122 = [latex]{11}^{2}[/latex] + 1, 1331 = [latex]{11}^{3}[/latex]
173 = [latex]{13}^{2}[/latex] + 4, 2197 = [latex]{13}^{3}[/latex]
197 = [latex]{14}^{2}[/latex] + 1, 2744 = [latex]{14}^{3}[/latex]
290 = [latex]{17}^{2}[/latex] + 1, 4913 = [latex]{17}^{3}[/latex]
Hence, the correct option is B.
10. In the following question, select the odd letter from the given alternatives.
    A. TRQO B. WUTR C. JHFD D. LJIG

Answer - Option C
Explanation :
Clearly, there is gap of 1 letter between first two letters followed by no gap in other two which is followed by gap of 1 letter.
T _ R Q _ O
W _ U T _ R
J _ H _ F _ D
L _ J I _ G
1. In the following question, select the odd word from the given alternatives.
    A. Constellation B. Galaxy C. Star D. Cluster

Answer - Option C
Explanation :
Constellation is a a group of stars forming a recognizable pattern that is traditionally named after its apparent form or identified with a mythological figure.
Galaxy is a system of millions or billions of stars, together with gas and dust, held together by gravitational attraction.
Cluster is a group of similar things or people positioned or occurring closely together.
All except option (C) are group of stars in different form.
Hence, option C is different from others.
2. In the following question, select the odd letter from the given alternatives
    A. DFH B. QSU C. MOQ D. VWX

Answer - Option D
Explanation :
Except VWX, in all other groups, the subsequent letter is 2 steps forward than the previous letter.

Hence, it is clear from above that option (D) is odd one out.
3. In the following question, select the odd number pair from the given alternatives.
    A. 43-6 B. 28-4 C. 50-7 D. 36-5

Answer - Option B
Explanation :
43: 6 * 7 + 1
28: 4 * 7 + 0
50: 7 * 7 + 1
36: 5 * 7 + 1
Clearly, 28 is different from others.
4. Select the odd word from the given alternatives.
    A. Moon B. Mars C. Saturn D. Jupiter

Answer - Option A
Explanation :
Mars, Saturn and Jupiter are planets whereas Moon is the satellite.
Thus Moon is the odd word.
5. Select the odd number from the given alternatives.
    A. 13 B. 17 C. 21 D. 23

Answer - Option C
Explanation :
Here 13, 17 and 23 are prime numbers whereas 21 is the composite number.
Thus 21 is the odd number.
6. In the following question, select the odd letters from the given alternatives.
    A. AEYB B. GKEH C. DHCF D. MQKN

Answer - Option C
Explanation :

Thus DHCF are the odd letters.
7. In the following question, select the odd number pair from the given alternatives
    A. 361 B. 289 C. 225 D. 256

Answer - Option D
Explanation :
As,
[latex]{16}^{2}[/latex] - 256
[latex]{17}^{2}[/latex] - 289
[latex]{15}^{2}[/latex] - 225
[latex]{19}^{2}[/latex] - 361
Only, option D is the square of even number.
Thus 256 is different from others.
8. Select the odd word from the given alternatives
    A. Book B. Page C. Pen D. Diary

Answer - Option C
Explanation :
Book, Page and Diary are all made up of Paper whereas Pen is not. Thus Pen is the odd word.
9. Select the odd number from the given alternatives.
    A. 341 B. 342 C. 810 D. 405

Answer - Option A
Explanation :
3 + 4 + 1 = 8
3 + 4 + 2 = 9
8 + 1 + 0 = 9
4 + 0 + 5 = 9
Hence, the correct option is A.
10. In the following question, select the odd letters from the given alternatives.
    A. DV B. GR C. FT D. JP

Answer - Option B
Explanation :
If we arrange alphabets in two rows as shown below, the letters are related as follows,
D opposite W, W-1 = V
G opposite T, T-1 = S
F opposite U, U-1 = T
J opposite Q, Q-1 = P

Thus GR are the odd letters.
1. Rahul leaves his home and walks 5 km towards east, turns in the south-east direction and walks for 10km, then he turns north-east and moves 10 km. Again, he moves towards the north for 10km. In which direction is he now from starting point?
    A. West B. East C. North-east D. North-west

Answer - Option C
Explanation :

Thus Rahul is now towards north east direction from his starting point.
2. Arun travels 8 km towards the North, then turns left and travels 3 km and then again turns right and covers another 4 km and then turns right and travels another 3 km. How far is he from the starting point?
    A. 18 km B. 11 km C. 12 km D. 15 km

Answer - Option C
Explanation :

8 + 4 = 12km
He is 12 km far from the starting point.
3. Raman starts walking in the morning facing the Sun. After sometime, he turned to the left. Later again he turned to his left. At what direction is Raman moving now?
    A. East B. West C. South D. North

Answer - Option B
Explanation :
In the morning sun rises from east. Hence Rahman is facing into ‘East’ direction in the starting. Let Rahman starts from point ‘A’ and moves into east direction. After reaching at ‘B’ Rahman turns left (i.e. in north direction) and moves up to point ‘C’. Now again rahman turns in to left (i.e. in west direction). Finally Rahman moves in to west direction.


4. Ramesh drives his car 4 km to east, then takes left turn and drives 6 km. He again turns to his left hand side and travels 4 km. How many km is Ramesh away from his starting place?
    A. 6 km B. 10 km C. 4 km D. 14 km

Answer - Option A
5. Ramesh went 20m to the east, then he turned left and after 15 m turned right and went 25 m and then turned right and went 15 m. How far was Ramesh from the starting point?
    A. 60 m B. 35 m C. 40 m D. 45 m

Answer - Option D
6. Direction: Anita drives from point A towards north and travels 30 kms. She then turns to her right and travels 4 kms, and then again turns to the right and drives straight for 30 kms. How much distance she has to cover to go straight to the starting point?
    A. 26 kms B. 8 kms C. 22 kms D. 4 kms

Answer - Option D
Explanation :
Anita drives from point A towards north and travels 30 kms and let she reaches at B. At point B she turns to her right (i.e. east direction) and travels 4 kms to reach at C. At point C, she again turns to the right (i.e. in south direction) and drives straight for 30 kms up to D.


From the diagram it is clear that she has to cover 4Km to go straight to the starting point (D ? A).
7. Abhinav started running from his house, he first ran for 12 km towards west, then he turned towards north and ran 16 km in that direction. How far Abhinav is from his house and in which direction?
    A. 13 km South B. 13 km North C. 15 km West D. 20 km North-West

Answer - Option D
Explanation :
The path followed by Abhinav is traced as follows,


162 + 122 = 400
Thus according to Pythagoras theorem distance between Abhinav’s house and his present position is 20 km.
Thus Abhinav is 20 km towards north-west from his house.
8. A girl walks 100 m straight from her home in south direction, to go to the market during her morning walk. Then she turns left for 60 m, then 85 m turning her right and then again turns right for 60 m. What is the shortest distance from her current location to the home ?
    A. 185 metre B. 180 metre C. 160 metre D. 165 metre

Answer - Option A
Explanation :
Required distance
= AE = AB + BE
= (100 + 85)m. = 185 metres
Hence, option A is the right answer.
9. A person walks in north direction and covers a distance of 7 meters. Now he takes a left turn and walks for 6 meters. Again he takes left turn and walks for 15 meters and reaches his office. Find the distance between his starting point and his office and in which direction is the person’s office from the starting point?
    A. 10m, North east B. 10m, North west C. 10m, South east D. 10m, South west

Answer - Option D
Explanation :

[latex]{8}^{2}[/latex] + [latex]{6}^{2}[/latex] = 64 +36 = 100
Square root of 100 = 10m
His office is in south west direction from the starting point.
10. Direction: Study the information given below and answer the questions based on it. ‘Y+Z’ means ‘Z is at 4 meter to the east of Y’ ‘Y-Z’ means ‘Z is at 4 meter to the west of Y’ ‘Y×Z’ means ‘Z is at 4 meter to the North of Y’ ‘Y÷Z’ means ‘Z is at 4 meter to the South of Y’
    A. North B. South-West C. North-East D. South

Answer - Option B
Explanation :

A ÷ B - C
‘A ÷ B’ means ‘B is at 4 meter to the South of A’
‘B - C’ means ‘C is at 4 meter to the left of B’
1. Vishal travels 8 km towards east, then turns left and travels another 5 km, and then he turns 270 degrees anticlockwise. Which direction is he facing now?
    A. West B. East C. North D. South

Answer - Option B
Explanation :


Thus Vishal is now facing towards East.
2. Ram and Shyam started from a fixed place. Ram moves 3 km to the North and turns right, then walks 4 km. Shyam moves towards West and walks 5 km, then turns to right and walks 3 km. How far Ram is from Shyam?
    A. 13 km B. 16 km C. 9 km D. 10 km

Answer - Option C
Explanation :

Diagram shows the position of Ram and Shyam from starting point O.
So 5 + 4 = 9 km
3. Medha moves towards South-East for 7 km, then moves towards West for 14 km. Then she moves towards North-West for 7 km and finally moves a distance of 4 km East find the distance of starting point and from where she stands now?
    A. 4 km B. 6 km C. 8 km D. 10 km

Answer - Option D
Explanation :
Let Medha starts from point A and walks towards South-East for 7 km and reaches at B, then she walks towards West direction for 14 Km and reaches at C, now she towards North-West for 7 km and reaches at D. Then she moves a distance of 4 km East and reaches at E.


From the figurative representation of Medha’s journey it is clear that finally Medha reaches at E which is in same level as A. The distance between A and E is (14 - 4= 10 KM)
4. Ravi in his evening walk travelled towards the sun 3 km, then he turned to his left walked 2 km. He again turned to his right and walked 3 km. Finally, he turned to his right walked another 2 km. In which direction is Ravi walking now?
    A. East B. West C. North D. South

Answer - Option C
Explanation :
Starting from A, Ravi in his evening walk travelled towards the sun 3 km (i.e. towards west as direction of sun in evening is west direction) up to B then he turned to his left (i.e. south direction) walked 2 km up to C. He again turned to his right (i.e. in west direction) and walked 3 km up to D. Finally, he turned to his right (i.e. towards north direction) walked another 2 km up to E.


Clearly Ravi is walking in north direction Finally.
Hence option C is the right answer.
5. Kalpana travelled from point B to straight to C a distance of 8 feet She turned left and walked 5 feet away. Again she turned left and walked 7 feet and finally turned left and walked 5 feet. How far is she from the starting point?
    A. 3 feet B. 4 feet C. 1 feet D. 5 feet

Answer - Option C
Explanation :
Let we draw the figure of the motion of Kalpana as given in the question:

Finally Kalpana reaches at F. Now the distance of Kalpana from she started = BF = (BC – FC) = (BC – AD) = (8 – 7) = 1feet
6. Direction: Raghu starts from his house in his car and travels 8 km towards the North, then 6 km towards East then 10 km towards his right, 4 km towards his left, 10 km towards North and finally 4 km towards his right. In which direction is he now with reference to the starting point?
    A. South B. North East C. South East D. North

Answer - Option B
Explanation :
Raghu starts from his house (A) in his car and travels 8 km towards the North and reaches at B, then 6 km towards East and reaches at C, then 10 km towards his right i.e. in south direction and reaches at D. Then he turns left i.e. in east direction and travels 4 km and reaches at E. Now he travels 10 km towards North to reach at F and finally 4 km towards his right i. e. in east direction and reaches at G.

He is now in North East direction with reference to the starting point.
7. A man is facing towards the east. He turns 270 degrees clockwise and then takes a right turn. Finally, he turns 90 degrees anticlockwise. Which direction is he facing now?
    A. West B. South C. North D. East

Answer - Option C
Explanation :
8. Prakash travelled 6 km north ward, then turned left and travelled 4 km, then turned left and travelled 6 km. How far was Prakash from the starting point?
    A. 6 m B. 4 km C. 10 km D. 8 km

Answer - Option B
Explanation :

Hence, he is 4 km far from the starting point.
9. Rama is standing at a point facing north. He walks 10 km straight, turns left and walks another 15 km straight and finally turns left and walks 10 km. How far is he from the starting points now?
    A. 10 km B. 5 km C. 12 km D. 15 km

Answer - Option D
Explanation :

10. Direction: Aman starts walking from his college, walks 10 km towards North, then he turns to his left and walks 10 km. From there he takes a right turn and walks 10 km. In which direction is he facing now?
    A. South B. North C. East D. West

Answer - Option B
Explanation :
Let Aman starts from point A and walks towards north 10 Km and reaches at B, then Ashok turns left (i. e. West direction) and walks 10 Km and reaches at C, now he takes a right turn (i.e. towards north) and walks 10 Km and reaches at D.

From the figurative representation of Aman’s journey it is clear that at last Aman is facing in north direction.
1. Introducing a man, a woman says, "His son's mother is the only daughter of my mother". How is the man related to the woman?
    A. Nephew B. Maternal uncle C. Husband D. Father

Answer - Option C
Explanation :
Only daughter [latex]\rightarrow [/latex] herself
Son’s mother [latex]\rightarrow [/latex] woman herself
Son’s father [latex]\rightarrow [/latex] woman’s husband
Hence, the correct option is C
2. Rahul said, ”The boy in that picture is the brother of the daughter of my paternal grandfather’s only son”. How is the boy in the picture related to Rahul?
    A. Cousin B. Father C. Nephew D. Brother

Answer - Option D
Explanation :
From the given information in the question,

Clearly, option D is the correct response.
3. ohnny’s father is the brother-in-law of Nisha’s only sister. How is Johnny related to Nisha ?
    A. Son B. Grandson C. Father D. Cousin

Answer - Option A
Explanation :
Nisha’s only sister brother-in-law will be Nisha’s husband.
And, Nisha’s husband is a father of Johnny.
Johnny is a son of Nisha.
Hence, the correct option is A
4. A man says to a lady, "Your father is the father in law of my father's only child." How is the lady related to the man?
    A. Sister in law B. Granddaughter C. Wife D. Daughter

Answer - Option C
Explanation :
A man says to a lady, "Your father is the father ­in ­law of my father's only child."
How is the lady related to the man?
Lady's Father [latex]\Rightarrow[/latex] Man's Father only child father in law.
Man's Father’s only child [latex]\Rightarrow[/latex] Man himself
Lady [latex]\Rightarrow[/latex] Man’s wife
Hence, the correct option is C.
5. Deepak has a brother named Aditya, Deepak is the son of Kuldeep. Bunty is Kuldeep’s father. How is Aditya related to Bunty?
    A. Uncle B. Brother C. Grandson D. Grandfather

Answer - Option C
Explanation :
Bunty’s son [latex]\Rightarrow[/latex] Kuldeep
Kuldeep’s son [latex]\Rightarrow[/latex] Deepak
Deepak’s grandfather [latex]\rightarrow [/latex] Bunty
Deepak’s brother [latex]\rightarrow [/latex] Aditya
Aditya’s grandfather [latex]\rightarrow [/latex] Bunty
i.e., Aditya is Grandson of Bunty.
Hence, the correct option is C.
6. Direction: Ravi’s father has a son Rohit who has an aunt Laxmi who has a husband Rao whose father-in law is Mohan. What is the relation of Mohan to Ravi?
    A. Nephew B. Grandfather C. Son D. Uncle

Answer - Option B
Explanation :
Rao is uncle of Rohit and Ravi. Therefore, Mohan is Grandfather of Ravi.
7. Pointing to a lady Simon said, “She is the daughter of the only sister of my father”. How is lady related to Simon?
    A. Mother B. Aunt C. Sister D. Cousin

Answer - Option D
Explanation :
Pointing to a lady Simon said, “She is the daughter of the only sister of my father”.
Hence sister of his father will be ‘Aunt’ of Simon.
Now it is clear that the pointed lady of Simon is daughter of his Aunt, therefore the pointed lady will be cousin sister of Simon.
8. Pointing to Mala, Kala said. “She is my brother’s only sister’s daughter.” How is Mala related to Kala? Note: Kala is a female.
    A. Mother B. Daughter C. Aunt D. Niece or daughter

Answer - Option B
Explanation :
Only sister of Kala’s brother means Kala herself. Kala is a female.
So, Mala is the daughter of Kala.
9. Pointing towards a woman in a photograph Vijay said, “She is the daughter of the father of sister of my brother”. How is the lady in the photograph related to Vijay?
    A. Wife B. Mother C. Sister D. Daughter

Answer - Option C
Explanation :
Vijay pointed a photograph of a woman and said “She is the daughter of the father of sister of my brother”
The sister of Vijay’s brother is sister of Vijay too.
Father of Vijay’s sister is father of Vijay too.
Now daughter of Vijay’s father would be sister of Vijay.
10. Babita is Ajay’s wife and Chiranjiv is Divya’s father. If Ajay’s mother-in-law is wife of Divya’s grandfather, then how is Babita related to Chiranjiv’s wife?
    A. Sister-in-law B. Sister C. Cousin D. Nephew

Answer - Option A
Explanation :
Babita is the wife of Ajay.
Chiranjiv is father of Divya.
Wife of Divya's grandfather means grandmother of Divya.
Grandmother of Divya is mother-in-law of Ajay.
Therefore, Babita is daughter of Divya's grandmother.
Chiranjiv is brother of Babita.
Therefore, Babita is sister-in ­law of Chiranjiv's wife.
Hence, option A is the right answer.
1. Direction: Read the following information carefully to answer the following questions. There are six persons C, D, E, F, G and H in a family. There are two fathers and a mother in the family. E is the sister of H. D is the brother of G’s husband. F is the father of C and grandfather of H. G has only one daughter.
    A. C B. E C. G D. H

Answer - Option C
Explanation :
E is the sister of H. F is the father of C and grandfather of H. D is the brother of G’s husband. As there are two fathers and a mother in the family so C must be husband of G and D is brother of C.

G is mother in the family.
Hence, option C
2. Direction: Read the following information carefully and answer the questions given below. There are seven members A, C, D, E, F, G and H in a family. There are two fathers, one mother, two sisters and four brothers. E is a sister-in-law of D. G is a daughter of C. F is the brother of E. A is a grandfather of G. E is a mother of H.
    A. Grandson B. Granddaughter C. Son D. Cannot be determined

Answer - Option A
Explanation :

3. Direction: Study the information given below and answer the questions based on it. ‘A $ B’ means ‘A is brother of B’ ‘A + B’ means ‘B is sister of A’ ‘A & B’ means ‘B is wife of A’ ‘A @ B’ means ‘A is mother of B’ ‘A ^ B’ means ‘A is father of B’ In expression ‘K & L @ M + N + O’, how is O related to K?
    A. Daughter-in-law B. Son-in-law C. Son D. Daughter

Answer - Option D
Explanation :

4. Directions: Study the following information carefully and answer the questions given below: In a family there are 12 members having three generation. There are five married couple in the family. And there are six male members. M has two children only one is male. B is husband of C who sister in law of D who daughter of N. P is husband of Q who mother in law of B who brother in law of A. C daughter in law of M who maternal grandfather of E. P has two children. K daughter in law of Q and wife of H. C aunty of F.
    A. Grandson B. Cannot be determined C. Daughter in law D. Son in law

Answer - Option B
Explanation :
M has two children only one is male. B is husband of C who sister in law of D who daughter in law of N.

P is husband of Q who mother in law of B who brother in law of A. C daughter in law of M who maternal grandfather of E.


5. Direction: Study the following information carefully and answer the questions given beside: A family has 6 members – Radhey, Krishna, Madhav, Kanha, Gaur and Hari among 3 generations. Further it is also known that: Radhey is the son-in-law of Krishna. Gaur who is unmarried, has a sister and Hari has an uncle. Kanha is the grandmother of Hari. Both the grandparents and parents of Hari are alive.
    A. Madhav B. Krishna C. Gaur D. Radhey

Answer - Option B
6. A family consisted of a man, his wife, his three sons, their wives and three children in each son's family. How many members are there in the family?
    A. 12 B. 13 C. 15 D. 17

Answer - Option D
Explanation :
The man and his wife = 2 members
Three sons and their wives=6 members
Three children each of the three sons = 3 × 3 = 9 members
Total number of members = 2 + 6 + 9
= 17 members.
7. A and B are a married couple. C and D are brothers. C is the brother of A. How is D related to B?
    A. Brother-in-law B. Brother C. Son-in-law D. Cousin

Answer - Option A
Explanation :
Since, A and B are a married couple. C and D are brothers. C is the brother of A. Therefore, D is also brother of A. Thus, D is
Brother-in-law of B.
Hence, A is correct.
8. Introducing a man to her husband, a woman said, "His brother's father is the only son of my grandfather". How is the woman related to the man?
    A. Mother-in-law B. Sister C. Daughter D. Sister-in-law

Answer - Option B
Explanation :
The only son of women’s grandfather will be women’s father who is the father of man’s brother and therefore of man also. Thus, the women is the sister of that man.
9. Direction: Study the information given below and answer the questions based on it. There are 7 members in a three-generation family. M is niece of N. O is sister-in-law of P. Q is sister of R’s paternal grandmother’s only son. N and P are not related by blood. S has only one granddaughter. How R is related to M?
    A. Father B. Mother C. Cousin D. Son

Answer - Option C
Explanation :
Q is sister of R’s grandmother’s only son. Q must be S’s daughter. Since N and P are not blood relatives and O is the sister in law of P so Q must be P’s wife and N must be O’s husband. M must be Q’s daughter and since S has only one granddaughter so R must be N’s Son.

10. Direction: Study the following information carefully and answer the question given below: a + b means a is the sister of b a - b means a is the brother of b a × b means a is the daughter of b a ÷ b means a is the mother of b
    A. l ÷m × n B. l - m × n C. l + m × n D. l + m ÷ n

Answer - Option A
Explanation :

As clear from the above diagram l and n are wife and husband.
1. If [latex]{5}^{th}[/latex] January 2011 was a Thursday, then what day of the week was it on [latex]{1}^{st}[/latex] January 2013?
    A. Thursday B. Wednesday C. Tuesday D. Monday

Answer - Option B
Explanation : Since [latex]{5}^{th}[/latex] of January, 2011 was a Thursday so,
05/01/2011: Thursday
05/01/2012: Friday
05/01/2013: Sunday………..(2012 was a leap year)
04/01/2013: Saturday
03/01/2013: Friday
02/01/2013: Thursday
01/01/2013: Wednesday
Hence, the correct option is B.
Note :
When we move to the same date next year there is an increase of one day i.e
1 January if there is friday then on the next year, 1 January will be saturday
But when there is a leap year( particularly 29 february in between the dates), then the day will increase by two days i.e. 1 January if there is friday then on the next year, 1 January will be sunday
2. If the day before yesterday was Wednesday, when will Sunday be?
    A. Today B. Tomorrow C. Day after tomorrow D. Two days after tomorrow

Answer - Option C
Explanation :
The day before yesterday was Wednesday

Now from the table it is clear that Sunday will be ‘day after tomorrow’.
3. If January 1st is a Friday, what is the first day of the month of March in a leap year?
    A. Thursday B. Friday C. Tuesday D. Wednesday

Answer - Option C
Explanation :
If the first January is Friday then the first February would be Monday.
[latex]{29}^{th}[/latex] February => Monday
Therefore;
[latex]{1}^{st}[/latex] March => Tuesday.
Hence Option C is correct
4. If the day before yesterday was Friday, what day will two days after the day after tomorrow be?
    A. Saturday B. Thursday C. Friday D. Sunday

Answer - Option B
Explanation :
If the day before yesterday was Friday, it means yesterday was Saturday and today is Sunday. Tomorrow will be Monday day after tomorrow will be Tuesday and two days after the day after tomorrow will be Thursday.

5. For a certain month, the dates of three of the Sundays are even numbers. Then, the [latex]{15}^{th}[/latex] of that month falls on a
    A. Thursday B. Friday C. Saturday D. Sunday

Answer - Option C
Explanation :
A day of week repeats itself after every 7 days. So there will be alternate odd and even dates on sundays. So, for there to be 3 sundays on even dates there must be 5 sundays in a week with the first sunday starting with even dates. Therefore, if a Sunday lies on 2nd of a month, other Sundays will lie on 9, 16, 23, 30. Hence, [latex]{15}^{th}[/latex] will fall on Saturday.
6. If day before yesterday was Wednesday, when will Sunday be?
    A. 3 days after today B. Tomorrow C. Today D. Day after tomorrow

Answer - Option D
7. Lisa was born on 1 March 2016. When will she be 5,000 days old?
    A. 6 September 2029 B. 18 August 2029 C. 8 November 2029 D. 29 November 2029

Answer - Option C
Explanation :
5000 days = 5000/365 = 13.69 years i.e. more than 13 years
13 years = 366*3 + 365*10 = 1098 + 3650 = 4748 days
Lisa will be 4748 days old on 1 March 2029
252 days from March 1 Ã 30 + 30 + 31 + 30 + 31 + 31 + 30 + 31 + 8 = 8 November 2029
8. Mishti's birthday is on Thursday 27th April. On what day of the week will be Aradhya's Birthday in the same year, if Aradhya was born on 20th October?
    A. Friday B. Wednesday C. Saturday D. Thursday

Answer - Option A
9. At what time between 8 and 9 o’clock will the minute hand and the hour hand are on the same straight line but facing opposite directions.
    A. 8hrs 9 minutes [latex]{\frac {9}{10}}^{th}[/latex] minute. B. 8hrs 10 minutes [latex]{\frac {10}{11}}^{th}[/latex] minute. C. 8hrs 11 minutes [latex]{\frac {11}{12}}^{th}[/latex] minute. D. 8hrs 12 minutes [latex]{\frac {11}{13}}^{th}[/latex] minute.

Answer - Option B
Explanation :
On straight line means 180 degree angle.
Taking 12 on a clock as reference
The hour hand moves 30 degree with each hour and minute hand move 6 degree with each minute
Since 1 hour has 60 minutes, hour hand moves [latex] \frac {60}{30}[/latex] degree i.e 0.5 degree with each minute.
Therefore
At 8 o’clock hour hand is at 240 degrees from the reference and minute hand is at 0 degree from the reference.
In 't' minutes
The Hour hand is at= 240 + 0.5t degrees
The Minute hand is at = 6t degree
Here,
(240 + 0.5t) - 6t = 180
Solving,
240 - 5.5t = 180
Or 5.5t = 60
Therefore t= [latex]\frac {60}{5.5}[/latex] = [latex]10 \frac {10}{11}[/latex]
Hence the required time, 8hrs 10 minutes [latex] \frac {10}{11}[/latex] th minute.
10. If it is Saturday on [latex]{27}^{th}[/latex] September, what day will it be on [latex]{27}^{th}[/latex] October of the same year?
    A. Thursday B. Sunday C. Friday D. Monday

Answer - Option D
Explanation :
Since, September has 30 days.
If [latex]{27}^{th}[/latex] is Saturday in the month of the September.
Therefore, 3 days left in September + 27 days in October = 30 days
Thus, [latex]{28}^{th}[/latex] day i.e, [latex]{25}^{th}[/latex] of October will be Saturday and hence, [latex]{27}^{th}[/latex] October will be the Monday.
Hence, D is correct.
1. Akshar remembers that the match is after [latex]{26}^{th}[/latex] April but before [latex]{30}^{th}[/latex] April, while Suresh remembers that the match is after [latex]{22}^{nd}[/latex] April but before [latex]{28}^{th}[/latex] April. On which date of April is the match?
    A. 29 B. 26 C. 27 D. 28

Answer - Option C
Explanation :
According to Akshar Date of match may be one of [latex]\frac {27}{4}[/latex], [latex]\frac {28}{4}[/latex], [latex]\frac {29}{4}[/latex],
According to Suresh Date of match may be one of [latex]\frac {23}{4}[/latex], [latex]\frac {24}{4}[/latex], [latex]\frac {25}{4}[/latex], [latex]\frac {26}{4}[/latex], [latex]\frac {27}{4}[/latex],
Common date is [latex]\frac {27}{4}[/latex].
Thus option C is the correct answer.
2. Divit's birthday is on Friday [latex]{30}^{th}[/latex] June. On what day of the week will be Samar's Birthday in the same year if Samar was born on 15th November?
    A. Tuesday B. Friday C. Wednesday D. Saturday

Answer - Option C
Explanation :

Days left in June = 30 – 30 = 0
Days in November to count = 15
Thus total number of days from Divit's birthday to Samar’s birthday = 31 + 31 + 30 + 31 + 15 = 138
Thus number of odd days = 138 | 7 = 5 ...........(applying
Modulo function which gives the remainder of the division. This is used here to separate the number of odd days from number of weeks.)
As Divit's birthday is on Friday, Samar’s birthday will on Friday + 5 days = Wednesday.
3. Akarsh's birthday is on Saturday [latex]{29}^{th}[/latex] July. On what day of the week will be Ojas's Birthday in the same year if Ojas was born on [latex]{12}^{th}[/latex] August?
    A. Wednesday B. Friday C. Saturday D. Sunday

Answer - Option C
Explanation :
Days left in July = 31 – 29 = 2
Days in August to count = 12
Thus total number of days from Akarsh's birthday to Ojas’s birthday = 2 + 12 = 14
Thus number of odd days = 14 | 7 = 0
As Akarsh's birthday is on Saturday, Ojas’s’s birthday will be on Saturday + 0 days = Saturday.
4. If reflection of the clock in mirror shows time as 6:55, what would be the actual time?
    A. 7:05 B. 7:55 C. 5:05 D. 5:55

Answer - Option C
Explanation :

Thus, the actual timing is 5:05
Hence, option C is the correct response.
5. Sam's birthday is on Friday [latex]{17}^{th}[/latex] May. On what day of the week will be Renu's birthday in the same year if Renu was born on [latex]{21}^{st}[/latex] November?
    A. Friday B. Thursday C. Tuesday D. Sunday

Answer - Option B
Explanation :

Days left in May = 31 – 17 = 14
Days in November to count = 21
Thus total number of days from Sam's birthday to Renu’s birthday = 14 + 30 + 31 + 31 + 30 + 31 + 21 = 188
Thus number of odd days = 188 | 7 = 6
As Sam's birthday is on Friday, Renu’s birthday will on Friday + 6 days = Thursday.
6. Priya's birthday is on Tuesday [latex]{11}^{th}[/latex] April. On what day of the week will be Rani's birthday in the same year if Rani was born on 31st August?
    A. Monday B. Wednesday C. Tuesday D. Thursday

Answer - Option D
Explanation :

Days left in April = 30 – 11 = 19
Thus total number of days from Priya's birthday to Rani’s birthday = 19 + 31 + 30 + 31 + 31 = 142
Thus number of odd days = 142 | 7 = 2
As Priya's birthday is on Tuesday, Rani’s birthday will on Tuesday + 2 days = Thursday.
7. Satya's birthday falls on [latex]{15}^{th}[/latex] August and Meena's birthday falls on [latex]{25}^{th}[/latex] June. If Meena's birthday was on Wednesday, what was the day on Sathya's birthday in the same year?
    A. Friday B. Monday C. Tuesday D. Saturday

Answer - Option A
Explanation :
For an easy approach start counting as the difference between the months is quite small. If we count from [latex]{25}^{th}[/latex] June then after 7 days Wednesday would be repeated
We’ll have Wednesdays on 2, 9, 16, 23, 30, 6, [latex]{13}^{th}[/latex] August
Now [latex]{13}^{th}[/latex] + 2= [latex]{15}^{th}[/latex] August
Hence Friday
Hence A
8. If [latex]{13}^{th}[/latex] August 2005 is a Saturday then what is the day on [latex]{19}^{th}[/latex] May 2013?
    A. Monday B. Friday C. Wednesday D. Sunday

Answer - Option D
Explanation :
When date, month and year are different then first we find the odd days in year i.e.-
2013 - 2005 = [latex]\frac{(8 year + leap year)}{7}[/latex]
leap year are 2008 , 2012 = 2 leap year
So odd day = [latex]\frac{(8 + 2)}{7}[/latex] = [latex]\frac{10}{7}[/latex]= 3 odd days
The given year is ascending order then odd days is +3.
Now we find odd days in month i.e.-
month – May - June – July - August
days - 12 - 30 - 31 - 13
odd days - 5 - 2 - 3 - 6
total odd day= 16/7=2 week 2 odd days
month is descending order so odd days is -2.
total odd day is = (+3)+(-2)
= +1
Saturday , Sunday (+1st odd day)
Answer is Sunday.
9. If [latex]{12}^{th}[/latex] August 2014 is a Tuesday then what is the day on [latex]{24}^{th}[/latex] October 2014?
    A. Monday B. Friday C. Thursday D. Sunday

Answer - Option B
Explanation :
When month and date are different and year is same then we count odd days in ascending order in months from august to October and starting month date is subtracted from given date i.e.-
31 - 12 = 19
Ending month date is written as it is i.e.- 24
Now we find odd days-
August - September - October
Days - 19 - 30 - 24
odd days - 5 - 2 - 3
total odd days is 5 + 2 + 3 = [latex]\frac {10}{7}[/latex] = 1 week 3 odd days
In given month is ascending order so we take positive(+) i.e.-
Tuesday , Wednesday , Thursday , Friday
1st odd day 2nd odd day 3rd odd day
So answer is friday.
10. Which of the following years did not have 29 days in February month?
    A. 2000 B. 2004 C. 1996 D. 1996

Answer - Option D
Explanation :
A year in which 29 day are there in February month is said to be leap year. That year is perfectly divisible by 4.
As year 1966 is not divisible by 4 hence in year 1966 there are not 29 days in February month. ([latex]\frac {1966}{4}[/latex] = 491.5)
1. Direction: Each of the questions below consists of a question and two statements numbered I and II given below it. You have to decide whether the data provided in the statements are sufficient to answer the questions. Read both the statements and Give Answer City F is towards which direction of City D? I. City D is towards south of City of K which is towards west of City F II. City M is towards south of City R and towards east of City D
    A. the data in statement I alone are sufficient to answer the question, while the data in statement II alone are not sufficient in answer the question. B. the data in statement II alone are sufficient to answer the question, while the data in statement I alone are not sufficient to answer the question. C. the data in either in statement I alone or in statement II alone are sufficient to answer the question D. the data in both the statements I and II together are not sufficient to answer the question.

Answer - Option A
2. Direction: Each of the questions below consists of a question and three statements numbered I, and II given below it. You have to decide whether the data provide in the statements are sufficient to answer the question. Read all the three statements and give answer. Among five friends A, B, C, D and E live in the five floors building, who is in the middle of the building? I. B is between C and D neither of whom is at the top and bottom floors. II. A and E are at the top and bottom ends and C is to the immediate down floor of A.
    A. If data in both statement I and II together are necessary to answer the questions. B. If data in statements II sufficient to answer the question, while the data in statement I are not required to answer the question C. If data in statements either I or II are sufficient to answer the question D. If data in statements I sufficient to answer the question while the data in statement II are not required to answer the question.

Answer - Option D
3. Direction: The question below consists of a question and two statements numbered I and II given below it. You have to decide whether the data provided in which of the statements are sufficient to answer the question. Choose your answer from the options based on this. H is the mother of T. How is T related to W? I. W is the only daughter of H. II. W is the sister of T.
    A. The data in statement I alone is sufficient to answer the question, while the data in statement II alone is not sufficient to answer the question. B. The data in statement II alone is sufficient to answer the question, while the data in statement II alone is not sufficient to answer the question. C. The data either in statement I alone or in statement II alone is sufficient to answer the question. D. The data in both the statement I and II together is not sufficient to answer the question.

Answer - Option A
Explanation : I. W is the only daughter of H.
Since, W is the only daughter, so T is the son and T is the brother of W.
II. W is the sister of T.

So, T may be Sister or Brother of W,
So, data in Statement I alone are sufficient to answer the question.
4. Direction: The question below consists of a question and two statements numbered I and II given below it. You have to decide whether the data provided in which of the statements are sufficient to answer the question. Choose your answer from the options based on this. Six persons Akhilesh, Bhavesh, Cindy, Divakar, Erika and Farheen have different heights. Who among the following is the tallest person? I. Only two persons are taller than Bhavesh but only one person is shorter than Erika. Akhilesh is taller than Cindy but not the tallest. II. Divakar is taller than Akhilesh but shorter than Farheen. Akhilesh is taller than Cindy but shorter than Bhavesh.
    A. The data in statement I alone is sufficient to answer the question, while the data in statement II alone is not sufficient to answer the question. B. The data in statement II alone is sufficient to answer the question, while the data in statement I alone is not sufficient to answer the question. C. The data either in statement I alone or in statement II alone is sufficient to answer the question. D. The data in both the statements I and II together are necessary to answer the question

Answer - Option D
Explanation :
Only two persons are taller than B but only one person is shorter than E.
__>__>Bhavesh>__>Erika>__
Akhilesh is taller than Cindy but shorter than Bhavesh.
__>__>Bhavesh>Akhilesh>Erika>Cindy
Divakar is taller than Akhilesh but shorter than Farheen.
Farheen>Divakar>Bhavesh>Akhilesh>Erika>Cindy
So, Statement I and II together are necessary to answer the question.
Hence, option E.
5. Direction: The question below consists of a question and three statements numbered I,II and III given. You have to decide whether the data provided in which of the statements are sufficient to answer the question. Choose your answer from the options based on this. Four friends are sitting around a rectangle table named A, B, C and D . B is sitting opposite to whom? I. A is sitting immediate of B and D . C is sitting immediate of B . D is sitting immediate left of A II. C is sitting second to right of A . A is facing center . D is sitting immediate right of C and A III. B is sitting immediate left of C . D is not sitting immediate of B . D is sitting immediate right of A
    A. Data in statement I alone or in the statement II alone or in the statement III alone is sufficient to answer the question B. Data in statement II and III are sufficient to answer the question, while the data in statement I is not sufficient to answer the question. C. Data in all the statement I, II and III are necessary to answer the question. D. Data in statement I and III are sufficient to answer the question, while the data in statement II is not sufficient to answer the question.

Answer - Option A
Explanation :

So from all 3 statements alone we can get that to whom B is sitting opposite to.
6. Direction: Each of the questions below consists of a question and two statements numbered I and II given below it. You have to decide whether the data provided in which of the statements are sufficient to answer the question. Choose your answer from the options based on this. Seven persons M, N, O, P, Q, R and S are having different ages. Who among the following is the youngest person in the group? I. O is elder to S but younger to R. Only two persons are elder to M. R is younger to P. P is not the eldest person. II. N is elder to M. O is elder to Q. M is elder to O
    A. The data in statement I alone are sufficient to answer the question, while the data in statement II alone are not sufficient to answer the question. B. The data in statement II alone are sufficient to answer the question, while the data in statement I alone are not sufficient to answer the question. C. The data either in statement I alone or in statement II alone are sufficient to answer the question. D. The data in both the statements I and II together are not sufficient to answer the question.

Answer - Option D
Explanation :
Only two persons are elder to M. N is elder to M. O is elder to S but younger to R. R is younger to P. P is not the eldest person. M is elder to O.
We get this-
N > P > M > R > O > S/Q
So statement I and II together are not sufficient to answer.
Hence, option D.
7. Direction: Each of the questions below consists of a question and two statements numbered I and II given below it. You have to decide whether the data provided in which of the statements are sufficient to answer the question. Choose your answer from the options based on this. In which of the following direction is point B with respect to point F? I. Point B is 6m to the west of point A. Point E is 6m to the west of point F. Point C is 3m to the west of point D. II. Point G is 6m to the north of point F. Point B is 4m to the north of point C. Point D is 4m to the north of point E.
    A. The data in statement I alone are sufficient to answer the question, while the data in statement II alone are not sufficient to answer the question. B. The data in statement II alone are sufficient to answer the question, while the data in statement I alone are not sufficient to answer the question. C. The data either in statement I alone or in statement II alone are sufficient to answer the question D. The data in both the statements I and II together are necessary to answer the question.

Answer - Option D
Explanation :

Clearly point B is in north-west of point F.
So statement I and II together are sufficient to answer.
Hence, option D.
8. Direction: Each of the questions below consists of a question and two statements numbered I and II given below it. You have to decide whether the data provided in which of the statements are sufficient to answer the question. Choose your answer from the options based on this. Seven persons G, M, P, S, T, U and V are sitting in a straight line and facing north. Who among the following sits exactly sits in the middle? I. S is 2nd to the left of G. Two persons are sitting between G and U. V is 2nd to the left of P. V is not neighbor of S. II. The number of persons are sitting between V and G is same as P and M. M is not neighbor of V or P.
    A. The data in statement I alone are sufficient to answer the question, while the data in statement II alone are not sufficient to answer the question. B. The data in statement II alone are sufficient to answer the question, while the data in statement I alone are not sufficient to answer the question. C. The data either in statement I alone or in statement II alone are sufficient to answer the question. D. The data in both the statements I and II together are not sufficient to answer the question.

Answer - Option D
9. Direction: Each of the questions below consists of a question and two statements numbered I and II given below it. You have to decide whether the data provided in which of the statements are sufficient to answer the question. Choose your answer from the options based on this. What is the code of “chair”? I. In a certain language, “video fan table” is written as “4 5 2” and “table chair fan” is written as “4 2 1”. II. In a certain language, “chair car table” is written as “4 6 1” and “bike pen fan” is written as “3 2 7”.
    A. The data in statement I alone are sufficient to answer the question, while the data in statement II alone are not sufficient to answer the question. B. The data in statement II alone are sufficient to answer the question, while the data in statement I alone are not sufficient to answer the question. C. The data either in statement I alone or in statement II alone are sufficient to answer the question. D. The data in both the statements I and II together are not sufficient to answer the question.

Answer - Option A
Explanation :
The code of “[latex]\frac {table}{fan}[/latex] = [latex]\frac {4}{2}[/latex]”.
Then the code of “chair” is “1”.
So statement I alone is sufficient to answer.
Hence, option A.
10. Direction: The following question below consists of a question and two statements numbered I and II given below it. You have to decide whether the data provided in which of the statements are sufficient to answer the question. Choose your answer from the options based on this. Six persons P, Q, R, S, T and U are staying on different floor. Lowermost floor is numbered 1 and topmost floor is numbered 6. Who among the following lives on 1st floor? I. P lives above U on an even numbered floor. R lives above T. Two persons are staying between S and R. II. Q lives above S on even numbered floor. One person is staying between U and T. S lives above R.
    A. If the data in statement I alone are sufficient to answer the question, while the data in statement II alone are not sufficient to answer the question. B. If the data in statement II alone are sufficient to answer the question, while the data in statement I alone are not sufficient to answer the question. C. If the data either in statement I alone or in statement II alone are sufficient to answer the question D. If the data in both the statements I and II together are necessary to answer the question.

Answer - Option D
1. Direction: The following question below consists of a question and two statements numbered I and II given below it. You have to decide whether the data provided in which of the statements are sufficient to answer the question. Choose your answer from the options based on this. Seven persons A, B, C, D, E, F and G are sitting in a straight line and facing north. Who among the following is sitting exactly in the middle? I. A is 2nd to the left of G. Two persons are sitting between G and B. D is 2nd to the left of B. II. B is 2nd to the right of D. E is 2nd to the left of C. Two persons are sitting between F and E.
    A. If the data in statement I alone are sufficient to answer the question, while the data in statement II alone are not sufficient to answer the question. B. If the data in statement II alone are sufficient to answer the question, while the data in statement I alone are not sufficient to answer the question. C. If the data either in statement I alone or in statement II alone are sufficient to answer the question. D. If the data in both the statements I and II together are not sufficient to answer the question

Answer - Option D
Explanation :
A is 2nd to the left of G. Two persons are sitting between G and B. D is 2nd to the left of B.

E is 2nd to the left of C. Two persons are sitting between F and E.

Either A or D in the middle.
So statement I and II are together are not necessary to answer the question.
Hence, option D.
2. Direction: In the question below consists of a question and two statements numbered I and II given below it. You have to decide whether the data provided in the statements are sufficient to answer the question. Five persons- A, B, C, D and E-are sitting in a circle facing the centre. Who is sitting to the immediate left of D? I. C is sitting second to the left of A. B and D are immediate neighbours of each other. II. D is sitting to the immediate left of B. E is not an immediate neighbour of D and B.
    A. The data in statement I alone are sufficient to answer the question, while the data in statement II alone are not sufficient to answer the question. B. The data in statement II alone are sufficient to answer the question, while the data in statement I alone are not sufficient to answer the question. C. The data either in statement I alone or in statement II alone are sufficient to answer the question. D. The data in both the statements I and II together are necessary to answer the question

Answer - Option D
Explanation :
From statement I- From statement II- From both the statements-

Hence, option E is correct.
3. Direction: The following below consists of a question and two statements numbered I and II below it. You have to decide whether the data provided in the statements are sufficient to answer the question. Read both the statements. On which day in July was definitely Mohan's mother's birthday? I. Mohan correctly remembers that his mother's birthday is before eighteenth but after twelfth July. II. Mohan's sister correctly remembers that their mother's birthday is after fifteenth but before nineteenth July.
    A. Data in statement I alone are sufficient to answer the question, while the data in statement II alone are not sufficient to answer the question B. Data in statement II alone are sufficient to answer the question, while the data in statement I alone are not sufficient to answer the question C. Data either in statement I alone or in statement II alone are sufficient to answer the question D. Data given in both the statement I & II together are not sufficient to answer the question

Answer - Option D
Explanation :
From Statement I:
Mohan’s mother’s birthday may be on 13th, 14th, 15th, 16th or 17th July.
From Statement II:
Mohan’s mother’s birthday may be on 16th, 17th or 18th July.
From both the statements:
Mohan’s mother’s birthday may be on 16th or 17th of July.
Hence Option D is correct
4. Directions: Each of the following below consists of a question and two statements numbered I and II below it. You have to decide whether the data provided in the statements are sufficient to answer the question. Read both the statements. How is 'near' written in a code language? I. 'go near the tree' is written as 'sa na pa ta' is that code language. II. 'tree is near home' is written as 'ja pa da sa' in that code language.
    A. Data in statement I alone are sufficient to answer the question, while the data in statement II alone are not sufficient to answer the question B. Data in statement II alone are sufficient to answer the question, while the data in statement I alone are not sufficient to answer the question C. Data either in statement I alone or in statement II alone are sufficient to answer the question D. Data given in both the statement I & II together are not sufficient to answer the question

Answer - Option D
5. Directions: Each of the questions below consists of a question and two statements numbered I and II given below it. You have to decide whether the data provided in the statements are sufficient to answer the question. On which date in November is definitely Chetan's father's birthday? I. Chetan correctly remembers that his father's birthday is before 22nd November but after 14th November. II. Chetan's brother correctly remembers that their father's birthday is after 19th November but before 28th November.
    A. The data in statement I alone are sufficient to answer the question, while the data in statement II alone are not sufficient to answer the question B. The data in statement II alone are sufficient to answer the question, while the data in statement I atone are not sufficient to answer the question. C. The data either in statement I alone or in statement II alone are sufficient to answer the question D. The data given in both the statements I and II together are not sufficient to answer the question.

Answer - Option D
Explanation :
From statement I, possible dates are 15th - 21st Nov.
From statement II, possible dates are 20th - 27th Nov.
Hence from statement I and II, we can conclude that possible dates are 20th or 21st Nov.
So both I and II are not sufficient.
6. Direction: Each of the questions below consists of a question, followed by two statements numbered I and II. You have to decide whether the data provided in the statements are sufficient to answer the question. Read both the statements and give the answer. Among A, B, C, D, E and F, who is the heaviest? I. B is heavier than A and E. II. F is heavier than B but lighter than D who is not the heaviest.
    A. The data in statement I alone are sufficient to answer the question, while the data in statement II alone are not sufficient to answer the question B. The data in statement II alone are sufficient to answer the question, while the data in statement I atone are not sufficient to answer the question. C. The data either in statement I alone or in statement II alone are sufficient to answer the question D. The data given in both the statements I and II together are not sufficient to answer the question.

Answer - Option D
Explanation :
Combining both, we get the order of their weights as C > D > F > B > A, E.
7. Directions: Each of the following questions below consists of question and two statements numbered I and II given below it. You have to decide whether the data provided in the statements are sufficient to answer the question. Read both the statements What was the grand total of Falcon Team of College ‘X’? I. Mayank correctly remembers that Falcon Team scored a grand total of above 82 but below 91. II. Animesh correctly remembers that Falcon Team scored a grand total of above 77 and below 84.
    A. The data in statement I alone are sufficient to answer the question, while the data in statement II alone are not sufficient to answer the question B. The data in statement II alone are sufficient to answer the question, while the data in statement I atone are not sufficient to answer the question. C. The data either in statement I alone or in statement II alone are sufficient to answer the question D. The data given in both the statements I and II together are not sufficient to answer the question.

Answer - Option D
Explanation :
From statement I:
Grand total of Falcon Team
= 83, 84, 85, 86, 87, 88, 89 or 90
From statement II:
Grand total of Falcon Team
= 78, 79, 80, 81, 82 or 83
From both the statements:
Grand total of Falcon Team = 83
8. Direction: The question below consists of a question and two statements numbered I and II given. You have to decide whether the data provided in which of the statements are sufficient to answer the question. Choose your answer from the options based on this. How is 'new' written in a code language? I. ‘new good clothes’ is written as '53 9' in that code language. II. good clothes are costly' is written as ' 9 6 7 3' in that code language.
    A. The data in statement I alone are sufficient to answer the question, while the data in statement II alone are not sufficient to answer the question B. The data in statement II alone are sufficient to answer the question, while the data in statement I atone are not sufficient to answer the question. C. The data either in statement I alone or in statement II alone are sufficient to answer the question D. The data given in both the statements I and II together are not sufficient to answer the question.

Answer - Option D
9. Direction: The question given below has few statements along with it. You have to determine which of the statement/s is/are sufficient/necessary for answering the question and mark your answer accordingly: What is the code for “writing paper”? I. “Read a paper” is written as “817”. Thinking and writing” is coded as “624”. II. “Writing with pen” is written as ‘’453’’. “Paper and pen” is written as ’’723”.
    A. The data in statement I alone are sufficient to answer the question, while the data in statement II alone are not sufficient to answer the question B. The data in statement II alone are sufficient to answer the question, while the data in statement I atone are not sufficient to answer the question. C. The data either in statement I alone or in statement II alone are sufficient to answer the question D. The data given in both the statements I and II together are not sufficient to answer the question.

Answer - Option D
Explanation :
From I. read a paper [latex]\rightarrow[/latex] 817 ...(a)
thinking and writing [latex]\rightarrow[/latex] 624 ...(b)
From II. writing with pen ? 453 ...(c)
Paper and pen [latex]\rightarrow[/latex] 723 ...(d)
From (c) and (d) pen [latex]\rightarrow[/latex] 3
Combining both (I) and (II), we have:
From (b) and (c), writing [latex]\rightarrow[/latex] 4
From (a) and (d), paper [latex]\rightarrow[/latex] 7
Hence both the statements I and II together are sufficient to answer the question.
10. Direction: Each of the question below consists of a question and two statements numbered I and II given below it. You have to decide whether the data provided in the statements are sufficient to answer the question. Is D the mother of S? I. L is the husband of D. L has only three children. II. N is the brother of S and P. P is the daughter of L
    A. The data in statement I alone are sufficient to answer the question, while the data in statement II alone are not sufficient to answer the question B. The data in statement II alone are sufficient to answer the question, while the data in statement I atone are not sufficient to answer the question. C. The data either in statement I alone or in statement II alone are sufficient to answer the question D. The data given in both the statements I and II together are not sufficient to answer the question.

Answer - Option D
Explanation :
From Statement I- L and D have only three children.
From statement II- N, P and S are children of L.
From both statements, N, P and S are children of L and D.
Therefore, D is the mother of N, P and S.
Hence, option D is correct.
1. Direction: In the following questions, the symbols d, %, *, @ and © are used with the following meaning as illustrated below: ‘P%Q’ means ‘P is not smaller than Q’. ‘P ©Q’ means ‘P is not greater than Q’. ‘PdQ’ means ‘P is neither greater than nor equal to Q’. ‘P@Q’ means ‘P is neither smaller than nor equal to Q’. ‘P*Q’ means ‘P is neither greater than nor smaller than Q’. Now in each of the following questions, assuming the given statements to be true, find which of the two conclusions I and II given below them is/are definitely true? Statement: M @ T, T % R, R * K Conclusion: I. K © T II. M @ K
    A. Only Conclusion I is true B. Only Conclusion II is true C. Either Conclusion I or II is true D. Both Conclusions I and II is true

Answer - Option D
Explanation :
M @ T = M > T
T % R = T = R
R * K = R = K
Therefore M > T = R = K
Conclusions are:
K © T = K = T (True)
M @ K = M > K (True)
Hence Option E is correct
2. Direction: In the following questions, the symbols *, $, d, © and % are used with the following meanings as illustrated below : 'PdQ' means 'P is neither greater than nor equal to Q'. 'P%Q' means 'P is neither smaller nor greater than Q'. 'P*Q' means 'P is not greater than Q'. 'P$Q' means 'P is greater than Q. 'P ©Q' means 'P is either greater or equal to Q'. Now, in each of the following questions, assuming the given statements to be true, find which of the two conclusions I and II given below them is/are definitely true. Statements: H*K, KdN, N$W Conclusions: I. N$H II. WdH
    A. Only Conclusion I is true B. Only Conclusion II is true C. Either Conclusion I or II is true D. Neither conclusion I nor II is true.

Answer - Option A
Explanation :
As given in the question,
d => <
% => =
* => =
$ => >
© => =
So,
H * K => H = K
K d N => K < N
N $ W =>N >W
Hence,
H = K W
Conclusions
I. N$H => N>H (TRUE)
II. WdH => W < H (Not True)
Therefore only conclusion I is true.
3. Which conclusion is definitely true? Given that M = N = O > P < Q is true. Conclusions: I. N = Q II. O Q IV. O < M
    A. Only II B. I and either II or III only C. Either II or III only D. None of these

Answer - Option D
Explanation :
From the given statement,
M = N = O > P < Q
No information is obtained about the relationship between Q and O or between N and Q.
Also, O>M is definitely false.
O can be greater than, lesser than or equal to Q. II and III cover only 2 conditions.
Hence none of the conclusions is true.
4. Direction: In each of these question, the relationships between two or more elements are shown in the statements. These statements are followed by two conclusions. Read the statements and give answer. Statement: A > B D = E > F Conclusion: I. B < A II. C = E
    A. Only Conclusion I is true B. Only Conclusion II is true C. Either Conclusion I or II is true D. Neither conclusion I nor II is true.

Answer - Option A
Explanation :
(i)A > B means B E. Thus, II does not follow.
5. Which of the following expressions has B>F and D>G definitely true?
    A. A<B=C<D=F=G<E B. G<A=C<DF<B C. E>C=B=D=G=A>F D. D=B=E=F<G=A<C

Answer - Option B
Explanation :
G<A=C<DF<B
6. Which of the following expressions is definitely true if the given expressions 'IH’ are definitely true?
    A. J>K=I>H>G B. J>K=I>H<G C. H>I=K>G=J D. G>H=I=K

Answer - Option D
Explanation :
In the expression G>H=I=K<J, both the conditions are fulfilled.
7. Direction: In the following question, some statements are followed by some conclusions. Assuming the given statements to be true, find which of the two conclusions follow(s) the given statements and choose appropriate answer choice. Statement: M > N = O = P, Q = R = S = P Conclusion: I. O = R II. M > S
    A. Only conclusion I is true. B. Only conclusion II is true. C. Either conclusion I or II is true. D. Both conclusion I and II are true.

Answer - Option D
Explanation :
I. O=P & P=R which means O=R
II. M>P & P=S which means M>S
8. In which of the following expressions will the expression ‘P < Q’ be definitely true?
    A. P = R > N = Q B. Q P C. P N D. P = N = M > Q

Answer - Option C
Explanation :
In P < R N, P < Q is definitely true.
Option C is the correct answer.
9. Directions: Now in each of the following questions, assuming the given statements to be true, find which of the two conclusions I and II given below them is / are definitely true. Statement: P = C > Y < S Conclusions: I. P = Y II. P > S
    A. If only conclusion I is true B. If only conclusion II is true. C. If either conclusion I or II is true D. If neither conclusion I nor II is true.

Answer - Option D
Explanation :
P > C > Y < S
Conclusions:
I. P > Y (false)
II. P > S (false)
neither conclusion I nor II is true
10. Direction: In the following question, some statements are followed by some conclusions. Assuming the given statements to be true, find which of the two conclusions follow(s) the given statements and choose appropriate answer choice. Statements: A = B > C = D, M L, P < W = D = F Conclusions: I. B > M II. F < A
    A. Only I follows B. Only II follows C. Either I or II follows D. Both I and II follow

Answer - Option D
Explanation :
Conclusions:
I. B > M (True as B > C = D = F = J > M)
II. F
C = D = F)
1. Direction: In the question, relationship between different elements is shown in the statements. These statements are followed by two conclusions. Find out which of the following conclusions follows. Statements: R< P=QM=B>D, A=G Conclusion: I. G>B II. Q>D III. A>P
    A. Only I is true B. Only II is true C. Only I and III are true D. Only I and II are true

Answer - Option C
Explanation :
After combining, we get
R< P = Q M = B > D
I. G>B, TRUE
II. Q>D NOT TRUE
III. A>P TRUE
2. Direction: In the following question, some statements are followed by two conclusions (I and II). Assuming the given statements to be true, find which of the two conclusions follow(s) the given statements and choose appropriate answer choice. Statements: M=GA, D>B Conclusions: I. G<C II. G<D
    A. Only conclusion I follows B. Only conclusion II follows C. Either conclusion I or conclusion II follows D. Both conclusion I and conclusion II follow

Answer - Option D
Explanation :
I. G<A<C
II. G<A=B=D(True)
3. Direction: Assuming the given statement/s to be true, find which of the conclusion among given conclusions is/are definitely true and then give your answers accordingly. Statements: M = Q; Q ? A; A = S; Z
T Conclusions: (i) Q = S (ii) M > A (iii) A > T
    A. None is true B. Only I and II are true C. Only II and III are true D. Only III and I are true

Answer - Option A
Explanation :
M = Q ? A = S;
U > T = A > Z
Q = S Not possible
M > A Not Possible
A > T Not Possible as there are no links
4. Direction: Assuming the given statement/s to be true, find which of the conclusion among given conclusions is/are definitely true and then give your answers accordingly. Statements: Z = X > K, D = K < P Conclusions: (i) D = Z (ii) X = P (iii) D < X
    A. Only I is true B. Only II is true C. Only III is true D. Both I and III are true

Answer - Option C
Explanation :
Z = X > K = D < P
(i) D = Z (not true)
(ii) X = P (not true)
(iii) D < X (true)
5. Direction: In the following question, some statements are followed by some conclusions. Assuming the given statements to be true, find which of the two conclusions follow the given statements and choose appropriate answer choice. Statements: E=A F=C Conclusions: I. E > B II. C<D
    A. Only conclusion I is true. B. Only conclusion II is true. C. Either conclusion I or II is true D. Neither conclusion I nor II is true

Answer - Option B
Explanation :
Statements: E = A F = C
The conclusion I do not hold true from E = A < D=B since E cannot be compared with B.
From D = B > F = C we can evaluate:
B > C and D=B hence D > C so conclusion II holds true.
6. Which of the following expressions will be true if the expression ‘H = G = K > F’ is definitely true?
    A. K =H B. G = F C. H < F D. F > G

Answer - Option A
7. Direction: Each question has one statement followed by three conclusion. You have to assume that given conclusions are true and according Fill the blanks in the statements. Statements. C = D = F ?J ? R = H ? T Conclusions. I. H = J II. D = H III. D > T If all the given conclusions are possibly true then fill in the blanks
    A. =, =, = B. =, =, > C. =, =, > D. =, =, =

Answer - Option C
Explanation :
'=, =, > '
Given statement - C = D = F ?J ? R = H ? T
After putting the sign we get,
C = D = F = J = R = H > T
Conclusions-
I. H = J is true.
II. D = H is true
III. D > T is true
8. Direction: In these questions, relationship between different elements is shown in the statements. These statements are followed by four conclusions. You have to decide which conclusion definitely follows from the statement. Statements: J > K, N < O, L = M, K = L, M = N Conclusions: I. J > N II. K > N III. O > L IV. L > J
    A. Only I and IV B. Only I and II C. Only III and IV D. Only I

Answer - Option D
Explanation :
J > K = L = M = N < O
9. Find the odd one out
    A. 100 B. 121 C. 125 D. 144

Answer - Option C
Explanation :
All others are the squares of the consecutive natural numbers that is 10, 11, 12 except 125 which is the cube of 5.
Hence Option C is correct
10. Direction: A series is given with one term missing. Select the correct alternative from the given ones that will complete the series. PR, VX, BD, ?
    A. EF B. HI C. HJ D. HK

Answer - Option C
Explanation :

Thus the next term will be HJ.
1. In which of the following expressions will the expression ‘P < F’ be definitely false?
    A. F = B > P = M B. P > B = M = F C. P = B < F = M D. B < P = M < F

Answer - Option B
Explanation :
P > B = M = F OR P > F
So, P < F is definitely false
2. Direction: In these questions, relationship between different elements is shown in the statements. These statements are followed by two conclusions. Statements: P = R, Y = T, U = V Conclusions: I. V > U, II. Y = R
    A. If only conclusion I is true. B. If only conclusion II is true C. If either conclusion I or II is true D. If neither conclusion I nor II is true

Answer - Option D
Explanation :
Since no information is given regarding the relation between Y and R as well as V and U.
3. Direction: In these questions, relationship between different elements is shown in the statements. These statements are followed by two conclusions. Statements: P = D, D > Z, D = X Conclusions: I. D = P, II. P = Z
    A. If only conclusion I is true. B. If only conclusion II is true C. If either conclusion I or II is true D. If neither conclusion I nor II is true

Answer - Option A
Explanation :
P = D > Z, D = X
I. D = P, which is true
II. P = Z is false since no relation can be established between P and Z.
4. Direction: In these questions, relationship between different elements is shown in the statements. These statements are followed by two conclusions. Statements: L = M, N = O, N = A Conclusions: I. N = M, II. O = M
    A. If only conclusion I is true. B. If only conclusion II is true C. If either conclusion I or II is true D. If neither conclusion I nor II is true

Answer - Option D
Explanation :
L = M, O = N = A
No information is given regarding the relation either between N and M or between O and M. Hence both conclusions I and II are not necessarily true.
5. Direction: In these questions, relationship between different elements is shown in the statements. These statements are followed by two conclusions. Statements: V Y, Z = V, Y = Z Conclusions: I. Y = V II. Y > V
    A. only conclusion I follows. B. only conclusion II follows. C. either conclusion I or conclusion II follows. D. neither conclusion I nor conclusion II follows.

Answer - Option C
Explanation :
Given Statement: V Y, Z = V, Y = Z
Y = Z=V
I. Y = V (false)
II. Y > V (false)
But this forms complementary pairs, hence either conclusion I or conclusion II follow
6. Direction: In these questions a relationship between different elements is shown in the statements. The statements are followed by conclusions. Find which of the conclusion follows the given statements: Statements: E > T, V = K, A < E, V < T Conclusions: I. T > V II. A > V
    A. only conclusion I is true B. either conclusion I or conclusion II is true. C. neither conclusion I nor conclusion II is true D. both conclusions I and II are true

Answer - Option A
Explanation :
Statement given
E> T, T>V, V = K
above can be written as
E>T>V=K & also A< E
from the above we concluded that
A is less than E so we can not say does A>V or not so only first statement is true.
7. Direction: In each of the following questions, assuming the given statements to be true, find which of the following options holds true: Statement: T = Q, G < K, O = M, G = J Conclusion: I. T = O II. M > G
    A. If only conclusion I is true. B. If only conclusion II is true. C. If either conclusion I or conclusion II is true. D. If neither conclusion I nor conclusion II is true

Answer - Option D
Explanation :
Ans. D
T = Q, G < K, O = M, G = J
Conclusions:
I. T = O ------ no relation between T & O, hence it is false.
II. M > G - no relation between M & G, hence it is false.
8. Direction: In each of the following questions, assuming the given statements to be true, find which of the following options holds true: Statement: F = H, A = C, G = H, H > C Conclusion: I. G > F II. G > C
    A. If only conclusion I is true. B. If only conclusion II is true C. If either conclusion I or conclusion II is true. D. If neither conclusion I nor conclusion II is true

Answer - Option B
Explanation :
Ans. B
F = H = G
Conclusions: I. G > F (false)
G = H > C
Conclusions: II. G > C (true)
9. Directions: In these questions, relationship between different elements is shown in the statements. These statements are followed by two conclusions. Statements: B = C = D = E A = F = P = D Conclusions: I. D = A II. B = F
    A. Only conclusion I follow. B. Only conclusion II follows. C. Either conclusion I or conclusion II follows D. Neither conclusion I nor conclusion II follows.

Answer - Option D
10. Which of the following expressions will be true if the expression M = N > O = P = Q > R is definitely true?
    A. M = P B. N = P C. R > M D. O > R

Answer - Option D
Explanation :
M =N > O = P =Q > R, from this we can say O > R is definitely true.
1. Direction: Study the following information carefully to answer the given questions. 'A%B' means 'A is neither smaller nor equal to B' 'A&B' means 'A is neither greater nor equal to B' 'A$B' means 'A is not smaller than B' 'A*B' means 'A is neither smaller nor greater than B' 'A @ B' means 'A is not greater than B' Now in each of the following questions, assuming the given statements to be true, find which of the two conclusions given below them is/are true. Statement: A*B, C @ D, E&A, A$F Conclusion: I. B$F II. E&B
    A. Only conclusion I is true. B. Only conclusion II is true. C. Either conclusion I or II is true D. Neither conclusion I nor conclusion II is true.

Answer - Option D
Explanation :
On decoding
g we get,F = A = B > E, C = D
B$F means B = F
We have F = A = B, so B = F.
E&B means E We have E
Thus, both conclusions I and II are true.
2. Directions: The symbols @, #, $, % and © are used with different meanings as follows: 'X @ Y' means 'X is smaller than Y'. 'X # Y' means 'X is not smaller than Y'. 'X $ Y' means 'X is neither smaller than nor greater than Y'. 'X % Y' means 'X is greater than Y'. 'X & Y' means 'X is not greater than Y'. In each of the following questions assuming the given statements to be true, find out which of the two conclusions I and II given below them is/are definitely true. Statements: G # S, S % I, I $ K, K & M Conclusions: I. G % K II. G # M
    A. Only conclusion I is true. B. Only conclusion II is true. C. Either conclusion I or II is true D. Neither conclusion I nor conclusion II is true.

Answer - Option A
Explanation :
G = S > I = K = M
G>K TRUE
G = M FALSE
3. Directions: In these questions, relationship between different elements is shown in the statements. The statements are followed by two conclusions. Statements: P = = R; S < R = T Conclusions: I. S < P II. T = P
    A. Only conclusion I is true. B. Only conclusion II is true. C. Either conclusion I or II is true D. Both conclusion I and II are true

Answer - Option D
4. Directions: In each of the following questions assuming the given statements to be true, find which of the two conclusions I and II given below them is/are definitely true and given your answer accordingly give answer Statements: U > V, W = X, X < U Conclusions: I. X < V II. X = V
    A. Only conclusion I is true. B. Only conclusion II is true. C. Either conclusion I or II is true D. Neither conclusion I nor conclusion II is true.

Answer - Option D
5. Directions: In these questions, relationships between different elements are shown in the statements. These statements are followed by two conclusions. Statement : A > L = T K Conclusions : I. H > L II. K > T
    A. Only conclusion I follows B. Only conclusion II follows. C. Either conclusion I or conclusion II follows. D. Neither conclusion I nor conclusion II follows

Answer - Option A
Explanation :
H>R>T and T=L
Hence H > L
T K
Hence K may or may not be greater than T
Hence conclusion 1 is correct
6. Direction: In these questions, relationship between different elements is shown in the statements. These statements are followed by two conclusions. Statements: W = R R. Conclusions: I. A = W II. M < A
    A. If only conclusion I follows B. If only conclusion II follows. C. If either conclusion I or II follows. D. If neither conclusion I nor II follows

Answer - Option D
Explanation :
If neither conclusion I nor II follows.
7. Directions: In these questions, relationship between different elements is shown in the statements. The statements are followed by two conclusions. Statements: E S< T< D Conclusions: I. G > S II. F= H
    A. If only Conclusion I is true. B. If only Conclusion II is true. C. If either Conclusion I or II is true. D. If both Conclusions I and II are true.

Answer - Option D
Explanation :
Givn Statement: E S<T<D Conclusions :
G>S (True)
F=(True)
both Conclusions I and II are true.
8. Direction: In the given question, assuming the given statements to be true, find which of the following options holds true. Statements: A = K, M > D < T, E = M, E = G Conclusions: I. D < E II. M < G
    A. Only conclusion I is true B. Only conclusion II is true C. Either conclusion I or conclusion II is true D. Neither conclusion I nor conclusion II is true

Answer - Option A
Explanation :
A=K, M>D<T, E =M, E=G
For conclusion I - E = M > D
I. D < E ----- True
For conclusion II - G = E = M
II. M
Hence, only conclusion I is true.
9. Directions: In each of the following questions, assuming the given statements to be true, find which of the following options holds true: Statements: B<Q=F< H, F=N Conclusions: I. N=Q II. Q>H
    A. if only conclusion I is true. B. if only conclusion II is true. C. if either conclusion I or conclusion II is true. D. if neither conclusion I nor conclusion II is true

Answer - Option A
Explanation :
B<Q=F< H, F=N
B<Q=F = N
I. N=Q (true)
B<Q=F< H
II. Q>H (false)
If only conclusion I is true.
10. Directions: In each of the following questions, assuming the given statements to be true, find which of the following options holds true: Statements: T=Q, G<Q, O = T, G=J Conclusions: I. T = J II. Q>J
    A. if only conclusion I is true. B. if only conclusion II is true C. if either conclusion I or conclusion II is true. D. if neither conclusion I nor conclusion II is true

Answer - Option B
Explanation :
T=Q, G<Q, O = T, G=J
I. T = J (false) there is no relation between T & J
J = GII. Q>J (true)
Hence only conclusion II is true.
1. Direction: Study the following information carefully and answer the given questions. Eight friends Pihu, Quba, Riah, Sunny, Tina, Uber, Vicky and Wadra are sitting in a straight line facing east. Tina sits to the immediate right of Quba. Three persons are sitting between Pihu and Wadra. Quba sits third to the right of Vicky who is second to the right of Sunny. Wadra who is sitting at one of the extreme ends and is sitting second to the left of Uber. Which of the following is sitting at extreme end of the row?
    A. Quba and wadra B. Sunny and tina C. Tina and wadra D. Riah and tina

Answer - Option C
Tina and wadra

2. Direction: Study the following information carefully and answer the questions given below: P, Q, R, T, V, W and Z are seven football players, each playing for a team, viz Green, Red and Blue, with at least two of them in each of these teams. Each of them likes a different fruit, viz Apple, Guava, Banana, Orange, Mango, Papaya and Watermelon, not necessarily in the same order.
Q plays with V for team Blue and he likes Banana. None of those who play for either team Red or team Green likes either Guava or Banana. T plays with only the one who likes Watermelon. W likes Papaya and he plays for team Red. The one who likes Orange does not play for team Red and Blue. Z likes Watermelon and he plays for team Green, P likes Apple and he plays for team Red. R does not like Guava and Plays for team Red. Which of the following players, plays for team Red?
    A. WPR B. TZR C. WQR D. WVR

Answer - Option A
Explanation :
From the given conditions, we can conclude:

Hence PRW plays for team Red. Hence options A is correct.
3. Direction: Study the following information carefully and answer the questions given below.
P, Q, R, S, T, V and W study in Std IV, V and VI with at least two in any of these standards. Each one of them has a favourite (likes) colour, viz black, red, yellow, green, white, blue and pink, not necessarily in the same order.
Q likes yellow and does not study in Std. VI. The one who likes black studies in the same Std as T. R likes blue and studies in the same Standard as W. S studies in Std V only with the one who likes pink. W does not study either in Std V or VI. V does not like black. W does not like either green or white. S does not like green. T does not like pink. Who likes white?
    A. P B. W C. S D. V

Answer - Option C
Explanation :

4. Directions: Study the following information carefully and answer the questions given below: P, Q, R, S, T, V and W are travelling in three buses A, B and C with at least two of them at any of these buses. Each of them has a favourite (likes) cuisine viz Punjabi, Rajasthani. Bangali, Maharashtrian, Gujarati, Kahmiri and Udipi not necessarily in the same order. Q is travelling in bus B with T. T's favourite cuisine is Udipi. Those who travel in bus A do not like Punjabi and Maharashtrian cuisines. The one who likes Rajasthani cuisine travels only with W in bus C. The one whoes favorite cuisine is Gujarati does not travel in the same bus with either T or W. P does not travel in bus B. P likes Kashmiri cuisine. S and V are travelling in the same bus. V does not like Bengali cuisine. The one whose favourite cuisine is Maharashtrian does not travel in Bus B. Which of the following combinations is correct?
    A. A-V-Gujarati B. B-S-Bengal C. C-W-Punjabi D. B-Q-Gujarati

Answer - Option A
Explanation :

It is clear from above table, the combination ‘A-V-Gujarati’ is correct.
Hence, option A is correct.
5. Direction: Study the following information carefully and answer the questions. A college held Inter-college competitions for Dance, Singing, Extempore, Cricket and Hockey during the Youth Fest 2016 week from Monday to Saturday, each game on one day, one day being a rest day. Extempore competition was held not on the first or on the last day but was held earlier than Singing competition. Cricket competition was held on the immediate next day of the Singing competition day. Dance competition was held on the immediate previous day of the rest day. Cricket competition day and Hockey competition day had a two day gap between them. Hockey competition was held on the immediate following day of the rest day. Which of the following was a rest day?
    A. Wednesday B. Tuesday C. Friday D. Thursday

Answer - Option B
6. Direction: Study the following information carefully and answer the given questions. Seven games namely TT, Football, Boxing, Cycling, Shooting, Swimming, Wrestling are to be played in a competition. Only one game will be played on each of the seven days of the week, starting from Monday and ending on Sunday. Boxing was held on Wednesday. Wrestling was scheduled the day next to Football. Football was not played on Monday or Friday. Two games were played between the days Football and Shooting were played. The shooting was not scheduled on Sunday. Cycling was scheduled a day before Swimming. On which of the following days does TT was scheduled?
    A. Friday B. Sunday C. Tuesday D. Thursday

Answer - Option C
7. If it is possible to make only one meaningful English word with the second, fourth, fifth and seventh letters of the word COURTESY, using each letter only once in the word, which of the following will be the third letter of that word? If no such word can be formed, give 'N' as your answer, and if more than one such word can be formed, give 'M' as your answer.
    A. R B. O C. S D. M

Answer - Option D
Explanation :
From COURTESY, second, fourth, fifth and seventh letters are O, R, T and S respectively. Out of these 4 letters, SORT and ROTS words can be formed.
8. Among J, K, L, M and N each having different height, M is shorter than only J. K is not taller than N and N is shorter than L. Who among them is the shortest?
    A. J B. N C. K D. L

Answer - Option C
Explanation :
J >M, L > N > K
J > M > L > N > K
9. Direction: Study the following information carefully and answer the questions given below: Among five friends P, Q, R, S and T each having different height, T is the second tallest. R and T are taller than Q. P is taller than only S but shorter than Q.
    A. R or T B. T C. R D. Q

Answer - Option C
Explanation :
R > T > Q > P > S
R is the tallest in the group.
10. Directions: Study the following information and answer the questions. Five friends A,B,C,D and E like five different types of TVs, Samsung, Sony,LG,Sansui and Panasonic, five different types of colors Black, Blue, Red, Green and Pink and also five different types of games Cricket, Basketball, Tennis, Football, Hockey (not necessarily in the same order). * D likes Black but does not like Tennis and Sansui. * A likes Sony and Basketball but he does not like Pink. * B likes Pink and LG. * E likes Red and Football. * The person who likes Black also likes Panasonic. * Cricket is liked by the person who likes Green and Sansui.
    A. D B. C C. A D. E

Answer - Option A
Explanation :
1. Direction: Study the information given below and answer the questions based on it. Eight boxes P, Q, R, S, T, U, V and W are kept one above another. Top position is 1st and bottom position is last. Three boxes are between S and Q. Box V is immediately above box S. 3 boxes are kept between R and P. Box R is above P. There are the same number of boxes between R and W as between W and S. One box is kept between V and U. Box U is below box V. How many boxes are between P and Q?
    A. 5 B. 1 C. 2 D. 3

Answer - Option B
2. Directions: Read the following information carefully and answer the following questions. Ten Boxes A,B,C,D,E,F,G,H,I and J are place one above the other in any particular order. Box no. 1 is at the bottom and Box no. 10 is at the top. There are two boxes between F and I. Box J is placed at the Bottom. There are three boxes between Box A and Box E. Box I is placed on even no. position. Box C is at 8th no. position. Box D is kept immediately below Box C and immediately above Box E. Box H is placed between the Box G and I. Which Box is placed at the top?
    A. Box B B. Box A C. Box G D. Box F

Answer - Option A
3. Direction: Study the following information carefully and answer the questions given below:
Five boxes P, Q, R, S and T are kept one above another. Their colours are i.e. Red, Pink, Yellow, Black and Blue, not necessarily in the same order. One box is between Q and Black box. Two boxes are between R and Black box. One box is between Red and Pink box and neither of them is at the bottom position. Box S is the yellow coloured box. The number of boxes between R and T is same as the number of boxes between Q and P. Box P is not at the bottom or top. Red box is above the Blue box. How many boxes are between R and S?
    A. 0 B. 1 C. 2 D. 3

Answer - Option D
4. Direction: Study the following arrangement carefully and answer the question given below:-
Eight friends P,Q, R, W, X, Y, H and J live on eight different floors of a building but not necessarily in the same order. The lowermost floor of the building is numbered 1 and the topmost floor of the building is numbered 8. All persons like different types of vegetables i.e. Chilli, Cauliflower, Capsicum, Carrot, Cabbage, Corn, Cucumber and Celery but not necessarily in the same order. The date of birth of each of the persons falls in different months i.e. January, February, May, June, September, October, November and December but not necessarily in the same order.
The one whose DOB is in December does not live on an odd-numbered floor and does not like Cauliflower & Cucumber. J lives just below the one whose DOB is in October. The one who lives on Third floor likes Chilli. The one whose DOB is in Febuary lives on an even-numbered floor but not on the topmost floor and likes either Celery or Corn Only one person lives between Y and the one whose DOB is in November. The one who likes Cabbage lives immediate above the floor in which J lives. X likes Cucumber . The one whose DOB is in June lives on an even numbered floor and lives just above the person whose DOB is in October. R’s DOB is not in November or October. The persons (who likes Cauliflower, Carrot, Celery and Capsicum) lives on even numbered floor. The one who likes Capsicum lives below the one who likes Carrot. Only two persons live between the one whose DOB is in December and the one whose DOB is in January. W’s DOB is not in September. Only two persons live between Y and the one whose DOB is in Febuary. Neither X nor R lives on the first floor. Only one person lives between R and the one whose DOB is in May. P lives just above Y. Only two persons live between X and P. The one whose DOB is in November does not live on floor number one. P does not like Capsicum or Carrot. Q lives on an even-numbered floor and just above R. How many persons live between the person whose date of birth month is in January and the one who likes Cauliflower?
    A. Six B. Two C. Four D. Five

Answer - Option C
5. Direction: Study the information given below and answer the questions based on it. Eight persons A, B, C, D, P, Q, R and S live on different floors of a building but not necessarily in the same order. The lowermost floor of the building is numbered 1 and the topmost floor of the building is numbered 8. These persons are going for tour to different places viz. Mumbai, Ajmer, Mirzapur, Jaipur, Jodhpur, Kanpur, Shimla and Patna. All persons like different types of colors i.e. Grey, Black, Green, Red, Violet, Blue, Yellow and Orange but not necessarily in the same order. The one who is going to Kanpur does not live on an odd numbered floor and does not like Black color and Yellow color. S lives just below the one who is going to Patna. The one who lives on third floor likes Grey color. The one who going to Ajmer lives on an even numbered floor but not on the topmost floor and likes either orange color or blue color. Only one person lives between Q and the one who is going to Jodhpur. The one who likes Violet color lives immediately above S. P likes Yellow color. The one who is going to Jaipur live on an even numbered floor and lives just above the person who is going to Patna. C is not going to Jodhpur or Patna. The persons who likes Black, Red, Orange and Green colors live on even numbered floor. The one who likes Green color lives below the one who likes Red color. Only two persons live between the one who is going to Kanpur and the one who is going to Mumbai. D is not going to Shimla. Only two persons live between Q and the one who is going to Ajmer. Neither P nor C lives on the first floor. Only one person lives between C and the one who is going to Mirzapur. A lives just above Q. Only two persons live between P and A. The one who is going to Jodhpur does not live on the floor number one. A does not like Green or Red color. B lives on an even numbered floor and just above C. Which of the following is correctly matched?
    A. A – Patna - Red B. P – Jodhpur- Orange C. C – Mumbai - Grey D. B – Shimla - Blue

Answer - Option C
6. Direction: Study the following information carefully and answer the questions.
Eight persons R, S, T, U, V, W, X and Y are going to different cities, i.e. Jaipur, Mysore, Mumbai, Kolkata, Delhi, Pune, Shimla and Amritsar but not necessarily in the same order. These persons like different foods, viz. Dosa, Burger, Samosa, Dhokla, Mathri, Khandvi, Khamman and Namkeen but not necessarily in the same order.
X likes neither Dhokla nor Khandvi. Neither W nor X likes Dosa and Samosa. V is going to the Amritsar. T is going to the Mysore and likes Burger. The person which likes Khamman is going to Amritsar. R and S like either Samosa or Namkeen. U does not like Dosa and W does not like Khandvi. The person which is going to the Delhi likes Dhokla and the person which is going to Shimla, likes Khandvi. R and X neither go to Kolkata nor to Jaipur. The person which has to go Pune, likes Namkeen. Y and X go either Mumbai or Jaipur. Which of the following is going to Delhi?
    A. U B. V C. W D. Y

Answer - Option C
7. Direction: Study the information given below and answer the questions based on it.
There are eight members J, K, L, M, E, F, G, H in a family. All are relatives of each other. They are going to the different places, i.e. Taj Mahal, Lal Quila, Hawa Mahal, Qutub Minar, Jantar Mantar, Chaar Minar, India Gate and Sanchi stupa but not necessarily in the same order.
Among them only two are the married couple, there are only 3 male members in the family. Brother of M is going to Lal Quila. One of the couples is going to India Gate and Hawa Mahal. Spouse of F is going to Chaar Minar. J is married to M’s grandmother. F is the only son of J. M is the sister of G. G is the daughter of L. L is the daughter-in-law of K. The person who is going to Qutub Minar is a male. One of the sisters of H is going to Taj Mahal. No male is going to either India Gate or Jantar Mantar. Sister-in-law of F is going to Sanchi Stupa. E is the aunt of M. L is married to F. J has only one grandson. G does not go to Jantar Mantar. Which of the following is true regarding blood relation?
    A. M is the daughter of J. B. H is the brother of G. C. No one is true D. E is the wife of F.

Answer - Option B
8. Direction: Read the following information carefully and answer the questions given below.
Eight persons, A, B, C, D, E, F, G, and H are living on different floors and lived one above the other. Each person is to visit different cities, viz Jaipur, Bhopal, Delhi, Indore, Ranchi, Patna, Dehradun, and Kota but not necessarily in the same order. Each person like different sweets, viz Laddu, Peda, Petha, Barfi, Ghevar, Pinni, Bal Mithai and Balushahi.
Only two persons are between the one who visits the capital of Rajasthan and C. The person who visits Jaipur is above the person C. The one who likes Petha is immediately above the person C. The famous sweet of Uttarakhand, Bal mithai is liked by the person who lived on Floor 2 and not liked by F. Only three persons are lived between the person who visits Jaipur and the person who like Bal Mithai. There are at least two persons below the person who visit Indore is as many as that of those above the one who likes Laddu. The person who visits Indore does not live on Floor No. 2. Only two persons are placed between the person who visit Delhi and the one who likes Laddu. D who is visiting the capital of India and is lived somewhere above the one who visit Jaipur. H visits Dehradun who likes Pinni. The person who visits Ranchi is placed immediately above the person who visits Dehradun. A neither like Barfi nor Balmithai. The person who likes Balushahi is lived somewhere below the person who like Laddu, but not on the ground floor. E visits to the capital of Bihar and the person who visit Indore like to eat Petha. There are only two persons lived between D and E respectively. The person who visits Patna is somewhere below the one who visit Jaipur. The famous sweet of Rajasthan is liked by the person, D who lived on the topmost floor and the famous sweet of Punjab is liked by the one who lived on the ground floor. Barfi is immediately below Ghewar and visit the city Bhopal. There are only three person lived below G. How many persons between the person who likes Barfi and the one who visit Ranchi?
    A. Two persons B. Four persons C. Six persons D. Other than the given options

Answer - Option B
9. Direction: Read the following information carefully and answer the questions that follow.
P, Q, R, S, T, U, V and W are eight different boxes, they are arranged in such a manner that Box 1 is at the bottom, the box 2 is above it and so on such that the topmost box is box number 8. R is box number 3. There are only two boxes between the box R and the box V. Box W is placed immediately above the Box Q. There is only one box between the box T and the box U. Box T is placed above box U. There is only one box between the Box R and Box S. Box S is somewhere below Box T. Which among the following boxes is the fifth numbered box?
    A. S B. Q C. W D. P

Answer - Option D
10. Directions: Study the following information carefully and answer the questions given below.
Eight friends - M, N, O, P, Q, R, S and T are members of the same society - Malik Enclave. They all belong to different age group - 16, 17, 18, 20, 22, 24, 27 and 30. T is 18 years old. P is 4 years elder than the age of O. S was born between Q and T. Age of N is 10 more than the half the age of R. Age of R is 10 years more than the age of S. Age of P is thrice the half of the age of Q who is 16 years old. Who is the youngest member of the society among these eight friends?
    A. O B. Q C. R D. S

Answer Option B
1. Directions: Study the following information carefully and answer the given questions. A, B, C, D, E, F, G and H are sitting around a circular table facing the centre. (a) B sits third to the right of F. (b) A sits second to the right of D. D is not an immediate neighbour of B and F. (c) C and E are immediate neighbours of each other. (d) H is not an immediate neighbour of A. (e) No one sits between C and F. Four of the following five are similar in a certain way based on their positions in the seating arrangement and so form a group. Which of the following does not belong to that group?
    A. DA B. BC C. HG D. AC

Answer - Option B
Explanation :

The solution for the above puzzle is:
Except in BC in all others the first person is sitting second to the left of the second person. B is second to the right of the C.
Hence, option B is correct.
2. Directions: Study the following information carefully and answer the questions given below: A, B, C, D, E, F, G, H and K are sitting around a circlefacing the centre. B is fourth to the right ofH and second to the left of A. F is fourth to the left of A and third to the right of C. G is third to the right ofK. E is second to the left of D. What is second to the left of K?
    A. A B. D C. B D. H

Answer - Option B
Explanation :

3. Study the following information carefully and answer the questions given below: Monica, Neelam, Peter, Rahul, Tarun, Waseem, Faiz and Harsh are sitting around a circle facing the centre. Peter is third to the left of Monica and second to the right of Tarun. Neelam is second to the right of Peter. Rahul is second to the right of Waseem, who is second to the right of Monica. Faiz is not an immediate neighbour of Peter. Who is to the immediate right of Peter?
    A. Harsh B. Faiz C. Rahul D. Sunny

Answer - Option A
Explanation :

4. Direction: Study the following information carefully and answer the questions given below: Ten people are sitting in two parallel rows having five people each in such a way that there is equal distance between adjacent persons. In row-I, F, G, H, I and J are seated and all of them are facing north. In row-II, R, S, T, U and V are seated and all of them are facing south but not necessarily in the same order. Therefore, in the given seating arrangement each member seated in a row faces another member of the other row.
S sits third to the left of R. U sits second to the right of S. The person facing U sits immediate left of H. Only one person sits between H and G. F is not an immediate neighbour of H. Only two people sit between F and I. Neither H nor F faces V. Which of the following statements is true regarding G?
    A. G faces one of the immediate neighbours of T B. H is an immediate neighbour of G. C. I sits on the immediate left of G. D. Only one person sits between J and G

Answer - Option C
Explanation :
Following are the steps;
1.S sits third to the left of R.
2.U sits second to the right of S.
3.The person facing U sits immediate left of H.
4.Only one person sits between H and G.
5.F is not an immediate neighbour of H.
6.Only two people sit between F and I.
7.Neither H nor F faces V.

5. Direction: Study the following information carefully and answer the questions.
Read the Information given carefully and answer the questions that follow.
Eight persons A, B, C, D, E, F, G and H are sitting around a square table such that there are two on each side and they are all facing the centre the table.
Only D sits between A and G. E sits two places to the left of A. F and H are sitting along one side of the square table. F sits opposite A. B sits two places to the left of F. Who sits opposite D?
    A. G B. B C. C D. H

Answer - Option D
Explanation :

6. A, B, D, M, P, Q, T, V and W are sitting around a circle facing the centre. D is third to the left of A, who is second to the left of W. B is second to the right of W and fourth to the left of M. V is fourth to the right of Q, who is not an immediate neighbor of A. T is fourth to the left of P. Who is on the immediate left of D?
    A. P B. V C. D D. M

Answer - Option A
Explanation :

7. Direction: Study the information carefully and answer the given questions. A, C, D, I, L, P and M are sitting in a straight line facing North. (a) P sits fourth to the right of A and C sits second to the left of P. D sits in the middle and is second to the right of M. (b) I sits at the farthest possible distance from P (five persons sits between I and P). If all the seven persons are made to sit in alphabetical order from left to right, the positions of how many will remain unchanged as compared to the original seating positions?
    A. 0 B. 1 C. 2 D. 3

Answer - Option B
Explanation :

8. Directions: Study the following information to answer the given questions. A, B, C, X, Y, Z are seated in a straight line facing North. C is third to the right of Z and B sits second to the right of C, X sits to the immediate right of A. What is X’s position with respect to Z?
    A. Immediate right of Z B. Second to the left C. Third to the right D. Second to the right

Answer - Option D
Explanation :

9. Directions: Read the following information carefully and answer the questions given below it— Eight players P, Q, R, S, T, U, V and W, out of whom one is a Archer, swimmer, boxer, wrestler, hockey player, football player, chess player and a tennis player (but not necessarily in the same order) are sitting around a circular table facing the centre.
S who is a football player sits third to right of W. The swimmer and the tennis player are sitting next to each other. Neither the swimmer nor the tennis player is sitting next to either W or S. The Archer is neither W nor V. Chess player and the Archer are immediate neighbours of each other. The hockey player T sits second to the left of swimmer. The chess player is neither U nor sitting next to U. Only R sits between the swimmer and the wrestler. Archer sits third to the left of P. Who sits third to the right of the swimmer?
    A. The football player B. V C. The Chess player D. Both B and C

Answer - Option B
Explanation :

V - the chess player sits third to the right of the swimmer.
10. Direction: Study the information given below and answer the questions based on it. Eight family members- M, N, O, P, Q, R, S and T were sitting around a circular table facing the centre. O, one of the sons of S, sits exactly opposite S and is an immediate neighbor of his mother. O's sister R is exactly opposite her husband P and is an immediate neighbour of T. O's niece, N, sits third to the left of her grandfather. Q, who has no child, is a male member while O and N are the only unmarried members of the family. No couple sits together. O's sister-in-law, T, sits second to the right of her husband. How many female members in the family?
    A. 3 B. 4 C. 5 D. 2

Answer - Option B
1. Study the following information carefully and answer the given questions.
P, Q, R, S, T and V are sitting in a straight line facing North, not necessarily in the same order.
(a) Q sits third to the right of T. (b) T sits second to the right of P. (c) V is not the immediate neighbor of T. (d) R sits second to the left of S. What is the position of S with respect to P?
    A. Third to the right B. Second to the left C. Immediately to the right D. Second to the right

Answer - Option A
Explanation :

Thus S is third to the right of P.
2. Direction: Read the given information and answer the following questions. There are 8 family members with 4 couples namely- A, B, C, D, E, F, G and H are sitting around a circular table, two of them are facing away from the center. Only two people sit between B and E. B faces the centre. F sits second to the right of B. E is the wife of A. No female is an immediate neighbor of E. C is not an immediate neighbor of B. C is the daughter of E. Both the immediate neighbors of C face the centre. Only three people sit between A and C’s brother. F is not the brother or husband of C. A, C and C’s brother none of them is an immediate neighbor of F. D sits three places away from G’s husband. H, the wife of B, sits to the immediate left of D. Both G and A face a direction opposite to that of C. C’s husband sits second to the left of G. B’s father sits to the immediate right of E. B sits second to the right of A’s father. Both the immediate neighbors of F are females. G is grandmother of B Who amongst the following sits exactly between G and E when counted from the left of G?
    A. B B. F C. A D. H

Answer - Option D
3. Direction: Study the following information carefully to answer the questions. A, B, C, D, E, F, G and H are sitting around a circle facing the centre. F is third to the right of C and second to the left of H. D is not an immediate neighbour of C or H. E is to the immediate right of A, who is second to the right of G. Who is second to the left of C?
    A. A B. B C. E D. D

Answer - Option A
4. Direction: Study the information given below and answer the questions based on it. There is a group of eight persons A, B, C, D, E, F, G and H, including two families. All of them are sitting around a circular table, facing the center. Three members of this group belong to one family and the remaining five belong to another family. There are four females in the group. G who is a female sits second to the left of E. H is the wife of D and is third to the left of B. D sits on the immediate right of his wife. F is the sister of H and is not exactly between G and E. B sits on the immediate left of G, who is the wife of E. D sits opposite her sister and her sister sits second to the left of A, who is the son of H. B is the son of E. Who among the following sits second to the right of A?
    A. D B. F C. C D. G

Answer - Option A
5. Study the following information and answer the questions given below it
Seven people – A, B, C, D, E, F and G are sitting in a circle. Five of them are facing the centre while two of them facing opposite to the centre. C sits third to the left of D and both are facing the centre. E is neither immediate neighbour of D nor of C. The one sitting exactly between D and F is facing opposite to centre. G is facing the centre. One of B’s neighbour is facing opposite to the centre. B sits second to the left of one of the person who is facing outside. D and F are not neighbours. Which of the following pairs represents persons facing opposite to the centre?
    A. A and F B. E and F C. A and E D. Cannot be determined

Answer - Option C
Explanation :

A and E pairs represents persons facing opposite to the centre
6. Direction: Study the following information carefully to answer the questions that follows.
Eight persons namely A, B, C, D, E, F, G and H are sitting in a straight line from East to West. Some are facing North direction and some are facing South direction. H sits to the third left of C and faces South. F is sitting at one of the extreme ends and is facing North. B and G are facing in same direction and person sitting between them is facing in opposite direction. A is sitting third from left end and is facing South direction. A is sitting adjacent to E and both are facing in opposite direction. D is sitting to the immediate right of B and both are facing in same direction. C is facing in North direction. A and D are not neighbours. Neighbours of H are facing in same direction which is opposite to the direction of H is facing. E sits to the right of A. G is sitting between A & H. Who is sitting between A and H?
    A. A B. B C. G D. R

Answer - Option C
Explanation :
1.F is sitting at one of the extreme ends and is facing North.
2.A is sitting third from left end and is facing South direction.
3. E sits to the right of A.
4. H sits to the third left of C and faces South.
5. C is facing in North direction.
6. B and G are facing in same direction and person sitting between them is facing in opposite direction.
7. D is sitting to the immediate right of B and both are facing in same direction.
8. A is sitting adjacent to E and both are facing in opposite direction.
9. Neighbours of H are facing in same direction which is opposite to the direction of H is facing.
10.G is sitting between A & H

7. Direction: Study the information given below and answer the questions based on it.
Six persons A, B, C, D, E and F are sitting in a straight line and facing north direction, distance between them is multiple of 3. A is 36m to the west of D. C is 45m east of F. A is sitting at extreme left end. D is 27m to the east of B. F is in east direction of D. Distance between C and E is 9m. Distance between D and F is double the distance of A and B (like-If distance between A and B is 2m then distance between D and F will be 4m). Distance between F and E is more than 50m. What is distance between B and C?
    A. 85 m B. 90 m C. 100 m D. 45 m

Answer - Option B
8. Direction: Study the following information and answer the questions:
Ten friends M, N, O, P, Q, R, S, T, U and V sit in a straight line. Four of them face South while the remaining six face North. Any of the two friends who face South is neither immediate neighbour of each other nor sits at any extreme ends of the line. M sits fourth to the right of S and fourth to the right of N. S sits on the immediate left of O. T sits adjacent to V and U and both of them face South. P sits second to the left of R. U and P are not the immediate neighbours of N. Q and R face the same direction. How many persons sit between V and R?
    A. Four B. Three C. Five D. Two

Answer - Option C
Explanation :
1. Any of the two friends who face South is neither immediate neighbour of each other nor sits at any extreme ends of the line.
2. M sits fourth to the right of S and fourth to the right of N.
3. S sits on the immediate left of O. T sits between V and U and both of them face South.
4. U and P are not the immediate neighbours of N.
5. P sits second to the left of R.
6. Q and R face the same direction.

9. Directions: Study the following information carefully to answer these questions.
Ashwini, Priya, Sudha, Rani, Meeta, Geeta and Mukta are sitting around a circle facing the centre. Ashwini is third to the left of Mukta and to the immediate right of Rani. Priya is second to the left of Geeta who is not an immediate neighbour of Meeta. Who is to the immediate right of Priya?
    A. Meeta B. Sudha C. Mukta D. Priya

Answer - Option C
Explanation :

Mukta is to the right of Priya
Hence option C is correct
10. Direction: Study the information given below and answer the questions based on it.
Five girls N, K, J, M and P are sitting in a circle and facing towards the centre. They like different colors Green, Blue, Yellow, Black and Red and study in different classes 12th ,8th ,7th, 6th and 3rd. One who studies in odd number class does not like Red and the oldest sit second to the left of N. J is neither oldest nor youngest and does not like Yellow. M sits between P and J.K is not studying in even number class. One who likes Black is in 8th class. Three people sit between the one who likes Blue and N. The one who is youngest like Green. The one who likes Blue sits second to the right of Yellow. N studying in 8th class. The girl who likes Red sits immediate left of Yellow. Who is in 7th class?
    A. One who likes Red B. One who is immediate right of N C. One who does not like Yellow D. One who sits second to the left of J

Answer - Option D
1. Read the following information carefully and answer the given questions.
Eight persons A, B, C, D, E, F, G & H are sitting around a circle facing towards the centre. There are three generations in the family and three married couples. (I) B and G have two children. E, the husband of A is sitting to her immediate right and their only child, who is sitting exactly opposite to A. (II) A’s brother is sitting 2nd to her left, who is sitting immediate right to his father. (III) D’s wife H is sitting to her immediate right. F & B are sitting together. B is an immediate neighbor of E. D is father of F. Who is the child of A?
    A. B B. D C. G D. C

Answer - Option D
Explanation :

2. Directions: Study the following information carefully and answer the given questions.
Google, Apple, HP, Samsung, TCS, Amazon, Sony and Facebook are eight companies and the heads of the eight companies are A, B, C, D, E, F, G and H, who are sitting around a circular table facing the centre, but not necessarily in the same order.
F sits second to the right of the person who is the head of Samsung. The head of Sony is an immediate neighbour of the head of Samsung. Two persons sit between the head of Sony and B. C and E are immediate neighbours. Neither C nor E are immediate neighbours of either B or the head of Samsung. The head of the HP sits second to the right of D, who is the head neither of Samsung nor of Sony. G and the head of Google are immediate neighbours. B is not the head of Google. Only one person sits between C and the head of Apple. H sits third to the left of head of the Facebook. The head of Amazon sits second to the left of the head of TCS. The head of the HP sits on immediate left of the head of Samsung and second to the right of the head of Apple. Who among the following is the head of Apple?
    A. C B. D C. B D. E

Answer - Option B
Explanation :

3. Direction: Study the following information carefully and answer the questions given below:
Eight friends A, B, C, D, E, F, G and H are going to visit eight different countries viz—China, India, France, Russia, Canada, Australia, US and Japan. They are sitting around a circle facing outward the centre but not necessarily in the same order. F sits second to the left of the one who is going to visit Russia. The one who is going to visit US is an immediate neighbour of the one who is going to visit Russia. Two persons sit between the one who is going to visit US and B. C and E are immediate neighbours of each other. Neither C nor E is an immediate neighbour of either B or the one who is going to visit Russia. The one who is going to visit France sits second to the left of D, who is going to visit India. G and the one who is going to visit China are immediate neighbours of each other. B is not going to visit China. Only one person sits between C and D. H sits third to the right of the one who is going to visit Japan. The one who is going to visit Australia sits second to the right of the one who is going to visit Canada. Four of the following five are alike in a certain way based on the given seating arrangements and thus form a group. Which is the one that does not belong to that group?
    A. H - China B. A - Russia C. D - France D. E - Canada

Answer - Option B
Explanation :

4. Directions: Study the following information carefully and answer the questions given below:
M, N, O, P, Q, R, S and T are eight friends sitting around a circular table. Six of them are facing the centre. All of them have a different hobby viz— Dancing, Singing, Cooking, Story Writing, Watching Movie, Jogging, playing cricket and Reading Novel, but not necessarily in the same order. O is an immediate neighbour of P. The person who is opposite N is facing outside and N is the immediate neighbour of the person his hobby is Story Writing. T sits third to the right of Q and his hobby is Playing cricket. M and T are not the neighbours of the person whom hobby is Jogging. R’s hobby is not cooking or Story Writing. The person whom hobby is Reading Novel is sitting opposite Q. The one whom hobby is Story Writing, sits immediate right of N. P and S are immediate neighbours of N, whom hobby is Watching Movie. S sits third to the right of M and his hobby is Dancing. P sits third to the left of M. Whom hobby is jogging?
    A. M B. Q C. O D. N

Answer - Option B
Explanation :

5. Direction: Study the following information and answer the given questions. A, B, C, D, E, F and G are sitting around a circular table facing the centre. Each of them likes a different flower i.e. – Rose, Marigold, Hibiscus, Lotus, Lily, Jasmine & Daffodil. Find out the correct seating arrangement along with the flower each one carries with the help of below given queries. 1) G is second to the left of C, who is on the immediate left of F; and C likes Jasmine 2) Person liking rose & lotus do not sit side by side 3) A is third to the left of E, who likes rose 4) Persons who love 2 flowers which have same initials sit side by side 5) B is between D & E and loves marigold 6) E is sandwiched between Daffodil and marigold. 7) D loves a red coloured flower but not rose Who sits between G and B?
    A. A B. C C. E D. D

Answer - Option D
6. Direction: Study the information given below and answer the questions based on it. Six captains G, H, I, J, K and L of 6 different teams U, V, W, X, Y and Z are sitting around a circular table facing to the centre. The difference between each of one is same. They all have of different ages 31, 32, 33, 34, 35, and 36. 1. I is seated second to the right of the captain of team X. The one whose age is not 36 is I. 2. The captain whose age is 33 is seated second to the left of the captain of team Z, which is not the team of G or I. 3. H is seated opposite to the captain of team Y, whose age is 32. 4. There is only one captain between the captain of team Z and Y but he is not U. Y sits to the right of Z. 5. The captain whose age is 34 is seated second to the right of G. 6. The captain whose age is 35 is not seated near to one whose age is 32. 7. L is seated immediate left of the captain of team W but not near to G. 8. The captain of team U is not seated near to the captain of team V but immediate right of J. Who is sitting opposite to the captain of team V?
    A. I B. H C. J D. Captain of team U

Answer - Option D
7. Direction: Study the information given below and answer the questions based on it.
12 people A, B, C, D, E, F, G, H, P, Q, R, S sitting around a square table, 6 sitting at the inner edge and 6 at the outer edge. Out of 6 at each edge, 2 should be at each longer side and 1 at each shorter side. They should be facing each other. A is the largest member in the family. A is sitting second to the right of B at inner edge. Only two persons are shorter than P. P is sitting at one of the shorter sides of outer edge. R is bigger than P and sit second to the left of P. Only two members are bigger than S. He is sitting just opposite side of P. E’s age is twice the age of R. He is sitting one of the shorter sides of inner edge. R’s age is 24 years. G and H both are the shortest than others but G is bigger than H. G is sitting third to the right of P and H is sitting second to the left of E. C is the second largest member of the family. D is the shorter than S and bigger than E. Q and F are the immediate neighbours and both are sitting at the longer side of outer edge. Q is the immediate left G. F is shorter than Q. Q and F both are shorter than R. C is sitting immediate right of A and D is sitting second to the left of F. B’s age is 35 years. Who is the Fourth largest member in the family?
    A. A B. B C. C D. D

Answer - Option D
8. Direction: Study the information given below and answer the questions based on it.
Eight friends - A, B, C, D, E, F, G, and H - are sitting around a circular table, but not necessarily in the same order, four of them are facing inside and the other four are facing outside. All eight friends like different colors – Green, Red, Yellow, Orange, Violet, Blue, Black, and White but not necessarily in the same order.
A likes Violet. F is sitting to the third right of the person who likes Black. D likes Yellow and faces the person who likes Blue. G sits third to the right of B, who likes Green. E is sitting between the person who likes Yellow and the one who likes White respectively. H is the immediate neighbor of the persons who like Violet and Blue. C likes Red. The person who likes Black is facing the same direction as the person who likes Orange. E is on the immediate left of D. No two persons facing same direction sit adjacent to each other. Who among the following likes Blue colour?
    A. A B. B C. F D. G

Answer - Option C
9. Direction: Study the information given below and answer the questions based on it.
Eight members of a family Roshni, Rohini, Rupika, Rupali, Rohan, Rohit, Roshan and Ravish are sitting around a circular table, some of them facing towards the centre while others facing opposite to the centre. There are three married couples in the family. No two members with the same gender are sitting together.
Rupika, who is daughter-in-law of Rohan sits opposite to Rohan’s wife. Roshni is not the mother of Roshan who is facing opposite direction to Ravish. Ravish is the father of Roshan. Rohini is the only child of Rohit and sits third to the right of her father, who is facing opposite to the centre. Roshni and her brother are immediate neighbors of each other and both of them are facing opposite directions. Rohan is facing same direction as Ravish. Roshni sits third to the left of Ravish’s father. Roshan’s aunt sits second to the right of Roshan’s mother and both of them face towards the centre. Roshan sits immediate right of his grandmother and third to the left of his mother. Rohini, who is the granddaughter of Rupali, sits second to her right and both of them face opposite directions. Four of the following are alike in a certain way and so form a group. Which of the following is different from the group?
    A. Rupali B. Roshni C. Rupika D. Rohit

Answer - Option D
10. Direction: Study the information given below and answer the questions based on it. There are 6 men, Ajeet, Akash, Amit, Abhi, Ankit, Aryan sitting around a circular table in a conference hall. Each one belongs to a different profession, i.e. Actor, Businessman, Dentist, Painter, Lawyer, Engineer not necessarily in the same order. All of them are from different cities, Bangalore, Delhi, Indore, Odissa, Mumbai, Haryana not in the same order. •Amit is neither from Bangalore nor on the immediate left of the person who is a Lawyer. Amit is not a lawyer. •The Actor, Lawyer and Painter are neither from Bangalore nor from Indore. •The person who is from Odissa and the one who is from Delhi is neither an Actor nor a Lawyer by profession. •Ajeet is a businessman and he is from Odissa. He is facing the Dentist. •The dentist is an immediate neighbour of Ankit and Aryan. The person who is on the left side of the person who is from Bangalore is not an Engineer. •The Actor is seated opposite to the person who is from Haryana and the person who is from Odissa is on the immediate left of Painter. •The Engineer is on the immediate right of the person, who is from Bangalore and on the immediate left of the person, who is a Lawyer. •Akash is not a Lawyer, while Aryan is not an Actor. Who sits opposite to the person who is 2nd to the left of the person who is Painter?
    A. Dentish B. Abhi C. Actor D. Engineer

Answer - Option C
1. Direction: Study the following information carefully and answer the questions given below- 8 persons P, Q, R, S, T, U, V and W are sitting around a circular table but not necessarily in the same order. Some of them are facing outward. They are working at four companies Tata, HCL, TCS and Moglix. Two persons are working in each company. • R sits 3rd to the left of W, who works in TCS, and both are facing the same direction. • V sits on the immediate right of Q, who works in Tata. • R and Q are not facing the same direction but R is an immediate neighbor of T, who is 4th to the left of V • T and V both are facing opposite directions but both work in the same company. • Those who work in Tata sit adjacent to each other but face opposite directions. • The person, who work in HCL sit opposite each other. • The immediate neighbor of T are not facing outward. • A person who works in TCS is an immediate neighbor of the both persons who work in Moglix. • S and U are immediate neighbor of W. • S is not facing the centre and works in Moglix. • The one who is on the immediate left of U is not facing the centre. U sits second to the right of R Who among the following work in TCS?
    A. S and U B. W and U C. V and R D. R and W

Answer - Option A
Explanation :
2. Direction: Study the following information carefully and answer the questions which follow.
There are 8 members of a family–A, B, C, D, E, F, G and H. All of them are sitting around a circular table facing the centre. They belong to 3 different generations and there are three married couples among them. They like different colors of clothes between white, red, black, yellow, orange, pink, green and silver but not necessarily in same order.
1) The son-in-law and maternal grandson of H are immediate neighbours. G cannot be an immediate neighbour of her sister-in-law. G likes black color. 2) B, a spinster, is sitting two places to the right of her grandfather E. Neither C nor E likes red color. A and his sibling D can’t sit together. Green color is liked by the brother-in-law of C. 3) There are only three persons sitting between the person who likes pink color and her aunt D. C, a married male, cannot be an immediate neighbour of either E or B. C does not like white or orange colors. 4) Red color is not liked by either D or H. F and B are not siblings. E does not like orange or silver colors. 5) There is only one person sitting between C and his brother-in-law. Silver color is not liked by the Son-in-law of H. Who is the maternal grandchild of E?
    A. B B. G C. D D. F

Answer - Option D
3. Direction: Read the following information carefully and answer the questions given below. Eight family members A, B, C, D, E, F, G and H are sitting around a circular table with equal distance between them but not necessarily in the same order. Some of them are facing the centre with some face outside i.e. opposite to centre. C sits second to the right of F, F faces the centre. A is a mother of C. Only two people sit between C and B i.e either from C’s right or C’s left. D’s Daughter, G sits second to the right of C. No female is an immediate neighbour of H. A’s mother H sits to the immediate right of B. G and B face opposite direction. Immediate neighbour of G face the same direction. C is the sister of E. F is father of E. Only three people sit between A’s Brother who is D and E. Neither D nor A is an immediate neighbour of F. E sits second to the right of A. E is the mother of B. Both H and E face a direction opposite to that of C. C is not an immediate neighbour of A's husband. D sits to the immediate right of which of the following person?
    A. G B. H C. Either option A or B D. Both option A or B

Answer - Option D
4. Directions: Study the following information to answer the given questions :
Eight friends A, B, C, D, E, F, G and Hare sitting around a circle facing the centre, not necessarily in the same order. F sits fourth to the left of B. A and H are immediate neighbours of F. C sits third to the left of A. G sits third to the right of E. What is D's position with respect to B?
    A. Immediate left B. Sixth to the right C. Second to the left D. Seventh to the left

Answer - Option A
5. Direction: Study the information given below and answer the questions based on it. Eight people P, Q, R, S, T, U, V and W are sitting around a circle facing outward. All of them belong to different cities viz—Mumbai, Kota, Jaipur, Indore, Pune, Goa, Raipur and Kolkata. S is sitting third to the right of U. The one who belongs to Kota sits on the immediate left of S, who doesn’t belong to Mumbai. Q sits fourth to the left of V. Neither Q nor V is an immediate neighbour of S. T belongs to Indore and sits third to the left of the one who belongs to Kota. The one who belongs to Goa sits second to the right of the one who belongs to Indore. The one who belongs to Raipur sits second to the right of S. P belongs to Jaipur and sits exactly between T and V. The one who belongs to Pune sits second to the left of the one who belongs to Jaipur. W sits third to the right of T. Who amongst the following belongs to Mumbai?
    A. U B. R C. W D. Q

Answer - Option A
Explanation :

6. Direction: Read the following information carefully to answer the given questions.
10 persons of a family I, J, K, L, M, N, O, P, Q, and R are sitting around a circular table. Some of them are facing the center while some are facing away from the center. Each of them is born in different year – 1964, 1969, 1971, 1981, 1983, 1984, 1985, 1987, 2004 and 2005 but not necessarily in the same order. Not more than 2 persons facing the same direction sit adjacent. There are only 3 generations of a family. No person in the third generation is married.
M’s wife was born in 1983. The one who was born in 1964 is the mother of M. K has 2 sons and 1 daughter. R is the brother of K and he sits between N and the one who born in 1969. K’s son-in-law was born in 1987. L and her husband sit opposite to each other and both of them faces opposite direction. N sits to the immediate left of L’s husband and faces outside. J, who is the son of L’s father, sits third to the right of N and faces inside. The one who was born in 1964 sits third to the right of L’s husband. I is facing outside. Only one person sits between the one who was born in 1985 and one who was born in 1964. 5 persons sit between one who was born in 1964 and K, who was born in 1969. L’s husband faces inside. P’s son sits adjacent to I and M and he is youngest among all. O was born in 2004 and sits immediate left of I, who is the wife of J’s father. L sits fourth left of the person who was born in 1984. O’s mother is the wife of M who sits immediate right of L’s husband. M sits second to the right of one born in 1964. K is facing outside. K’s wife was born in 1964. M was not born in 1985. O has no sibling. P is a male. M faces the same direction as his wife and also he is one year younger to his wife. J is younger to his uncle. How is O related to N?
    A. Nephew B. Niece C. sort D. can not be determined

Answer - Option D
7. Direction: Study the following information carefully and answer the questions given beside:
There are eight friends in which four females i.e. P, Q, R and S and four males viz. J, K, L and M were sitting around a Square table in which four persons sit at the corner while four sits at the middle side of the table. Persons sit at corner faces outside while other faces inside. They are like different actors and actress and work in a different department but not necessarily the same order.
K sits second to the left of P, who likes Ranbir Kapoor. The person who works in the post office department exactly opposite the one who like Katrina Kaif. S, who works in Lic department and second to the left of the one who likes Shahid Kapoor. The person who works in Railway department likes Salman Khan. J immediately right of Q who exactly opposite the one who work in Income tax department who also likes Katrina kaif. The one person in this group likes Amir Khan. The person who works in IT company second to the left of P and immediately neighbour of M who likes Katrina Kaif. The person who likes Deepika third to the left of the one who likes Shahid Kapoor. There are two persons between Q and the one who work in the banking department. The person who works in Lic department exactly opposite the one work in the Railway department. S, who likes Akshay Kumar third to the right of the one who likes Rajnikant. No three male person sits together. The person who works in gradeup content writer department sits exactly opposite of P. The person who works in Railways department he is a male person. The one person in this group work in the electricity department. Who is the following person likes Ashay kumar?
    A. L B. S C. R D. J

Answer - Option B
8. Direction: Study the information given below and answer the questions based on it.
There are 9 people-M, N, O, P, Q, R, S, T and U are sitting around a circular table out of which some are females and some are males. Some are facing towards the center and some are facing away from the centre. The following information is known about them.
Q’s neighbours face in same direction as that of O’s sister who is sitting to the immediate right of O. The female sitting between N and P is not Q. Q sits 4th to the right of R. R’s spouse is N. U sits between S and O and they face in opposite directions. S is a male sitting with two females and one of them has husband facing towards the centre who is sitting 3rd to the left of N. The person sitting 2nd to the left T is the wife of T and they face in same direction. M’s immediate neighbours are S and N. U’s spouse sits second to the right of U. P sits second to the left of N. The person 4th to the left of T sits faces in a direction opposite to that of T but same as P. How many persons facing away from the center?
    A. 3 B. 5 C. 4 D. 6

Answer - Option C
9. Direction: Study the information given below and answer the questions based on it.
Eight persons A, B, C, D, E, F, G and H sit around a circular table. They all are facing centre. They like different News Channels viz. Times Now, NDTV, India Today, CNBC, ABP News, Aaj Tak, IBN 7 and Zee News. They also like different colours like Red, Black and White. At least two persons like one colour.
B sits third to the right of the person who likes ABP News. Neither of them likes Black. F sits between B and E. None of them likes India Today and White. The persons who like Red are not neighbours. F likes neither Aaj Tak nor IBN 7. H sits opposite C. H likes Zee News and C likes Times Now. They like same colour. A and the one who likes CNBC are neighbours. Both of them like same colour. D sits second to the right of the one who likes India Today. Both the neighbours of C like same colour. Who likes NDTV?
    A. A B. B C. D D. F

Answer - Option D
10. Direction: Study the information given below and answer the questions based on it.
Seven friends P, M, J, Q, Z, V and F sit around a circular table. Some of them are facing the centre while some are facing away from the centre. They work in different companies viz. HCL, Mahindra, Infosys, IBM, Accenture, Cognisant and TCS.
V sits to the immediate left of F and they face opposite directions. The one who works in TCS sits third to the right of Z. Neither V nor F sits beside Z. Either V or F works in TCS. The one who works in IBM sits second to the left of Q. Q works in Infosys. Neither P nor M sits beside Q. Z faces the centre. Q does not sit adjacent to Z. Z does not work in IBM. M sits second to the right of V. Only two people sit between P and J. Q faces away from the centre. J and P work in neither HCL nor Mahindra. P and Z face the same direction. M and the one who works in TCS face same direction. Q sits to the immediate left of the one who works in Cognisant. Who sits second to the left of the one who works in Mahindra?
    A. The one who works in Infosys B. The one who works in Accenture C. The one who works in Cognisant D. Either option (b) or (c)

Answer - Option D
1. Direction: Study the information given below and answer the questions based on it.
Eight persons A, B, C, D, E, F, G and H use different brands laptop, Lenovo, Dell, Apple, Toshiba, Sony, Asus, Samsung and HP not necessarily in the same order. They are sitting around a circular table such that half of them are facing inside and half of them are facing outside of the circle. Each of them uses different brands shoes, Nike, Puma, Adidas and Reebok. There are exactly two persons having same shoes.
A and H sit opposite to each other and have same shoes. No other pair of persons sitting opposite to each other has the same shoes. The person who is opposite to F doesn’t face inside. C and both her immediate neighbors face in the same direction. D who has Asus sits opposite to E. B and C have Reebok and Nike shoes respectively. G who has Sony sits immediately to the left of A, who has Toshiba. D, who has Rebook, sits third to the left of the one who has HP. H has Lenovo. He is immediately to the left of both C who has Dell and B who has Apple. The person who has Samsung has Puma and is the only one who is between the two persons having Nike. A’s neighbours face in opposite directions to each other. E likes which of the following shoe?
    A. Reebok B. Nike C. Pume D. Adidas

Answer - Option C
2. Direction: Study the following information carefully and answer the questions based on it. Ten friends A, B, C, D, E, F, G, H, I and J are sitting at a rectangular table in such a way that Four of them sit at the corners, two each on the longer sides and one each on the smaller sides, but not necessarily in the same order, Some of them are facing the centre while the rest are not facing the centre. Not more than two friends sitting together face the same direction. A sits on one of the smaller sides and third to the right of the F who faces outward. I is not an immediate neighbour of A, B or F, but sits on the immediate right of C, who is not facing the centre. Only two among four sitting on the corners face outward. Those who sit on the smaller side face inwards. H and D are sitting on the immediate left and third to the left of J respectively who faces outward. I is sitting at one of the four corners. E sits on the immediate left of D and is not an immediate neighbour of C. A and E face the same direction. D and G sit diagonally and face opposite directions. H faces the centre. Five of them face the centre. Which of the following groups sit on the corners of the rectangular table?
    A. DIHJ B. JDIG C. HDGI D. HDGC

Answer - Option B
3. Direction: Study the following information carefully and answer the given question.
Nitin, Jahanvi, Lalit, Mohak, Piya, Khanak, Omkar and Qasim are sitting around a circular table facing the centre. Each of them was born in different years, 1981, 1983, 1985, 1987, 1989, 1991, 1993 and 1995, not in this particular order.
Mohak is sitting second to the right of Khanak. Lalit is sitting third to the right of Jahanvi. Only person who born in 1991 is sitting between Jahanvi and Khanak. Nitin is the eldest person among all, is not an immediate neighbour of Jahanvi or Mohak. Qasim is older than only Mohak. Qasim is sitting second to the left of Piya. Piya does not sit beside Nitin. Lalit does not sit beside Nitin. The sum of the ages of Jahanvi and Khanak is equal to that of Lalit and Omkar. One of the neighbours of Nitin was born in 1983. Khanak is older than Jahanvi. (Ages are to be calculated taking 2017 into consideration).
How many people are younger than Lalit?
    A. Five B. Three C. Four D. Two

Answer - Option B
4. Directions: Study the following information and answer the questions given below.
Eight people E, F, G, H, J, K, L and M are sitting around a circular table facing the centre. Each of them is of a different profession Chartered Accountant, Columnist, Doctor, Engineer, Financial Analyst, Lawyer, Professor and Scientist but not necessarily in the same order.
F is sitting second to the left of K. The Scientist is an immediate neighbour of K. There are only three people between the Scientist and E. Only one person sits between the Engineer and E. The Columnist is to the immediate right of the Engineer. M is second to the right of K. H is the Scientist. G and J are immediate neighbours of each other. Neither G nor J is an Engineer. The Financial Analyst is to the immediate left of F. The Lawyer is second to the right of the Columnist. The Professor is an immediate neighbour of the Engineer. G is second to the right of the Chartered Accountant. Who is sitting second to the right of E?
    A. The lawyer B. G C. H D. F

Answer - Option B
Explanation :
F is sitting second to the left of K.
M is second to the right of K.
The Scientist is an immediate neighbour of K.
H is the Scientist.
There are only three people between the Scientist and E.
Only one person sits between the Engineer and E.
The Professor is an immediate neighbour of the Engineer.
The Columnist is to the immediate right of the Engineer.
The Lawyer is second to the right of the Columnist.
G and J are immediate neighbours of each other.
Neither G nor J is an Engineer.
The Financial Analyst is to the immediate left of F.
G is second to the right of the Chartered Accountant.

G is second to the right of E.
5. Direction: Read the information carefully and answer the s given below.
Eight members of a family viz. A, B, C, D, E, F, G and H are sitting around a circular table facing the center, but not necessarily in the same order. They belong to different state. Each of them are married to different girls and have different number of children (1 to 9 number). Maximum number of children is nine and minimum one. All the given information is not necessarily in the same order. The one who has three children sits third to left of the one who married is to Simran. C married Sona and has six children. The one who belongs to U.P face the one who belongs to Jharkhand who sits third to the right of the one who has four children. The one who belongs to Gujarat sits immediate next of the one who has two children. H sits third to left of F who married with Kavita. G face the one who has one child who sits second to right of A who not belongs to Kerala. E sits second to right of the one who has six children. The one who has 6 children faces the one who married Ruchi. The one who belongs to Goa has maximum number of children and face the one who has five children. The one who married with Kavita sits third to the right the one who married Ruchi who belongs to Bihar. The one who belongs to Karnataka has six children and face H who sits second to right the one who married Khusbu. D who married Ambica belongs to M.P and face the one who belongs to Kerala. There are two persons sitting between D and C. The one who belongs to Goa is nor an immediate neighbour of the one who belongs to M.P neither the one who belongs to Karnataka. The one who married Surbhi has even number of children but does not sit immediate next to the one who married Alpa. Which of the following persons married to Alpa?
    A. G B. A C. B D. D

Answer - Option B
6. Direction: Study the following information carefully to answer the questions.
Eight friends: A, B, C, D, E, F, G and H – went to a restaurant and seated around a circular table facing the center and each one of them have a taste in a different type of a cuisine: Chinese, Mughlai, American, Italian, Thai, North Indian, south Indian and continental.
1) G is two positions away from E. There is one person sitting between E and D and D doesn’t like Thai food. 2) The one who likes American is seating to the immediate right of D. 3) The one who likes Italian sits opposite to the one who likes north Indian and one of them is C. 4) The one who likes Mughlai sits opposite to H. A like neither American nor Thai. 5) D doesn’t like Mughlai. 6) The one who likes Thai and the one who likes South Indian sits opposite to each other and neither of them are B. 7) C sits to the immediate left of the one who likes Thai and G likes Chinese. 8) A doesn’t sit opposite to the one who likes continental and F sits to the three positions to the left of E, who likes north Indian. Who is sitting opposite to B?
    A. A B. C C. D D. H

Answer - Option D
7. Direction: Study the following information carefully and answer the questions given below.
There are 12 persons i.e. M, N, O, P, Q, R, T, W, Z, Y, J and K are sitting around circular arrangement. There is an outer circle with eight people sitting in it and they all face the centre. And an inner circle inside the outer circle with four people sitting in it and all are face outside the centre. The persons sitting on inner circle face the persons sitting on outer circle. Each of them study in different class ( i to xii) and get different marks.
Note: Any person sitting between two other persons means that they are sitting on the same circle.
The one who study in class (ii) sits second to right of M who has got 20% marks. The one who studies in class (vi) face the one who has got 60% who sits second to left of M.Y gets 85% marks sits second right of the one who gets 90% marks who face the one who study in class (i). J sits 3rd to the left of K who faces P. P's neighbor faces M. There is one person sitting between J and Q. P sits to the immediate left of W who faces the person who sits exactly in between J and N. R faces T's neighbor. The one who gets 65% marks sits to second left of N who sits second left of the one who studies in class (ix). The one who faces R studies in class (xi) who sits third left of the one who study in class v. T sits 2nd to the right of N. O faces W's neighbor. Q and Y are not neighbors. Q who sits to second right of the one who gets 55% marks who sits second right of the one who study in class (viii). The one who study in class (iii)sits to second right of Y who study in class (vii). The maximum marks scored was 95%. Q gets 27% marks and studies in class (xii). The one who gets 50% marks sits second to the left of R who is an immediate neighbour of the one who gets 80% marks and the one who gets 60% marks. One person in this group who gets 30% marks and also studies in class (iv). The one who studies in class (viii) gets more than 50% marks. Which of the following person Study in class x?
    A. N B. K C. R D. W

Answer - Option D
8. Direction:Study the information given below and answer the questions based on it.
Eight persons M, N, O, P, Q, R, S and T are sitting around a circular table and they were born in the same month but different dates i.e. 6th, 9th, 12th, 16th, 19th, 24th, 27th and 29th, not necessarily in the same order.
Note: The persons whom are facing inside were born on an even number date and the persons whom are facing outside were born on an odd number date. T was born on 24th and Q is 2nd to the left of T. Two persons are sitting between Q and the one who was born on 12th. P is 2nd to the left of the one who was born on 12th. Three persons are sitting between O and P. N was born on 9th and M is 2nd to the right of N. R is immediate right of M. The one who was born on 29 is 3rd to the left of M. The one who was born on 27th is 2nd to the right of the one who was born on 29th. P is facing outside. Q was born after M. Q, O are immediate neighbors. Who among the following is 3rd to the left of S?
    A. M B. N C. R D. Q

Answer - Option A
9. Direction: Study the information given below and answer the questions based on it.
There are eight friends Sanjeet, Sumit, Suman, Suraj, Suresh, Saurabh, Shambhu and Shyam sitting around a circular table facing the centre and playing cards but not necessarily in the same order. All of them have a favourite card among these eight cards. Out of these eight cards, four are Kings and four are Queens of spade, club, diamond and heart.
Sanjeet likes the Queen of spade and is not an immediate neighbour of the one who likes the King of club. The one who likes the Queen of diamond sits on the immediate left of Sumit, who likes the Queen of club. Suman likes the King of club and sits third to the left of Suresh. The one who likes the King of spade and the one who likes the Queen of diamond are immediate neighbours of each other but both of them are the neighbours neither of Suresh nor of Suman. The one who likes the King of diamond and the one who likes the Queen of spade are immediate neighbours of each other. Neither Suresh nor Suraj likes the King of diamond. Only Saurabh sits between the one who likes the Queen of diamond and the one who likes the Queen of heart. Shambhu sits third to the left of the person who likes the King of diamond. Who likes the King of spade?
    A. Saurabh B. Shambhu C. Shyam D. None

Answer - Option A
Explanation :

10. Direction: Study the following information carefully and answer the questions which that follow.
A family consists of 8 members A, C, L, M, P, R, S and U, who are sitting around a circle facing the centre but not necessarily in the same order. C sits second to the left of U’s husband. No female is an immediate neighbor of C. The daughter of M sits second to the right of R. R is the sister of S. R is not an immediate neighbor of U’s husband. Only two person sits between A and U. A is the father of S. U’s brother M sits on the immediate left of U’s mother. Only one person sits between U’s mother and P. Only one person sits between S and U. S is the mother of L . S is not an immediate neighbor of P. How is U related to R?
    A. Mother B. Father C. Uncle D. Grandmother

Answer - Option A
1. Direction: Study the following information and answer the questions given below. There are seven cars A,B,C,D,E,F,G to be parked in a cellular facing the north direction from left to right and the cars are parked as follows. There are three cars between the C and E cars and C is to the left of E. Both of them are not on ends. There is only one car parked between A and C cars. D car is parked immediate left of E car. B car is parked immediate left of C car. G car is not parked adjacent to E car. Who has parked their cars on extreme left?
    A. B B. C C. E D. D

Answer - Option A
Explanation :
A B C G F D E
We will first list out all the points given
1. _C _ _ _ E_
2. _C_A_ E _
3. _C_A D E_
4. B C _ A D E _
5. B C G A D E F
Hence, the final arrangement is
B C G A D E F
2. Direction: Study the information given below and answer the questions based on it. There are two parallel rows with a sitting capacity of six on each side but only five people are sitting in each row keeping one seat vacant in each row. In row-1, R1, R2, R3, R4 and R5 are sitting and facing south. In row 2, A1, A2, A3, A4 and A5 are sitting and facing north. All of them like to visit different tourist places i.e. Nanital, Mussoorie, Assam, Shimla, Goa, Kerala, Ooty, Shillong, Dehradun and Gujarat.
  • A5 sits third to the right of A2 and wish to go Shimla.
  • Only two people sit between A1 and the vacant seat.
  • A1 does not like to go Goa or Assam.
  • R5 is an immediate neighbour of R3.
  • R2 likes to go Gujarat. The one who is willing to go Assam faces the one who willing to go to Shillong.
  • The one who likes Assam sits opposite the one who sits third right of the one who sits opposite A5.
  • R3 is not an immediate neighbour of R4.
  • A4, who likes neither go to Goa nor Kerala, does not face the vacant seat
  • Neither A5 nor A2 sits at the extreme ends. R4 faces A2.
  • Vacant seat are not opposite each other.
  • Two person sit between A4 and A3.
  • The one who likes to go Dehradun faces the one who likes to go Shimla
  • The person who likes to go Nanital and Shillong are adjacent to each other.
  • The vacant seat of row one is not adjacent to R4.
  • A1 sits at one of the extreme end.
  • A2 does not like Nanital and Shillong.
  • The vacant seat of row one does not face A5 and also it is not at the extreme ends.
  • A3 is an immediate neighbour of A5, who faces the person who likes to go Dehradun.
  • R2 faces the vacant seat in row-2 and sits third to the right of R3.
  • R3 likes to go Ooty.
Who sits second to the right of the one who faces vacant seat of row one?
    A. R2 B. R3 C. R5 D. A1

Answer - Option D
Explanation :
As per given information let’s write confirm condition first and draw accordingly:
In row-1, R1, R2, R3, R4 and R5 are sitting and facing south.
In row 2, A1, A2, A3, A4 and A5 are sitting and facing north.
R3 likes to go Ooty.
R2 likes to go Gujarat. The one who is willing to go Assam faces the one who willing to go to Shillong.
R5 is an immediate neighbour of R3.
The vacant seat of row one is not adjacent to R4.
A5 sits third to the right of A2 and wish to go Shimla.
Only two people sit between A1 and the vacant seat.
A1 does not like to go Goa or Assam.
The one who likes Assam sits opposite the one who sits third right of the one who sits opposite A5.
R3 is not an immediate neighbour of R4.
A4, who likes neither go to Goa nor Kerala, does not face the vacant seat
Neither A5 nor A2 sits at the extreme ends. R4 faces A.
Vacant seat are not opposite each other.
Two people sit between A4 and A3.
The one who likes to go Dehradun faces the one who likes to go Shimla
The person who likes to go Nanital and Shillong are adjacent to each other.
A1 sits at one of the extreme end.
A2 does not like Nanital and Shillong.
The vacant seat of row one does not face A5 and also it is not at the extreme ends.
A3 is an immediate neighbour of A5, who faces the person who likes to go Dehradun.
R2 faces the vacant seat in row-2 and sits third to the right of R3

3. Direction: Study the information given below and answer the questions based on it.
Twelve persons are sitting in two parallel rows containing six persons in each row. In row1 M, N, O, P, Q, and R are sitting and all of them are facing north. In row2 A, B, C, D, E and F are sitting and all of them are facing south. In the given sitting arrangement, each member is seated in a row faces another member of the other row but not necessarily in the same order. No two successive friends are sitting together according to alphabetical order. For example U does not sit with V; similarly V does not sit with W and so on. Each of them also lives in different states like MP, UP, Kerala, Delhi, Punjab, Sikkim, Manipur, Bihar, Bengal, Maharashtra, UK, HP but not necessarily in the same order.
A sits third to the left of E and none of them sits on the extreme ends of the line. Immediate neighbour of A faces the person who lives in HP. There is one person sitting between the one who lives in HP and R. Immediate neighbour of E faces Q. Immediate neighbour of B faces P. Immediate neighbour of O faces the one who lives in Delhi. There are two persons sitting between the one who lives Delhi and Manipur. D dies not sit adjacent to B, who does not sit at any of the ends. One of the immediate neighbour of A lives in UK. There are two persons sitting between the one who likes UK and Punjab. The immediate neighbour of one who lives in Punjab faces the one who live in Maharashtra. The one who lives in Bengal sits third to the right of one who lives in Bihar. A does not live in Bengal. The one who lives in Sikkim sits on the immediate right of one who lives in Punjab. The immediate neighbour of one who lives in Sikkim does not live in Kerala. A does not live in Kerala.D faces the one who is third to the right of one who lives in Bihar. The one who lives in Kerala faces the person who lives in UP. Who among the following lives in MP?
    A. M B. N C. O D. Q

Answer - Option D
4. Direction : Study the information given below and answer the questions based on it.
Nine persons P, Q, R, S, T, U, V, W and X of a family are sitting in a row. Some of them are facing north while some of them are facing south. There are three generation in the family. No two persons are sitting according to their names in the alphabetical order (For example P does not sit with Q. Q does not sit either with P or with R and so on).
V sits second to the right of P. S’s spouse does not face south. W sits third to the right of his wife. There is only one male whose both immediate neighbours are females. None of the couples are from third generation. W has only one married son. The unmarried maternal aunt of X’s son sits at one of the extreme ends of the row. The daughter of S and the son – in – law of Q are from different generation. Both Q and S are sitting together but one of them is facing north. Only three persons sit between U and V and both are same gender. U and V face the same direction. Only one couple sit together but they face opposite direction and also neither of them sits end. U is not immediate neighbour of P, who sits exactly in the middle. Q faces north. One of the immediate neighbours of P is a female. S has no uncle or aunt and also does not have mother – in – law or sister – in – law. X is not son – in – law of Q. S has a brother – in – law. T is a female and is facing the same direction as P. Both the immediate neighbours of P are facing the same direction. U is not the son – in – law of W. Both the persons sitting at the extreme ends of the row are of the same gender. S and X are not a married couple. T is not the sister of S. S is the only brother of R and does not sit at end. S’s daughter sits to the right of V, who faces north. R and T face opposite directions. Who is daughter – in – law of Q?
    A. R B. P C. S D. T

Answer - Option B
5. Direction : Study the following information to answer the given questions: Ten people are sitting in two parallel rows having five persons each, in such a way that there is an equal distance between adjacent persons. In row 1 - F, G, H, I and J are seated (but not necessarily in the same order) and all of them are facing north. In row 2 -A, B, C, D and E are seated (but not necessarily in the same order) and all of them are facing south. Therefore, in the given seating arrangement, each member seated in a row faces another member of the other row. The one who faces I sits third to the right of A. Only one person sits between I and F. B is not an immediate neighbour of A. B does not sit at any of the extreme ends of the line. The one who faces C sits on the immediate right of G. C is not an immediate neighbour of B. D faces J.
Four of the following five are alike in a certain way based on the given seating arrangement and thus form a group. Which is the one that does not belong to that group?
    A. D B. A C. H D. I

Answer - Option D
Explanation :
The one who face I its third to the right of A.
Only one person sits between I and F.
B is not an immediate neighbour of A.
B does not sit at any of the extreme ends of the line.
The one who faces C sits on the immediate right of G.
C is not an immediate neighbour of B. D faces J.

6, Direction : Study the information given below and answer the questions based on it. Twelve people are sitting in two parallel rows containing six people each in such a way that there is an equal distance between adjacent persons. In row 1, A, B, C, D, E and F are sitting and all of them are facing South. In row 2, P, Q, R, S, T and V are sitting and all of them are facing North. Therefore, in the given seating arrangement, each member of a row faces another member of the other row. V sits third to the right of S. S faces F and F does not sit at any of the extreme ends of the lines. D sits third to the right of C. R faces E. The one facing E sits third to the right of P. B and P do not sit at the extreme ends of the lines. T is not an immediate neighbour of S and F is not an immediate neighbour of D.
Who among the following faces D?
    A. T B. P C. Q D. R

Answer - Option A
Explanation :
V sits third to the right of S.
S faces F and F does not sit at any of the extreme ends of the lines.
D sits third to the right of C.
R faces E.
The one facing E sits third to the right of P.
B and P do not sit at the extreme ends of the lines.
Direction (7): Study the following information carefully and answer the given questions:
7 people A, B, C, D, E, F and G are sitting equidistant from each other in a row facing north but not necessarily in the same order.
B sits at one of the extreme ends of the row. Neither C nor D sits to the immediate left of G. Only 3 persons sit between B and G. Only 2 persons sit between C and D. E sits to the immediate right of D, who is an immediate neighbor of neither A nor F. Equal number of persons sit between B and C as that in A and F.
How many persons sit between C and G?
    A. None B. 1 C. 2 D. 3

Answer - Option B
8. Direction : Study the following information carefully to answer the given questions: Twelve people are sitting in two parallel rows containing six people each, in such a way that there is equal distance between adjacent persons. Thus each person from row 1 is facing another person from row 2. Three persons from each row face north and three persons from each row face south. No two persons sitting opposite to each other face the same direction (if one faces north then other is facing south and vice-versa). In row-1 A, B, C, D, E and F are seated. They are seated such that no person sits next to other person as in English alphabetical series. (For example B is not sitting next to A and C and so on).In row-2 P, Q, R, S, T and U are seated. (Q is not sitting next to P and R and so on). They like different number of chocolates such as 1,2,3,4,5,6,7,8,9,10,11 and 12 but not necessarily in the same order. B sits second left to A but none of them sits at the extreme ends. Both A and B face the same direction. The number of chocolates liked by F is square of the number of chocolates liked by E. Three persons sit between B and E. Two persons sit between E and D. P likes 5 chocolate and sit second to the left of T who sits at one of the extreme ends. A sits opposite to P likes 8 chocolates. P faces north. The person sitting opposite to U likes 9 chocolates. The person sitting at both the extreme ends face the same direction. Two persons sit between P and U. U likes 7 chocolates. Two person sits between T and S who likes 6 chocolate. C sits second to the left of D. The number of chocolates liked by D is square of the number of chocolates liked by Q. The one who likes 12 chocolates is sitting at the extreme end and is immediate neighbour of the one who likes 1 chocolate. The person who likes 2 chocolates sits opposite to the one who likes 11 chocolates.
Who among the following sit at the extreme end?
    A. A B. Q C. F D. P

Answer - Option C
Direction (9): Study the following information carefully and answer the given questions. A certain number of people were sitting in a row facing North and they like different colours. i) Suraj sits fourth from the left end. ii) Number of persons between Suraj and Tanmay are same as that of people between Abhay and Sushant. iii) Sushant likes Red colour and Sumit likes Green colour. iv) Person sitting third from left end of row likes violet colour. v) Rohit was sitting 11th from left end of row. vi) Sushant is the immediate neighbor of Rohit. vii) The person sitting adjacent to Tanmay and Abhay likes Orange Colour. viii) Pulkit is sitting 4th to the left of Abhay. ix) Sushant is sitting 7th to the right of Pulkit. x) Number of Persons between Pulkit and Abhay are equal to that of persons between Pulkit and Sumit. xi) Number of Persons between Sumit and Mayank is same as that of persons between Rohit and Mayank. xii) Persons sitting at extreme ends like Red and Green Colour.
how many persons are there in the row?
    A. 13 B. 14 C. 15 D. None of these

Answer - Option D
10. Direction: Study the following information carefully to answer the given questions.
Ten people sit in a metro. Five of them i.e. Amit, Vipul, Deepu, Ravi and Roshan face north while Shashi, Monu, Pankaj, Ramu and Tinku face south. Each person faces the other in the row, which is opposite. They like different fruits i.e. grapes, cherry, apple, orange, pineapple, banana, kiwi, strawberry, mango and guava.
Shashi sits opposite to the one who likes banana and sit at one of the extreme ends. The one who likes kiwi, is not Monu, sits second to the right of Shashi. Tinku sits between Monu and the one who likes Cherry who is not Shashi. Vipul likes orange and does not sit at the extreme ends and sit opposite to the one who likes apple. The one who likes pineapple sits opposite to the one who is to the immediate left of Vipul. The person who likes strawberry faces Ramu and is not Deepu. Pankaj does not sit at any of the extreme ends, but sits opposite to the one who likes guava who sits beside Amit and the one who likes mango. Ravi neither likes guava nor strawberry. Who among the following likes grapes?
    A. Shashi B. Monu C. Amit D. Vipul

Answer - Option A
1. Arrange the given words in the sequence in which they occur in the dictionary. i. Permanence ii. Permanent iii. Permafrost iv. Permeability
    A. ii, iii, iv, i B. iii, ii, i, iv C. iii, i, ii, iv D. i, ii, ii, iv

Answer - Option C
Explanation :
Alphabetical order is:
iii. Permafrost
i. Permanence
ii. Permanent
iv. Permeability
Hence, the order is (iii, i, ii, iv)
2. In the following question, select the missing number from the given series. 6 10 9 7 2 1 3 4 ?
    A. 9 B. 8 C. 6 D. 7

Answer - Option C
Explanation :
Column I: 6 + 7 + 3 = 16
Column II: 10 + 2 + 4 = 16
Similarly,
Column III: 9 + 1 + ? = 16
? ? = 16 – 9 – 1 = 6
Thus the missing number is 6.
3. Arrange the given words in the sequence in which they occur in the dictionary. i. Manufacture ii. Manualism iii. Manumission iv. Manual
    A. iV, ii, i, iII B. iii, ii, i, iv C. iii, i, ii, iv D. i, ii, ii, iv

Answer - Option A
Explanation :
MANU is common to all. Hence we will look at the next letters.
Out of ‘f’ ‘a’ ‘m’ and ‘a’, we can see that again ‘a’ is common. So let’s first compare the next letters after ‘a’.
We have ‘l’ and ‘l’ again common. So jump for next. Now Manual is over and hence it is an indication that it will come first.
So 4.
Then 2.
Then out of ‘f’ & ‘m’, ‘f’ will come first and then ‘m’.
So, 1
And at last 3
Hence the sequence is 4,2,1,3
Hence A
4. Select the related word/letters/number from the given alternatives. Subhas Chandra Bose : Orissa :: Mahatma Gandhi : ?
    A. Bihar B. Jammu and Kashmir C. Gujarat D. Delhi

Answer - Option C
Explanation :
Subhash Chandra bose was born in Cuttak, Orissa and hence we need to match mahatma Gandhi to his birth place. Out of all the options present, as we all know Gandhi was born in Porbandar which is in present day Gujarat. Hence C
5. A series is given with one term missing. Choose the correct alternative from the given ones that will complete the series. JN, OR, UW, BC, ?
    A. KM B. JJ C. JK D. KJ

Answer - Option B
Explanation :
J + 5 = O
O + 6 = U
U + 7 = B
B + 8 = J
N + 4 = R
R + 5 = W
W + 6 = C
C + 7 = J
Hence JJ i.e. B
6. Select the odd word/letters/number /number pair from the given alternatives.
    A. IJKPQP B. FGHUTS C. BCDWXY D. KLMPON

Answer - Option C
Explanation :
All except ‘BCDWXY’ are written in two part where 1st part is in normal order and 2nd part is
written in reverse order of their occurrence in English language.
Hence, the correct option is C.
7. Ankita travels 35 km towards the west, takes a right turn and travels 50 km more. Next, she takes another right turn and travels 35 km in that direction. How far is she now from her original position?
    A. 75 km B. 25 km C. 50 km D. 20 km

Answer - Option C
8. The age of Ravi is twice the age of Prakash and three times the age of Kumar. If the age of Prakash is 24 years, then what will be the age of Kumar after 4 years ?
    A. 16 years B. 20 years C. 24 years D. 22 years

Answer - Option B
Explanation :
Going by the conditions given:
Ravi’s age = 2 × 24 ( Prakash’s age) 48
Kumar’s age = 48 (Ravi’s age) ÷ 3 16
After 4 years, Kumar will be: 16 + 4 = 20
Hence, the correct option is B.
9. A series is given with one term missing. Choose the correct alternative from the given ones that will complete the series. 64, 100, 169, 289, ?
    A. 484 B. 444 C. 404 D. 468

Answer - Option A
Explanation :
64 = 8 * 8
And 8 + 2 = 10
10 * 10 = 100
And 10 + 3 = 13
13 * 13 = 169
And 13 + 4 = 17
17 * 1 = 289
And 17 + 5 = 22
22 * 22 = 484
Hence A
10. Arrange the given words in the sequence in which they occur in the dictionary. i. Parachute ii. Pacable iii. Pachometer iv. Pabloism
    A. iV, ii, i, iII B. iii, ii, i, iv C. iii, i, ii, iv D. iV, ii, iiI, i

Answer = Option D
Explanation :
Arranging the words in the order they appear in dictionary:
iv. Pabloism
ii. Pacable
iii. Pachometer
i. Parachute
Hence, the correct option is D.
1. Which one set of letters when sequentially placed at the gaps in the given letter series shall complete it? T, R, P, N, L, _?_
    A. J, G B. J, H C. K,H D. K, I

Answer - Option B
2. Direction: Find the missing number in the following series: 4320, 720, ?, 36, 12, 6
    A. 144 B. 24 C. 72 D. 48

Answer - Option A
Explanation :
[latex]\frac {4320}{6}[/latex] = 720
[latex]\frac {720}{5}[/latex] = 144
[latex]\frac {144}{4}[/latex] = 36
[latex]\frac {36}{3}[/latex] = 12
[latex]\frac {12}{2}[/latex] = 6
Thus 144 is the required number.
Hence Option A is correct
3. In each of the following questions, choose the correct alternative from the given ones that will complete the series. 480, ?, 24, 8, 4
    A. 98 B. 96 C. 104 D. 88

Answer - Option B
Explanation :
If we look the series in reverse order then,
8 = 4 × 2;
24 = 8 ×3;
[latex]\rightarrow[/latex] Multiplier increased by ‘1’ in every next step.
Therefore ?= 24 × 4 = 96
[latex]\rightarrow[/latex] 480 = 96 × 5
Hence ‘?’ = 96
4. In the following question, select the related letters from the given alternatives. AF : BE : : ? : DH
    A. CH B. CG C. DG D. CI

Answer - Option D
Explanation :

Similarly,

Thus DH is related to CI.
5. Arrange the given words in the sequence in which they occur in the dictionary. i. Speaker ii. Surreptitious iii. Spontaneous iv. Spurious
    A. iv, ii, i, iii B. iii, ii, iv, i C. iii, ii, iv, i D. i, iii, iv, ii

Answer - Option D
Explanation :
Alphabetical order is:
i. Speaker
iii. Spontaneous
iv. Spurious
ii. Surreptitious
So the correct order is (i, iii, iv, ii).
6. If PALE is written as RCNG, how can LEAP be written in that code?
    A. NGCR B. RCGN C. CRNG D. NCRG

Answer - Option A
Explanation :
PALE is written as RCNG
P [latex]\rightarrow[/latex] R (P+2)
A [latex]\rightarrow[/latex] C (A+2)
L [latex]\rightarrow[/latex] N (L+2)
E [latex]\rightarrow[/latex] G (E+2)
Similarly
LEAP [latex]\rightarrow[/latex] ?
L [latex]\rightarrow[/latex] N
E [latex]\rightarrow[/latex] G
A [latex]\rightarrow[/latex] C
P [latex]\rightarrow[/latex] R
Hence LEAP [latex]\rightarrow[/latex] NGCR
7. Direction: Two statements are given followed by two conclusions I and II. You have to consider the statements to be true even if they seem to be at variance from commonly known facts. You are to decide which of the given conclusions, if any, follow from the given statements. Indicate your answer. Statements :
    1. AIDS is a killer disease. 2. It is easy to prevent AIDS than to treat it.
Conclusions :
    I. AIDS prevention is very expensive. II. People will not cooperate for AIDS prevention.

    A. Only conclusion I follows B. Only conclusion II follows C. Neither conclusion I nor II follows D. Both conclusions I and II follow

Answer - Option C
Explanation :
None of the Conclusions follows. If one take precution he/she may prevent it. It does not imply that AIDS prevention is very expensive.
8. Direction: Select the one which is different from the other three responses.
    A. Tangent B. Arc C. Radius D. Diagonal

Answer - Option D
Explanation :
Diagonal is related to such geometrical figures which consist of lines. All others are related to circle.
9. Select the related word/letters/number from the given alternatives. 1272: 212 :: 720: ?
    A. 132 B. 195 C. 150 D. 120

Answer - Option D
Explanation :
Since 1272 ÷ 6 = 212
Likewise, 720 ÷ 6 = 120
Hence, the correct option is D.
10. If the Universal Set U = {1, 2, 3, 4, 5, 6, 7, 8} and A = {1, 2, 3, 4}, then [latex] {A}^{C}[/latex] is equal to?
    A. {5, 6, 7, 8} B. {5, 6, 1, 2} C. {5, 6, 2, 3} D. {5, 6, 3, 4}

Answer - Option A
Explanation :
[latex] {A}^{C}[/latex] = U - A = {1, 2, 3, 4, 5, 6, 7, 8} - {1, 2, 3, 4}
[latex] {A}^{C}[/latex] = {5, 6, 7, 8}
1. DirectionIn the following question, two statements are given each followed by two conclusions I and II. You have to consider the statements to be true even if they seem to be at variance from commonly known facts. You have to decide which of the given conclusions, if any, follows from the given statements. Statements: (I) All dogs are rats. (II) Some rats are lizards. Conclusion: (I) Some dogs are lizards. (II) Some lizards are dogs
    A. Conclusion I follows B. Conclusion II follows C. Neither I nor II follows D. Both I and II follows

Answer - Option C
2. A series is given with one term missing. Choose the correct alternative from the given ones that will complete the series. FI, KM, PQ, ?
    A. MQ B. ZA C. TO D. UU

Answer - Option D
Explanation :
There are 26 alphabets in English and if we assign numbers to each and every alphabet starting from ‘A’, ‘B’, ‘C etc., it will appear to be:
A = 1, B = 2, C = 3, D = 4……. likewise, till Z = 26
F + 5 = K
K + 5 = P
P + 5 = U
I + 4 = M
M + 4 = Q
Q + 4 = U
Hence, the correct option is D.
3. If"#" means "subtraction", "&" means "division", " @ " means "addition" and "%" means "multiplication", then 217 & 7 # 3 @ 2 % 7 = ?
    A. 21 B. 19 C. 22 D. 42

Answer - Option D
Explanation :
# means -,
& means ÷,
@ means + and
% means x
After changing the signs we get,
[latex] {217}^{7 - 3}[/latex] + 2 x 7
= 31 – 3 + 14
= 28 + 14
= 42
Hence, option D is the correct response.
4. A series is given with one term missing. Select the correct alternative from the given ones that will complete the series. ZA, XC, TG, NM, ?
    A. KL B. FU C. LM D. TI

Answer - Option B
Explanation :
The series is as follows,

Thus, the next term in the series is FU.
5. In the following question, select the odd word pair from the given alternatives.
    A. Tea : Beverages B. Legumes : Nodules C. Beans : Pulses D. Rice : Cereals

Answer - Option B
Explanation :
Except B, In all other pairs, second element denotes the class to which the first belongs.
Hence, option B is the correct response.
6. Sandeep has a brother named Ankur. Sandeep is the son of Kishan. Baljor is Kishan's father. How is Ankur related to Baljor?
    A. Grandson B. Brother C. Son D. Grandfather

Answer - Option A
Explanation :
Baljor is Kishan’s father.
Kishan is Sandeep’s father.
Ankur is Sandeep’s brother.
Therefore, Ankur and Sandeep are Grandson of Baljor.
Hence, the correct option is A.
7. Which of the conclusions can be drawn from the given statements? Statements :
    I: All the students in my class are intelligent. II: Kaushik is not intelligent

    A. Kaushik is not a student of my class. B. All other than Kaushik are intelligent C. Some students are not intelligent D. Non-intelligent are not students

Answer - Option A
Explanation :
All students (without exception) of a particular class are intelligent and Kaushik is not intelligent. So, Kaushik is not a student of that class.
Hence Option A is correct
8. A series is given with one term missing. Select the correct alternative from the given ones that will complete the series. AMN, BOP, CQR, ?
    A. BAS B. DST C. EQP D. FRS

Answer - Option B
Explanation :
A(+1) = B ; B(+1) = C; C(+1) = D
M(+2) = O; O(+2) = Q; Q(+2) = S
N(+2) = P; P(+2) = R; R(+2) = T
Hence ? = DST
9. P, Q, R and S are four friends. P is shorter than Q but taller than R who is shorter than S. Who is the shortest among all?
    A. P B. Q C. R D. S

Answer - Option C
Explanation :
Q > P > R and S > R
The shortest is R
10. Q is the son of P, X is the daughter of Q, R is the aunty (Bua) of X and L is the son of R, then what is L to P?
    A. Grandson B. Granddaughter C. Daughter D. Nephew

Answer - Option A
Explanation :
L is the son of R and R is the daughter of P.
Therefore L is the grandson of P.
Hence Option A is correct
1. Find the odd one
    A. 48 B. 30 C. 18 D. 34

Answer - Option D
Explanation :
All the numbers are divisible by 3 except 34
2. If FADE is coded as 1654 then how can GAGE be coded?
    A. 2834 B. 1757 C. 2814 D. 1824

Answer - Option B
Explanation :
With the reference to the place value of the given alphabets.

Clearly, FADE- 1654, the place of F and A interchanged, similarly position of D and E interchanged.
therefore, GAGE will be coded as 1757
Hence, option B is the correct answer.
3. A series is given, with one term missing. Choose the correct alternative form the given ones that will complete the series.
    A. SXI B. TXI C. SYJ D. SXJ

Answer - Option A
Explanation :
Clearly,
Every letter in the term is increasing by +1,
O+1 = P, P + 1 = Q, Q + 1 = R, R + 1 = S
T + 1 = U, U + 1 = V, V + 1 = W, W + 1 = X
E + 1 = F, F + 1 = G, G + 1 = H, H + 1 = I
Thus, OTE, PUF, QVG, RWH, SXI
Hence, A is correct.
4. Direction: Select the one which is different from the other three responses.
    A. 48,6 B. 21,7 C. 24,3 D. 56,7

Answer - Option B
Explanation :
[latex]\frac {48}{6}[/latex] = 8
[latex]\frac {21}{7}[/latex] = 3
[latex]\frac {24}{3}[/latex] = 8
[latex]\frac {56}{7}[/latex] = 8
Hence, option B is correct.
5. In a class, there are 80 students who study both Computer Science and Electronics. While 100 students study Computer Science, 120 students study Electronics. How many of them study Computer Science only?
    A. 100 B. 40 C. 180 D. 20

Answer - Option D
Explanation :
Number of students who study both Computer
Science and Electronics = 80
Number of students study Computer Science = 100
Number of students study Electronics = 120
The number of students who study Computer Science only = 100 – 80 = 20
6. Identify the missing term in the following series. ?, ONM, GFE, XWV
    A. TUV B. VUT C. GHI D. PQR

Answer - Option B
Explanation :
The letters are arranged in a manner shown below:

Thus, the missing term is VUT.
7. Direction: Select the one which is different from the other three responses.
    A. M B. D C. O D. B

Answer - Option C
Explanation :
O is a vowel of English alphabet while rest of others are consonant.
Hence, option C is correct.
8. 3 daily wage workers A, Band C are distributed Rs. 178 in such a way that A gets Rs. 4 less than C. B gets. Rs. 15 more than A and C gets Rs. 11 less than B. What is the ratio of their shares ?
    A. 57 : 53 : 68 B. 50 : 51 : 52 C. 53 : 56 : 68 D. 53 : 68 : 57

Answer - Option D
Explanation :
Suppose A gets Rs. x.
C gets Rs. x + 4
B gets Rs. x + 15
Now.
x + x + 4 + x + 15 = 178
3x = 178 = 19 = 159
x = 53
A => ? 53
B => 53 + 15 = ? 68
C => 53 + 4 = ? 57
Hence, option D is the right answer.
9. Directions: In each of the following questions select the related letters/ number/figure from the given alternative. 212 : 436 :: 560 : ?
    A. 786 B. 682 C. 784 D. 688

Answer - Option C
Explanation :
212 + 224 = 436
560 + 224 = 784
10. In the following question, select the related letters from the given alternatives. VZS : EAH : : ? : JFM
    A. QSM B. NUQ C. QUN D. QNU

Answer - Option C
Explanation :
A B C D E F G H I J K L M Z Y X W V U T S R Q P O N
The first series is the first half of the English alphabet while the other series is the other half spelled backwards.
In the question, each element in the ratio is the corresponding alphabet from the other series.
Q<=J
U<=F
N<=M
Hence, option C is the right answer.
Select the one which is different from the other three responses:
    A. Mango B. Apple C. Orange D. Guava

Answer - Option A
Explanation :
There is only one seed in mango, while others have many seeds.
2. If train is called bus, bus is called tractor, tractor is called car, car is called scooter, scooter is called bicycle, bicycle is called moped, which is used to plough a field?
    A. Train B. Bus C. Tractor D. Car

Answer - Option D
Explanation :
A tractor is used to plough a field. But a tractor is called a car.
Thus car is used to plough a field.
Hence Option D is correct
3. Direction: Which one set of letters when sequentially placed at the gaps in the given letter series shall complete it? p_ _p _ _ p q r p _ r
    A. q r s t q B. q q r r q C. q r q r q D. q q r q r

Answer - Option C
Explanation :
The complete series is : p qrp qrp q r p qr
Thus q r q r q is correct.
Hence Option C is correct
4. Select the correct response. If RAJ = 29, EDUCATION = ?
    A. 85 B. 86 C. 88 D. 92

Answer - Option D
Explanation :
RAJ = 29
EDUCATIONS = 5 + 4 + 21 + 3 + 1 + 20 + 9 + 15 + 14 = 92
5. Vitamin A : Carrot : : Vitamin C : ?
    A. Meat B. Fish C. Egg D. Orange

Answer - Option D
Explanation :
Carrot is the source of Vitamin A ,similarly Orange is the source of Vitamin C.
6. An official meeting is attended by 130 department employees. Of them, 66 drink tea, 56 drink coffee and 63 drink juice. 27 can drink either tea or coffee, 25 can drink coffee or juice and 23 can drink juice and tea. 5 employees can drink any of the three. How many drink only tea ?
    A. 21 B. 22 C. 18 D. 20

Answer - Option A
Explanation :
There are 130 employees.

21 employees drink only tea.
15 employees do not drink any beverag
7. Choose the correct figure that represents the given relation : Keyboard, function keys, letter ­ keys
    A. B. C. D.

Answer - Option B
Explanation :
Function keys are different from letter keys but both are parts of keyboard.
8. In each of the following questions, a series is given, with one term/number/letter missing. Choose the correct alternative from the given ones that will complete the series. 14, 28, 20, 40, 32, 64, ?
    A. 52 B. 56 C. 128 D. 48

Answer - Option B
Explanation :
14 × 2 = 28
28 – 8 = 20
20 × 2 = 40
40 – 8 = 32
32 × 2 = 64
64 – 8 = [56]
9. At a restaurant, five guests are sitting in a row. Neeta is to the left of Manju and to the right of Navin. Rishab is to the right of Kavita, but to the left of Navin. Where is Navin sitting?
    A. Extreme right B. Centre C. Exxtreme left D. Second from the right

Answer - Option B
Explanation :

10. Directions: In the following question, a series is given with one term miss.ing. Choose the correct alternative from the given ones that will complete the series. 12, 26,54, 110, ?
    A. 223 B. 222 C. 220 D. 225

Answer - Option B
Explanation :
1. Arrange the following according to the stages in the life cycle of a butterfly : 1. Butterfly 2. Caterpillar 3. Eggs 4. Cocoon
    A. 1, 2, 3, 4 B. 3, 2, 4, 1 C. 3, 4, 2, 1 D. 1, 4, 2, 3

Answer - Option B
Explanation :
Arrangement according to the stages in the life cycle of a butterfly :
3. Eggs
[latex]\downarrow[/latex]
2. Caterpillar
[latex]\downarrow[/latex]
4. Cocoon
[latex]\downarrow[/latex]
1. Butterfly
2.Direction: Select the related word/letters/ number from the given alternatives. 64 :100 :: 16 :?
    A. 81 B. 36 C. 50 D. 55

Answer - Option B
Explanation :
64 = [latex] {(8)}^{2}[/latex]
100 = [latex] {((8 + 2))}^{2}[/latex] = [latex] {(10)}^{2}[/latex]
16 = [latex] {(4)}^{2}[/latex]
36 = [latex] {((4 + 2))}^{2}[/latex] = [latex] {(6)}^{2}[/latex]
3. Direction: In a zoo, there are Rabbits and Pigeons. If heads are counted, there are 200 and if legs are counted, there are 580. How many pigeons are there?
    A. 90 B. 100 C. 110 D. 120

Answer - Option C
Explanation :
R + P= 200...(i)
4R + 2P = 580...(ii)
By solving equations i and ii
4R + 2 (200 - R) = 580
4R + 400 - 2R = 580
2R = 180
R = 90
Since R + P = 200
P = 110
4. Find the odd one out
    A. Mumbai B. Chennai C. Kolkatta D. Bangalore

Answer - Option D
5. For the following questions Find the odd word/letters/number pair from the given alternatives
    A. 49 B. 140 C. 112 D. 97

Answer - Option A
6. Identify the diagram that best represents the relationship among the given classes. Fishes, Snakes and Birds
    A. B. C. D.

Answer - Option D
Explanation :
Fishes live in water. Birds fly in the sky and Snakes in ground. Therefore birds cannot be either snake or Fish. Similarly Fish cannot be either a snake or bird. Hence a snake cannot be either bird or fish.
Therefore the relationship among Fishes, Snakes and Birds can be described as:

7. If FIREWOOD is written as ERIFDOOW, how is FRACTION written as?
    A. ARFTCNO B. NOITCARF C. CARFNOIT D. CRAFNOIT

Answer - Option C
Explanation :
FIREWOOD is written as ERIFDOOW

Using the same rule:

Hence FRACTION can be written as CARFNOIT.
8. In the following question, choose the correct alternative from the given ones that will complete the series. 2, 7, 17, 32, 52, 77, ?
    A. 107 B. 91 C. 101 D. 92

Answer - Option A
Explanation :
2 [latex]\overrightarrow{+ 5}[/latex] 7 [latex]\overrightarrow{+ 10}[/latex] 17 [latex]\overrightarrow{+ 15}[/latex] 32 [latex]\overrightarrow{+ 20}[/latex] 52 [latex]\overrightarrow{+ 25}[/latex] 77 [latex]\overrightarrow{+ 30}[/latex] 107
9. Directions: In each of the following questions, choose the correct alternative from the given ones that will complete the series. 30, 24, 19, 15, 12,?
    A. 6 B. 8 C. 10 D. 11

Answer - Option C
Explanation :
30 [latex]\overrightarrow{- 6}[/latex] 24 [latex]\overrightarrow{- 5}[/latex] 19 [latex]\overrightarrow{- 4}[/latex] 15 [latex]\overrightarrow{- 3}[/latex] 12 [latex]\overrightarrow{- 2}[/latex] 10
10. Sita cycled 8 km southward from her home turned right and cycled 5 km. turned right and cycled 8 km, turned left and cycled 10 km. How many kms will she have cycle to reach straight home?
    A. 8 km B. 10 km C. 15 km D. 13 km

Answer - Option C
1. Direction: Select the related word/letters/ number from the given alternatives. 49 : 216 : : 36 : ?
    A. 25 B. 125 C. 625 D. 3125

Answer - Option B
Explanation :
[latex]\sqrt {49} = 7 \Longrightarrow 7 - 1 = 6 \Longrightarrow {6}^{3} = 216[/latex]
[latex]\sqrt {36} = 6 \Longrightarrow 6 - 1 = 5 \Longrightarrow {5}^{3} = 125[/latex]
49 : 216 : : 36 : 125
Hence, option B is correct.
2. Direction: Select the one which is different from the other three responses.
    A. B E H L N B. E J L S Z C. D K Q V Z D. A C I K T

Answer - Option C
Explanation :
Except 'D K Q V Z' all the other options have vowels.
Hence option C is the right choice.
3. Direction: Select the missing numbers from the given alternatives. 63 7 9 30 5 6 20 4 ?
    A. 5 B. 2 C. 8 D. 3

Answer - Option A
Explanation :
First Row = 7 × 9 = 63
Second Row = 5 × 6 = 30
Therefore = [latex]\frac {20}{4}[/latex] = 5
Hence Option A is correct.
4. Direction: Select the related word/letters/ number from the given alternatives. Picture : See ::Book : ?
    A. Library B. Buy C. Read D. Listen

Answer - Option C
Explanation :
We see picture. Similarly, we read books.
Hence option C is the right answer.
5. 6, 12, 20, 30, ?
    A. 40 B. 41 C. 42 D. 48

Answer - Option C
Explanation :
6 + 6 = 12
12 + 8 = 20
20 + 10 = 30
30 + 12 = 42
6. Direction: Find the missing number in the following series: 1000, 200, 40, ?
    A. 10 B. 20 C. 15 D. 8

Answer - Option D
Explanation :
[latex]\frac {1000}{5}[/latex] = 200
[latex]\frac {200}{5}[/latex] = 40
[latex]\frac {40}{5}[/latex] = 8
Thus 8 is the required number.
Hence Option D is correct
7. From the given alternatives, select the word which cannot be for Moderate using the letters of the given word. ESTABLISHMENT
    A. TABLE B. BLUNT C. TENTS D. STATE

Answer - Option B
Explanation :
In ESTABLISHMENT there is No U word, so BLUNT can’t be for Moderate.
8. Direction: Which one set of letters when sequentially placed at the gaps in the given letter series shall complete it? N, P, R, ?
    A. T B. U C. O D. V

Answer - Option A
Explanation :
N [latex]\overrightarrow{+ 2}[/latex] P [latex]\overrightarrow{+ 2}[/latex] R [latex]\overrightarrow{+ 2}[/latex] T
Hence A is the right answer.
9. In the given diagram, the rectangle represents males, the circle represents actors and the triangle singers. Which numbered area represents male singers only?
    A. 8 B. 6 C. 5 D. 3

Answer - Option D
Explanation :
The number '3' is common to rectangle and triangle only
10. X Is poorer than W, but not as poor as B. C is not as poor as X. Who is the poorest of all?
    A. B B. X C. C D. W

Answer- Option A
1. Directions: In each of the following questions, select the related word/letters/number from the given alternatives. xxyzyy : ccbabb : : xyyzzy :
    A. abbacc B. bbacca C. bbacca D. cbbab

Answer - Option D
Explanation :
x x y z y y
[latex]\downarrow[/latex] [latex] \downarrow[/latex] [latex]\downarrow[/latex] [latex]\downarrow[/latex] [latex]\downarrow [/latex][latex]\downarrow[/latex]
c c b a b b
x y y z z y
[latex]\downarrow[/latex] [latex] \downarrow[/latex] [latex]\downarrow[/latex] [latex]\downarrow[/latex] [latex]\downarrow [/latex][latex]\downarrow[/latex]
c b b a a b
2. In the following question, select the related word from the given alternatives. Cobbler : Leather : : Carpenter : ?
    A. Furniture B. Wood C. Hammer D. Chair

Answer - Option B
Explanation :
As, cobbler does the leather work, similarly carpenter does wood work.
Hence, Cobbler : Leather : : Carpenter : Wood.
3. Direction: In the following questions select the related words/letters/number from the given alternatives. Chair : Wood : : ?
    A. Book : Print B. Mirror : Glass C. Plate : Food D. Purse : Money

Answer - Option B
Explanation :
As chair is made from wood, similarly mirror is made from glass.
Chair : Wood : : Mirror :Glass
4. Six friends A, B. C. D. E and F are sitting in a row facing East. C is between A and E. B is just to the right of E but left of D. F is not at the right end. Who is at the right end?
    A. D B. B C. E D. C

Answer - Option A
5. Direction: F has less money than H but more than G. E has more than F but less than H. Who is the poorest?
    A. F B. E C. H D. G

Answer - Option D
Explanation :
H > F > G …………… (i) H > E > F …………… (ii)
From both the statements
H > E > F > G
Hence, “G” is the poorest among all.
6. In each of the following question, a series is given, with one term / number / letters missing. Choose the correct alternative from the given ones that will complete the series. SCD, TEF, UGH, ? , WKL
    A. VIJ B. VJI C. CMN D. UJI

Answer - Option A
Explanation :
S [latex] \underrightarrow {+ 1}[/latex] T [latex]\underrightarrow {+ 1}[/latex] U [latex]\underrightarrow {+ 1} [/latex] V [latex]\underrightarrow {+ 1} [/latex] W
C [latex] \underrightarrow {+ 1}[/latex] E [latex]\underrightarrow {+ 1}[/latex] G [latex]\underrightarrow {+ 1} [/latex] I [latex]\underrightarrow {+ 1} [/latex] K
D [latex] \underrightarrow {+ 1}[/latex] F [latex]\underrightarrow {+ 1}[/latex] H [latex]\underrightarrow {+ 1} [/latex] J [latex]\underrightarrow {+ 1} [/latex] L
7. Sheela is Ravi’s sister-in-law. Ram is Ravi’s brother. Ram’s wife is Sheela. Deepa is Ravi’s sister. Deepa’s mother is Shanti. How is Sheela related to Shanti?
    A. Mother-in-law B. Daughter-in-law C. Granddaughter D. Daughter

Answer - Option B
Explanation :

Clearly, Sheela is the Daughter-in-law of Shanti.
8. Direction: Select the missing numbers from the given alternatives.
    A. 12 B. 39 C. 3 D. 80

Answer - Option D
Explanation :
[latex] {1}^{2}[/latex] + [latex] {5}^{2}[/latex] = 1 + 25 = 26
[latex] {3}^{2}[/latex] + [latex] {2}^{2}[/latex] = 9 + 4 = 13
[latex] {4}^{2}[/latex] + [latex] {8}^{2}[/latex] = 16 + 64 = [80]
Hence option D is the right answer.
9. ABCD, IJKL, QRST, ?
    A. YZAB B. ABYZ C. BAZY D. YAZB

Answer - Option A
Explanation :
A [latex] \underrightarrow {+ 1}[/latex] I [latex]\underrightarrow {+ 1}[/latex] Q [latex]\underrightarrow {+ 1} [/latex] Y
B [latex] \underrightarrow {+ 1}[/latex] J [latex]\underrightarrow {+ 1}[/latex] R [latex]\underrightarrow {+ 1} [/latex] Z
C [latex] \underrightarrow {+ 1}[/latex] K [latex]\underrightarrow {+ 1}[/latex] S [latex]\underrightarrow {+ 1} [/latex] A
D [latex] \underrightarrow {+ 1}[/latex] L [latex]\underrightarrow {+ 1}[/latex] T [latex]\underrightarrow {+ 1} [/latex] B
10. Direction:Which one of the following diagrams best depicts the relationship among Pigeons, Birds and Dogs.
    A. B. C. D.

Answer - Option A
Explanation :
Pigeons come under the class Birds. Dog is a mammal.
1. In each of the following questions select the related word/letters/number from the given alternatives AB : L : : BC : ?
    A. X B. V C. Y D. W

Answer - Option D
Explanation :
A is [latex] {1}^{st}[/latex] alphabet and B is [latex] {2}^{nd}[/latex] alphabet of English. L is [latex] {12}^{th}[/latex] alphabet of English. Hence AB: L ? 1 2 : 12
Now B is [latex] {2}^{nd}[/latex] and C is [latex] {3}^{rd}[/latex] alphabet of English; now [latex] {23}^{rd}[/latex] alphabet of English is W. 2 3 :23
Therefore BC : W
2. Directions: In each of the following questions, a series is given with one term missing. Choose the correct alternative from the given ones that will complete the series. ACDF : MOPR :: TVWY : ?
    A. LNPR B. PRSU C. STVW D. RUWZ

Answer = Option B
3. Direction: In each of the following questions, select the related letters/word/number from the given alternatives. LKJ : MNO : : DCB : ?
    A. FEG B. EGF C. FGH D. EFG

Answer - Option D
4. In each of the following questions select the related word/letters/number from the given alternatives 25 : 81 : : 49 : ?
    A. 121 B. 93 C. 65 D. 193

Answer - Option A
Explanation :
25 : 81
25 = [latex] {5}^{2}[/latex];
81 = [latex] {9}^{2}[/latex] = [latex] {(5+4)}^{2}[/latex]
Therefore 49 : ?
49 = [latex] {7}^{2}[/latex]
[latex] {(7+4)}^{2}[/latex](7+4)2
? [latex] {11}^{2}[/latex] = 121
49 : 121
5. Direction: Select the related word/letters/ number from the given alternatives. 292 : 146 : : 582 : ?
    A. 272 B. 286 C. 291 D. 292

Answer - Option C
Explanation :
First number is double of second.
292 = 146 × 2
Similarly 582 = ? × 2 ? ? = [latex]\frac {582}{2}[/latex] = 291
Hence 582 : 291
6. In the following question, Select the related word/letters/number from the given alternatives. Antonym : Synonym : : Complicate : ?
    A. Synthesis B. Synchronize C. Simplify D. Summarise

Answer - Option C
Explanation :
As ‘Antonym’ and ‘Synonym’ are opposite of each other, similarly the opposite of ‘complicate’ is ‘Simplify’.
7. In the following question, select the related word from the given alternatives. Democracy : India : : Communism : ?
    A. France B. China C. Britain D. America

Answer - Option B
Explanation :
As 'India' follows 'Democracy’ likeise, 'Communism' is followed by 'China'.
Hence, the correct option is B.
8. In the following question, select the related letters from the given alternatives. AGREE : FGPIB : : RULES : ?
    A. TJGSU B. TGJWS C. TGJSU D. THJUS

Answer - Option B
Explanation :
9. Direction: Select the related word/letters/ number from the given alternatives. ELIMS : SMILE : : KRAPS : ?
    A. KPAPS B. SPARK C. PARKS D. KARPS

Answer - Option B
10. In the following question, Select the related word/letters/number from the given alternatives. 167 : 14 :: 143:?
    A. 7 B. 8 C. 6 D. 5

Answer - Option B
Explanation :
167 = 1 = 6 = 7 = 14
143 [latex]\leftarrow[/latex]1 + 4 + 3 = 8
1. In the following question, Select the related word/letters/number from the given alternatives. 24 : 60 : : 210 : ?
    A. 505 B. 425 C. 420 D. 525

Answer - Option D
Explanation :
[latex]\frac {24}{4}[/latex] × 10 = 60
Similarly;
[latex]\frac {210}{4}[/latex] × 10 = 525
Hence Option D is correct
2. Direction: Select the related word/letters/ number from the given alternatives. Large : Enormous : : ?
    A. Cat : Tiger B. Warmth : Frost C. Plump :Fat D. Royal : Regale

Answer - Option C
Explanation :
‘Large’ and ‘Enormous’ are similar in meaning. ‘Plump’ and ‘Fat’ are similar in meaning.
3. ACE : KIG : : MOQ : ?
    A. WUS B. WVU C. WVT D. WUT

Answer - Option A
Explanation :
[latex]A \overrightarrow{+ 2} C \overrightarrow{+ 2} E[/latex]
[latex]K \overrightarrow{+ 2} I \overrightarrow{+ 2} G[/latex]
Similarly,
[latex]M \overrightarrow{+ 2} O \overrightarrow{+ 2} Q[/latex]
[latex]W \overrightarrow{+ 2} U \overrightarrow{+ 2} S[/latex]
4. 3 : 27 : : 7 : ?
    A. 21 B. 42 C. 147 D. 343

Answer - Option D
Explanation :
3 * 3 * 3 = 27
Similarly, 7 * 7 * 7 = 343
5. In the following question, select the related word/letters from the given alternatives. Saw: Carpenter :: Scissors : ?
    A. Farmer B. Tailor C. Painter D. Artist

Answer - Option B
Explanation :
As SAW is an instrument of carpenter, Similarly SCISSORS is an instrument of tailor
6. Direction: Select the related word/letters/ number from the given alternatives. 8 : 28 :: 27 : ?
    A. 8 B. 28 C. 64 D. 65

Answer - Option D
Explanation :
[latex]{2}^{3} = 8[/latex]
[latex]28 = {3}^{3} + 1 = 27 +1[/latex]
[latex]{3}^{3} = 2[/latex]
[latex]? = {4}^{3} + 1= 65[/latex]
Hence Option D is correct
7. Directions: In each of the following questions, select the related word/letters/number from the given alternatives. AZBZ: CYDY :: EXFX : ?
    A. FWFV B. GWHW C. IVJW D. HWIW

Answer - Option B
Explanation :
A B C D E F G H I J K L M
Z Y X W V U T S R Q P O N
Series are in the reverse order
8. Select the related word/letter/number from the given alternatives. Telephone : Mobilephone :: Computer : ?
    A. Keyboard B. Tlevision C. Printer D. Laptop

Answer - Option D
Explanation :
Telephone Updating is mobile phone, similarly computer updating is laptop.
9. Directions: In each of the following questions select the related word/letters/numbers from the given alternatives MOQS :ACEG : : PRTV : ?
    A. OQSU B. EGIJ C. FHJM D. KMNP

Answer - Option A
Explanation :
M [latex] \overrightarrow{+ 2}[/latex] O [latex]\overrightarrow{+ 2}[/latex] Q [latex]\overrightarrow{+ 2} [/latex] s
[latex]A \overrightarrow{+ 2} C \overrightarrow{+ 2} E \overrightarrow{+ 2} G [/latex]
[latex]P \overrightarrow{+ 2} R \overrightarrow{+ 2} T \overrightarrow{+ 2} V [/latex]
Similarly,
[latex]O \overrightarrow{+ 2} Q \overrightarrow{+ 2} S \overrightarrow{+ 2} U[/latex]
10.In each of the following questions, select the related word/letters/ number from the given alternatives.
    A. RTQS B. TRQS C. RQTS D. QTRS

Answer - Option A
Explanation :
[latex]B \overrightarrow{+ 4} F[/latex]
[latex]D \overrightarrow{+ 4} H[/latex]
[latex]A \overrightarrow{+ 4} E[/latex]
[latex]C \overrightarrow{+ 4} G[/latex]
Similarly
[latex]N \overrightarrow{+ 4} R[/latex]
[latex]P \overrightarrow{+ 4} T[/latex]
[latex]M \overrightarrow{+ 4} Q[/latex]
[latex]O \overrightarrow{+ 4} S[/latex]
1. Select the related word/letters/number from the given alternatives. RESISTANCE : OHM : : CURRENT : ?
    A. Faraday B. Radian C. Ampere D. Volt

Answer - Option C
Explanation :
As ‘Resistance’ is measured in ‘Ohm’ likewise, ‘Current’ is measured in ‘Ampere’.
Hence, the correct option is C.
2. In the following question, select the related letters from the given alternatives. ACFJ : ZXUQ :: DFIM : ?
    A. WURN B. WURP C. WUQO D. WTQP

Answer - Option A
Explanation :
ACFJ : ZXUQ
A( + 2) = C( + 3) = F( + 4) = J : Z( - 2) = X( - 3) = U( - 4) = Q
A is 1st from starting and Z is 1st from last alphabet.
Similarly
DFIM : ?
As D is 4th from starting, therefore W is 4th from the last. Hence ? will starts from W.
D( + 2) = F( + 3) = I( + 4) = M : W( - 2) = U( - 3) = R( - 4) = N
? = WURN
Hence, option A is the correct response.
3. Direction: Select the related word/letters/ number from the given alternatives.
    A. Enjoyment B. Learning C. Injury D. Champion

Answer - Option A
Explanation :
While sleeping people take rest, Similarly people do Enjoyment while Playing, So that Enjoyment Should be placed here.
4. Directions: In each of the following questions, select the related word/letters/ number from the given alternatives. ABZY : CDXW : : EFVU : ?
    A. IJRQ B. KLPO C. MNST D. GHTS

Answer - Option D
5. Directions: In each of the following questions, select the related word/letters/ number from the given alternatives. 4, 16, 4 : 2, 4, 2 : : ? : 3, 9, 3
    A. 6, 9, 6 B. 9, 81, 9 C. 9, 27, 9 D. 9, 18, 9

Answer - Option B
Explanation :
The product of the first and the last numbers is equal to the middle numbers in each term.
4 * 4 = 16; 2 * 2 = 4; 3 * 3 = 9
Therefore,
? = 9 * 9 = 81
Hence, option B is correct.
6. In each of the following questions, select the related word/number from the given alternatives. Gramophone : Sound : : Camera: ___?____
    A. T V B. Recording C. Cassette D. Videos

Answer - Option D
Explanation :
Gramophone is used for recording sound. Similarly camera is used to record videos.
Hence option D is correct.
7. Directions: In the following questions some equations are solved on the basis of a certain system. Find the correct answer for the unsolved equation on that basis. 6 × 4 × 5 = 456 3 × 2 × 8 = 283 4 × 9 × 3 = ?
    A. 349 B. 934 C. 394 D. 493

Answer - Option B
Explanation :

Therefore

8. In the following question, select the related word from the given alternatives. 16 : 40 : : 20 : ?
    A. 29 B. 21 C. 50 D. 60

Answer - Option C
Explanation :
16 * 2.5 = 40
So, 20 * 2.5 = 50
Thus, 20 is related to 50
9. Direction: Select the related word/letters/ number from the given alternatives. ET : VG : : NO : ?
    A. IM B. MI C. ML D. LM

Answer - Option C
Explanation :
If we arrange alphabets in two rows as shown below, the letters are related as follows,

Thus NO is related to ML.
10. Select the related word/letters/number from the given alternatives. JK : PQ : : FG : ?
    A. HI B. TU C. EF D. UT

Answer - Option B
Explanation :
If we arrange alphabets in two rows as shown below, the letters are related as follows,

Thus FG is related to TU.
1. In the following question, which one set of letters when sequently placed at the gaps in the given letter series shall complete it ? A_C_E_B_DE
    A. CADB B. DBAC C. BDAC D. BADC

Answer - Option C
Explanation :
If we divide the sequence in two parts of 5 alphabets each, we will observe that they appear to be same:
1. A_C_E
2. _B_DE
Therefore, 2nd alphabet of 1st part is ‘B’ and 4th is ‘D’ with ‘A’ being 1st alphabet in 2nd part along with ‘C’ as 3rd alphabet.
Hence, the correct option is C.
2. How many M’s, such that the M is preceded by W and followed by V, occur in the following series? X U V M R S T M W N V M W O P M W U V M W A C W M V H P N V W M W T U N
    A. 3 B. 2 C. 1 D. 5

Answer - Option C
Explanation :
In the given series, the highlighted M’s are those which are preceded by W and followed by V
X U V M R S T M W N V M W O P M W U V M W A C W M V H P N V W M W T U N
Clearly, there is only 1 M.
Hence, option C is the correct response.
3. Direction: Unscramble the following letters to frame a meaningful word and find out the correct numerical sequence of the letters. E S R T A R U N A T 1 2 3 4 5 6 7 8 9 10
    A. 10 2 3 5 16 4 7 8 9 B. 3 1 2 4 5 7 6 9 8 10 C. 1 3 5 2 9 4 8 6 7 10 D. 9 1 3 6 2 7 54 8 10

Answer - Option B
Explanation :
Let we form a meaningful word from given alphabets:
R E S T A U R A N T
3 1 2 4 5 6 9 8 10
Hence correct option is (3 1 2 4 5 7 6 9 8 10)
4. A group of alphabets are given and each of them has assigned to a number. The letters have to be unscrambled into a meaningful word and correct order of letters may be indicated from the given responses. R T E F A 1 2 3 4 5
    A. 2, 5, 1,3, 4 B. 4, 3, 5, 1, 2 C. 5, 4, 1, 3, 2 D. 5, 4, 2, 3, 1

Answer - Option D
Explanation :
The correct meaningful word formed will be AFTER and the sequence of numbers will be 5,4,2,3,1
Hence, option D is correct.
5. Which one set of letters when sequentially placed at the gaps in the given letter series shall complete the series. A _ C _ _ A A C _ B _ A A C _ B C _
    A. AACBCAB B. CCBCACB C. BCCABCB D. CBCCCCA

Answer - Option D
Explanation :
The original series is as follows:
A _ C _ _ A A C _ B _ A A C _ B C _
On observing the series we are unable to infer any pattern. In such questions, it’s more prudent to take help of the options. On checking each option, we find that a pattern indeed emerges when the letters given under option (D) are used to complete the series. The completed series has been represented below:
A CCBC A/ A C C B C A / A C C B C A
We can see that the pattern ‘ACCBCA’ is being repeated in the series given above.
Hence, option (D) is the correct answer.
6. Which one set of letters when sequentially placed at the gaps in the given letter series shall complete the series. _Q _ R P _ Q _ _ _ Q R
    A. PQQRPQ B. QPRPPQ C. RQPQPQ D. PRQQPQ

Answer - Option A
Explanation :
The original series is as follows:
_ Q _ R P _ Q _ _ _ Q R
On observing the series we are unable to infer any pattern. In such questions, it’s more prudent to take help of the options. On checking each option, we find that a pattern indeed emerges when the letters given under option (A) are used to complete the series. The completed series has been represented below:
P Q Q R /P QQR /PQQ R
We can see that the pattern ‘PQQR’ is being repeated in the series given above.
Hence A is the correct answer.
7. If it is possible to make only one meaningful word with the second, third, sixth and eighth letters of the word BASEMENT, which of the following will be the third letter of that word. If no such word can be made, give 'X’ as your answer and if more than one such word can be formed, give ‘Y’ as the answer
    A. X B. Y C. T D. N

Answer - Option B
Explanation :
In the word BASEMENT, second, third, sixth and eighth letters are A, S, E and T.
From these letters, meaningful words will be SEAT and EAST.
Hence Y will be the answer since more than one such word can be formed.
8. How many such pairs of letters are there in the word SEQUENTIAL each of which has as many letters between them in the word as in the English alphabet?
    A. None B. One C. Two D. More than three

Answer - Option D
Explanation :

Hence S-Q, Q-N, E-A and S-N are possible pairs as per the given condition.
9. Directions: In each of the following questions, select the missing number / letters from the given responses.
    A. HR B. HS C. HV D. HU

Answer - Option B
Explanation : In each sector pair of opposite letters given. [latex]G \underrightarrow{+ 1} H[/latex] The opposite letter of H is S.
10. How many meaningful English words can be formed with the letters LTSO using each Letter only once in each word’
    A. None B. One C. Two D. Three

Answer - Option D
Explanation :
Lost , Slot, Lots
1. ‘DEAN’ is related to ‘NDAE’ and ‘ROAD’ is related to ‘DRAO’ in the same way as ‘SOME’ is related to
    A. ESMO B. EOMS C. EMOS D. MSEO

Answer - Option A
2. How many such pairs of letters are there in the word HOARDINGS each of which has as many letters between them in the word as in the English alphabet?
    A. None B. One C. Two D. More than three

Answer - Option D
Explanation :
Hence there are 4 pairs HN, DG, GI and DH.
3. A group of alphabets are given with each being assigned a number. These have to be unscrambled into a meaningful word and correct order of letters may be indicated from the given responses. L, R, T, O, A, I 1, 2, 3, 4, 5, 6
    A. 3, 4, 1, 6, 2, 5 B. 3, 5, 6, 1, 4, 2 C. 5, 3, 6, 2, 4, 1 D. 6, 2, 5, 3, 4, 1

Answer - Option B
Explanation :
From the given alphabets (L R T O A I); the word that can be formed is:
T, A, I, L, O, R
3, 5, 6, 1, 4, 2
4. How many meaningful English words can be fanned with the letters ‘OLBT’ using all the letters but each letter only once in each word?
    A. 1 B. 2 C. 3 D. 4

Answer - Option B
Explanation :
There are two words are Bolt and Blot.
5. Each consonant in the word BISCUIT is replaced by the next letter in the English alphabet and each vowel is replaced by the previous letter in the English alphabet and the letters so obtained are rearranged in alphabetical order, which of the following will be the third from the left end after the rearrangement?
    A. C B. D C. H D. T

Answer - Option C
Explanation :
B I S C U I T
According to question,
C H T D T H U
According to alphabet,

So, H is third from left.
6. How many such pairs of letters are there in the word TACKLE, each of which has as many letters between them in the word (in both forward and backward directions) as they have between them in the English alphabetical series?
    A. None B. One C. Two D. Three

Answer - Option C
Explanation :

There are two such pairs AE, KL.
Hence, option C is the right answer.
7. Direction: Following questions are based on the five words given below: LAP BUT CAR SON HID (The new words formed after performing the mentioned operations may or may not be meaningful English word) If the second alphabet in each of the word is changed to the next alphabet in the English alphabetical order, how many words have no vowels will be formed?
    A. None B. One C. Two D. More than three

Answer - Option D
Explanation :
Given Arrangement: LAP BUT CAR SON HID
If the second alphabet in each of the word is changed to the next alphabet in the English alphabetical order
Modified arrangement: LBP BVT CBR SPN HJD
All the words have no vowels.
8. Each letter of the alphabet from Z to A has been given a value from 1 to 26 serially. What is the total value of the word CONSEQUENCE?
    A. 137 B. 154 C. 176 D. 196

Answer - Option C
Explanation :
Clearly, the letters have been assigned numerical value as shown below:
Z Y X W V U T S R Q P O N M L K J I H G F E D C B A
So, CONSEQUENCE = C + O + N + S + E + Q + U + E + N + C + E
= 24 + 12 + 13 + 8 + 22 + 10 + 6 + 22 + 13 + 24 + 22 = 176
9. How many meaningful words can be formed with the letters S L I K L using each letter only once?
    A. One B. Two C. Three D. More than Three

Answer - Option B
Explanation :
SKILL and KILLS are the two words which can be formed by rearranging the given letters.
Hence, option B is correct.
Direction: The question is based on five words given below. AND FOR THE BIG SUM
10. If the given words are arranged in the order as they would appear in a dictionary from left to right, which of the following will be second from the right?
    A. AND B. FOR C. THE D. SUM

Answer - Option D
Explanation :
The arrangement of the words will be
AND, BIG , FOR SUM, THE
Hence second word from the right will be SUM
1. Directions: In each of the following questions which one of the given responses would be a meaningful order of the following? 1. Metal 2. Alloy 3. Ore 4. Purification 5. Spare parts
    A. 3, 1, 2, 4, 5 B. 3, 4, 1, 2, 5 C. 3, 4, 1, 2,5 D. 3, 5, 1, 2, 4

Answer - Option C
Explanation : Ore is purified to extract metal. Different metals are mixed to form alloy and in turn, alloy is used to manufacture spare pars.
Thus,
Meaningful order of words : [latex]\downarrow[/latex] 3. Ore [latex]\downarrow[/latex] 4. Purification [latex]\downarrow[/latex] 1. Metal [latex]\downarrow[/latex] 2. Alloy [latex]\downarrow[/latex] 5. Spare parts
2. Direction: Which one set of letters when sequentially placed at the gaps in the given letter series shall complete it? d_fde_ _deee_
    A. eeff B. cdef C. ddee D. deee

Answer - Option A
Explanation :
From option A, the series becomes,
def / deef / deeef
Hence, 'eeff' is the correct set of letters.
3. A series is given with one term missing. Select the correct alternative from the given ones that will complete the series. 12, 60, 240, 720, ?
    A. 1440 B. 1420 C. 1400 D. 1480

Answer - Option A
Explanation :
12 × 5 = 60,
60 × 4 = 240,
240 × 3 = 720,
720 × 2 = 1440
Thus, next number in the series is 1440.
Hence, option A is the right answer.
4. If "A" denotes "added to", "B" denotes "divided by", "C" denotes "multiplied by" and "D" denotes "subtracted from", then 116 B 29 C 6 A 24 D 45 = ?
    A. 4 B. 3 C. 5 D. 6

Answer - Option B
Explanation :
Interchanging alphabets with proper operators we get,
116 ÷ 29 × 6 + 24 – 45
Applying BODMAS,
116 ÷ 29 × 6 + 24 – 45
= 4 × 6 + 24 – 45
= 24 + 24 – 45
= 48 – 45
= 3
5. Select the related word/letters/number from the given alternatives. 2 : 16 : : 3 : ?
    A. 340 B. 81 C. 243 D. 122

Answer - Option B
Explanation :
[latex]{2}^{4}[/latex] = 16
Similarly,
[latex]{3}^{4}[/latex] = 81
Thus 3 is related to 81.
6. A series is given with one term missing. Select the correct alternative from the given ones that will complete the series. BC, GHI, NOPQ, ?
    A. BCDEF B. UVWXY C. WXYZA D. STUVW

Answer - Option C
Explanation :

Thus the next term in the series is WXYZA.
7. Find out the odd word/letters/number/number pair from the given alternatives.
    A. Tiger B. Jaguar C. Leopard D. Fox

Answer - Option D
Explanation :
Tiger, Jaguar, Leopard belongs to the family of Cats (Felidae) whereas Fox belongs to the family of Dogs (Canidae).
8. Select the related word/letters/number from the given alternatives. BCD : YXW : : FGH : : ?
    A. UTS B. RQP C. STU D. TUS

Answer - Option A
Explanation :
If we arrange alphabets in two rows as shown below, the letters are related as follows,

Thus FGH is related to UTS.
9. A series is given with one term missing. Choose the correct alternative from the given ones that will complete the series. ?, Oxygen, Argon, Carbon dioxide
    A. Nitrogen B. Water vapour C. Methane D. Carbon monooxide

Answer - Option A
Explanation :
By volume dry air contains 78.09% Nitrogen, 20.95% Oxygen, 0.93% Argon, 0.04% carbon dioxide and then small amount of other gases.
So the required gas here is Nitrogen.
10. A series is given with one term missing. Choose the correct alternative from the given ones that will complete the series. ABC, BDF, DHL, ?
    A. RST B. HPX C. CDE D.

Answer - Option B
Explanation :
A + 0 = B + 0 = C
B + 1 = D + 1 = F
D + 3 = H + 3 = L
similarly, if we continue, we know that H would be our next letter, When we look at the options present, we can see only one option starts with H and hence it will be our answer. This method is called option elimination and is very useful in saving time and getting the correct when “none of these” option is not present.
Hence B
1. A series is given with one term missing. Choose the correct alternative from the given ones that will complete the series. Dadabhai Naoroji, Bal Gangadhar Tilak, Lala Lajpat Rai, ?
    A. Mahatma Gandhi B. Jawaharlal Nehru C. Subhash Chandra Bose D. Bhagat Singh

Answer - Option A
Explanation :
The names are all of freedom fighters of India. Out of the options given, Mahatma Gandhi was the only freedom fighter, all others were revolutionaries.
Hence A
2. A series is given with one term missing. Choose the correct alternative from the given ones that will complete the series. Mesopotamian civilization, Egyptian civilization, ? , Chinese civilization
    A. Greek Civilization B. Roman Civilization C. Persian Civilization D. Indus valley Civilization

Answer - Option D
Explanation :
This is an ordinal series of civilization which existed in the history:
Mesopotamian civilization, Period: 3500 BC -500 BC
Egyptian civilization, Period: 3100 BC-2686 BC
Indus Valley Civilization, Period: 2600 BC -1900 BC
Chinese Civilization, Period: 1600 BCE-1046 BCE
Hence, the correct option is D.
3. In the following question, select the odd number from the given alternatives.
    A. 3284 B. 4056 C. 2137 D. 2363

Answer - Option D
Explanation :
If we sum all the digits of the numbers given, we will observe:
3 + 2 + 8 + 4 = 17
4 + 0 + 5 + 6 = 15
2 + 1 + 3 + 7 = 13
2 + 3 + 6 + 3 = 14
Therefore, only in ‘option D’ we are getting sum as an even number.
Hence, the correct option is D.
4. A series is given with one term missing. Choose the correct alternative from the given ones that will complete the series. Qutb ­ud ­din Aibak, Iltutmish, ?, Ghiyas ud din Balban
    A. Razia Sultana B. Ala ud din Khilji C. Froz Shah Tuhlaq D. Bahlol Lodi

Answer - Option A
Explanation :
This is an ordinal series of Mamluk or Slave dynasty:

Hence, the correct option is A.
5. A series is given with one term missing. Choose the correct alternative from the given ones that will complete the series. Ground water, Evaporation, Condensation, Cloud formation, ?
    A. Precipitation B. Sublimation C. Decantation D. Ice

Answer - Option A
Explanation :
This is an ordinal series of ‘Water Cycle’ where,
Ground water ? Evaporation ? Condensation ? Cloud formation ? Precipitation.
Hence, the correct option is A.
6. A series is given with one term missing. Choose the correct alternative from the given ones that will complete the series. ? , Planning, Strategy, Marketing, Finance
    A. Realization B. Success C. Failure D. Idea

Answer - Option D
Explanation :
This is an ordinal series of any business plan where Idea comes first followed by:
Planning ? Strategy ? Marketing ? Finance
Hence, the correct option is D.
7. In the following question, which one set of letters when sequentially placed at the gaps in the given letter series shall complete it? p_rrq_p_rr_p
    A. qqqp B. qpqq C. pqqr D. rppq

Answer - Option B
Explanation :
The long sequence we can observe in the above question is ‘rrq’. Based on this let’s fill up the blanks.
p_rrq_p_rrqp
The next long sequence is: ‘rrqp’
p_rrqpp_rrqp
Out of all options present, we now can cancel out option A and C.
So let’s try both options B and D.
Prrrqppprrqp – This isn’t making any sequence. So cancel out option D
Hence B
Pqr/rqp/pqr/rqp
Hence, option B is the right answer.
8. A series is given with one term missing. Choose the correct alternative from the given ones that will complete the series. Arctic Ocean, Southern Ocean (Antarctic ocean), Indian Ocean, ?
    A. Atlantic Ocean B. Pacific Ocean C. South Pacific Ocean D. North Pacific Ocean

Answer - Option A
Explanation :
The series is clearly the series of the world’s 5 oceans. The next ocean which belongs to this series is: Atlantic ocean Hence A
9. Direction: In each of the following equations, select the missing number from the given responses.

    A. 22 B. 24 C. 25 D. 28

Answer - Option B
Explanation :
7 + 4 = 11, 7 * 4 = 28
8 + 3 = 11, 8 * 3 = 24
10. Find the odd word/number from the given alternatives.
    A. QWBS B. MPTD C. UIAE D. RVGW

Answer - Option C
Explanation :
Option C is vowel remaining all is consonants.
1. For the following questions Find the odd word/letters/number pair from the given alternatives
    A. 49 B. 140 C. 112 D. 97

Answer - Option A
Explanation :
The number 49 is a perfect square (7 × 7 = 49) while other given numbers are not.
2. Direction: Select the one which is different from the other three responses
    A. Short - Long B. Man - Woman C. Light - Heavy D. Crime - Blame

Answer - Option D
Explanation :
Except the pair Crime – Blame, all the other pairs of words are opposite (antonym) of the other.
Hence, option D is correct.
3. Find the odd one out
    A. Mumbai B. Chennai C. Kolkatta D. Bangalore

Answer - Option D
Explanation :
Bangalore is the odd one out because all others are the changed names of the cities whereas Bangalore is the old name.
Hence Option D is correct
Direction: A series is given with one term missing. Select the correct alternative from the given ones that will complete the series.
4. 70, 71, 75, ___, 85, 86, 100, 101
    A. 76 B. 70 C. 80 D. 71

Answer - Option A
Explanation :
Two series:
1. 70, 75, 85, 100 i.e. difference of + 5, +10, + 15
2. 71, 76, 86, 101 i.e. difference of + 5, +10, + 15
5. Select the answer figure in which the question figures are hidden/embedded.
    A. B. C. D.

Answer - Option A
Explanation :
After carefully viewing the figures, it can be say that the given question figure is embedded in answer figure (1).

6. In the following question, select the odd word from the given alternatives.
    A. Humility B. Modesty C. Simplicity D. Vanity

Answer - Option D
Explanation :
Humility, Modesty and Simplicity all are related with Humility. As Modesty is synonym and Vanity is antonym of Humility. Hence, option D is the correct response.
7. Directions: In each of the following questions select the related word/letters/number from the given alternatives. ABCD : EFGH : : IJKL : ?
    A. MNOP B. UVWX C. MNPQ D. NOPQ

Answer - Option A
Explanation :
These three segment are in serial from their first latter like
ABCD (Start from A)
EFGH (start from A + 4 = E)
IJKL (start from I + 4 = M)
So next series will be
MNOP (start from M)
8. In the following question, select the related letters from the given alternatives. EGIK : LJHF : : SUWY : ?
    A. ZXVT B. LNPQ C. MOQS D. TVZ

Answer - Option A
Explanation :
EGIK : LJHF
E(+1) = F; G (+1) = J; I (+1) = H, K (+1) = L

Similarly
S (+1)= T; U (+1) = V; W (+1) = X; Y (+1) = Z
Hence

Therefore EGIK : LJHF : : SUWY : ZXVT
9. Direction: Which one set of letters when sequentially placed at the gaps in the given letter series shall complete it? V, VIII, XI, XIV, ?, XX
    A. XV B. XXVII C. IX D. XXIII

Answer - Option B
Explanation : V --> (+3)--> VIII --> (+3)--> XI-->(+3)-->XIV --> (+3) --> XVII -->(+3)-->XX
Hence, XVII will complete the series.
10. Directions: In each of the following questions, select the related word/letters/ number from the given alternatives. Clock : Time : : Thermometer : ?
    A. Heat B. Radiation C. Energy D. Temperature

Answer - Option D
Explanation :
Clock shows time. Similarly, thermometer shows the temperature.
1. In the following question, choose the correct alternative from the given ones that will complete the series. 3, 5, 8, 13, 21, 34, 55, ?.
    A. 34 B. 55 C. 21 D. 89

Answer - Option D
Explanation :
The sum of first two consecutive terms is equal to third consecutive term.
3 + 5 = 8; 5 + 8 = 13; 13 + 21 = 34; 34 + 55 = 89
Hence ? = 89
2. A series is given with one term missing. Select the correct alternative from the given ones that will complete the series. 2, 5, 8, 11, 14, ?
    A. 17 B. 16 C. 15 D. 18

Answer - Option A
3. In the following question, select the odd word from the given alternatives.
    A. Cotton B. Nylon C. Silk D. Linen

Answer - Option B
Explanation :
All except Nylon are natural fibres, while nylon is a synthetic fibre.
4. Directions: In each of the following questions select the related letters/ number/figure from the given alternative. UNDATED : ATEDUND :: CORRECT : ?
    A. PRECTOC B. RECTROC C. EXTRORC D. RECTCOR

Answer - Option D
5. Select the correct combination of mathematical signs to replace ‘*’ signs and to balance the given equation. 24 * 34 * 2 * 5 * 12
    A. = ÷ - + B. + - = x C. + ÷ × = D. = ÷ + -

Answer - Option A
Explanation :
24 * 34 * 2 * 5 * 12
=>24 = 34 ÷ 2 – 5 + 12
=>24 = 17 – 5 + 12
Hence, option A is correct.
7. Directions: In each of the following questions select the related word/letters/number from the given alternatives. CAT : DDY : : BIG : ?
    A. CLL B. CLM C. CML D. CEP

Answer - Option A
8. Directions: In each of the following questions, select the related word/letters/ number from the given alternatives 30 : ? : : 130 : 222
    A. 30 B. 40 C. 66 D. 68

Answer - Option D
Explanation :
Here, 130 = 5 × 5 × 5 + 5
222 = 6 × 6 × 6 + 6
and 30 = 3 × 3 × 3 + 3
So, ? = 4 × 4 × 4 + 4 = 68
9. Direction : One statement is given followed by two Assumptions, I and II. You have to consider the statement to be true, even if it seems to be at variance from commonly known facts. You are to decide which of the given assumptions can definitely be drawn from the given statement. Indicate your answer.
Statements: All the people in Kerala are literate. Assumption I: People of Kerala are well educated and cultured. Assumption II: People of Kerala are hardworking and sincere.
    A. Only I is implicit B. Only II is implicit. C. Both I and II are implicit D. Neither I nor II is implicit

Answer - Option D
Explanation : It is given in the statement that all people of Kerala are literate. This is opposite to what is given in assumption I. So, I is not implicit. Also, assumption II is not implicit as it is not given in the sentence whether the people of Kerala are hardworking and sincere or not. Thus, we can say that option D is the correct answer.
10, In the following questions, a series is given, with one term missing. Choose the correct alternative from the given ones that will complete the series. 3, 9, 6, 36, 30, ?
    A. 950 B. 400 C. 900 D. 800

Answer - Option C
Explanation : The second term is the perfect square of the first term. Take two numbers at a time.
3 × 3 = 9
6 × 6 = 36
30 × 30 = 900
Hence Option C is correct
1. In the following question, select the one which is different from the other three responses.
    A. Ink – Pen B. Petrol-Car C. Dust - Vacuum cleaner D. Electricity – Television

Answer - Option C
Explanation :
Dust is cleaned through vacuum cleaner while in all other pairs of words, the first item is necessary for the second item for proper functioning.
Hence, option C is correct.
2. In each of the following question, select the one which is different from the other three responses
    A. Lawyer B. Carpenter C. Doctor D. Engineer

Answer - Option B
Explanation :
Carpenter is skilled labour while the rest of others requires some certain studies.
Hence, option B is correct.
3. Which one set of letters when sequentially placed at the gaps in the given letter series shall complete it? 16, 25, 36, ?,64
    A. 56 B. 49 C. 80 D. 72

Answer - Option B
Explanation :
9 + 7 = 16;
16 + 9 = (7 + 2) = 25;
25 + 11 = (9 + 2) = 36
Hence every time the difference between two consecutive terms are increased by 2.
Hence ? = 36 + (11 + 2) = 36 + 13 = 49 &
49 + 15 = (13 + 2) = 64
Therefore ? = 49
4, Which one set of letters when sequentially placed at the gaps in the given letter series shall complete it? BE, HK, NQ, ?
    A. PR B. SU C. TW D. UW

Answer - Option C
Explanation :
B( +3) = E
E( +3) = H; H( +3) = K
K( +3) = N; N( +3) = Q
Similarly; Q( +3) = T; T( +3) = W
Hence ‘?’ = TW
5. Select the correct combination of mathematical signs to replace * signs and to balance the given equation. 8 * 5 * 2 * 72 * 4
    A. = × + ÷ B. × = + ÷ C. × + = ÷ D. + × = ÷

Answer - Option D
Explanation :
8 * 5 * 2 * 72 * 4
8 + 5 * = 72 ÷ 4
8 + 10 = 18
6. In the following question, select the odd letter from the given alternatives
    A. 926 B. 122 C. 225 D. 440

Answer - Option C
Explanation :
Here only 225 is the perfect square of 15, rest all numbers are not perfect square of any whole number.
Hence, option C is the correct response.
7. Select the one which is different from the other three responses
    A. Carrom B. Golf C. Cricket D. Hockey

Answer - Option A
Explanation : \Carrom is indoor game, while others are outdoor games.
8. Direction : One statement is given followed by two conclusions I and II. You have to consider the statements to be true even if they seem to be at variance from commonly known facts. You are to decide which of the given conclusions if any, follow from the given statement Indicate your answer. Statement : Metals expand when heated. Conclusions : I. Non-metals do not expand when heated. II. All things which expand when heated are metals.
    A. Only conclusion I follow B. Only conclusion II follows. C. Both conclusions I and II follow D. Neither conclusion I nor II follow

Answer - Option D
9. Direction : One statement is given followed by two conclusions I and II. You have to consider the statements to be true even if they seem to be at variance from commonly known facts. You are to decide which of the given conclusions if any, follow from the given statement Indicate your answer.
I. Non-metals do not expand when heated. (not follow as nothing is mentioned about the property of non metals.)
II. All things which expand when heated are metals. (not follow, as the thing which can expand after heating may be thing which is neither metal nor non- metal). Hence, option D is the right answer.
Direction: Select the related word/letters/ number from the given alternatives. CHAIR : RIAHC : : TABLE : ?
    A. BATLE B. EBATL C. ELBAT D. ELTAB

Answer - Option C
10. In a certain code ‘KINDLE’ is written as ‘ELDNIK’ how ‘EXOTIC’ can be written in that code?
    A. EXIOTC B. COXITE C. CSOTIE D. CITOXE

Answer - Option D
1. Direction: APPRECIATION is coded as 177832419465. How will you code PERCEPTION?
    A. 7382379465 B. 7392378465 C. 7292373465 D. 7383297465

Answer Option A
2. In the following question, select the related word/letters from the given alternatives. Ravishanksr : Sitar :: Bismillah khan : ?
    A. Sarod B. Santoor C. Shehna D. Flute

Answer - Option C
Explanation :
Ravishankar played sitar, Similarly Bismillah Khan played Shehnai.
3. Direction: Select the related word/letters/ number from the given alternatives. DB: IC:: YE: ?
    A. FJ B. DF C. TH D. WL

Answer - Option B
Explanation :
The pattern is:
D + 5 = I
B + 1 = C
Similarly,
Y + 5 = D
E + 1 = F
4. In a certain code COURSE is written as ESRUOC. How can BREATH be written in that code?
    A. HATEBR B. HTAERB C. HTBREA D. HEATRB

Answer - Option B
Explanation :
The letters have been written in the reverse order in the code “COURSE”. Therefore “BREATH” can be written as HTAERB.
5. In a certain code COURSE is written as ESRUOC. How can BREATH be written in that code?
    A. HATEBR B. HTAERB C. HTBREA D. HEATRB

Answer - Option B
6. In the following questions, find the odd letters/ words/ number pair from the given alternatives.
    A. 48 – 68 B. 71 – 87 C. 5 – 21 D. 29 – 45

Answer - Option A
Explanation :
68 – 48 = 20
87 – 71 = 16
21 – 5 = 16
45 – 29 = 16
Thus 48 – 68 is odd term.
Hence Option A is correct
7. Direction: Select the one which is different from the other three responses
    A. River B. Ocean C. Lake D. Rain

Answer - Option D
Explanation : Except Rain, all others are water bodies.
8. If 'EXPANSION' is written as 248537693, in a certain code, how would 'PENSION' be written in that code ?
    A. 8236793 B. 8237639 C. 8237693 D. 8233769

Answer - Option C
9. Direction:In each of the following questions select the related word/letters/number from the given alternatives. ZA : YB :: XC : ?
    A. YZ B. BC C. OP D. WD

Answer - Option D
Explanation :
ZA : YB
Z is 1st from last and A is 1st from starting
Y is 2nd from last and B is 2nd from starting
Z (-1) = Y; A(+1) = B
Similarly
XC : ?
X is 3rd from last and C is 3rd from starting
Hence X(-1) = W; C(+1) = D
? = WD
10. Direction: Select the related word/letters/ number from the given alternatives. AEG: GKM :: MQS : ?
    A. SWY B. SWZ C. ZWQ D. SWB

Answer - Option A
1. Five boys A, B, C, D and E are standing in a row. D is on the right of E. B is on the left of E, but on the right of A. D is on the left of C, who is standing on the extreme right. Who is standing in the middle ?
    A. E B. B C. C D. D E. A

Answer - Option E
2, Directions: In the following question, which one of the given responses would be a meaningful order of the following words in ascending order? 1) Point 2) Triangle 3) Square 4) Angle 5) Line
    A. 4, 1, 5, 2, 3 B. 3, 2, 1, 5, 4 C. 2, 1, 4, 5, 3 D. 1, 5, 4, 2, 3

Answer - Option D
Explanation :
Ascending order of words:
Point (1) > Line (5) > Angle (4) > Triangle (2) > Square (3)
Hence, Option (D) is correct.
3. If police is called teacher, teacher is called politician, politician is called doctor, doctor is called lawyer and lawyer is called surgeon, who will arrest the criminals?
    A. Teacher B. Doctor C. Police D. Lawyer

Answer - Option A
Explanation :
Police arrests the criminals. Here police has been called as Teacher.
4. In a certain code COURSE is written as ESRUOC. How can BREATH be written in that code?
    A. HATEBR B. HTAERB C. HTBREA D. HEATRB

Answer - Option B
Explanation :
The letters have been written in the reverse order in the code “COURSE”. Therefore “BREATH” can be written as HTAERB.
5. The sum of the ages of a father and a son presently is 70 years. After 10 years the son's age is exactly half that of the father's. What are their ages now ?
    A. 45 years, 25 years B. 50 years, 20 years C. 47 years, 23 years D. 50 years, 25 years

Answer - Option B
Explanation :
Let present age of Son = x year
and present age of father = y years
x + y = 70 years ..........(i)
After 10 years [latex]\rightarrow[/latex]
2{x + 10) = y + 10
2x - y = -10 ........(ii)
From Eq. (i) and (2) [latex]\rightarrow[/latex]
x = 20 years and y = 50 years
Hence, option B is the correct response.
6. Find the odd one
    A. KJML B. GFIH C. TSVU D. ZABY

Answer - Option D
7. In the following question, select the related word from the given alternatives.
    A. Malnutrition B. Illiteracy C. Drought D. Death

Answer - Option A
Explanation :
Unemployment results into poverty. Similarly, malnutrition results into Anaemia. Hence, option A is the correct response.
8. In each of the following questions, select the related word/letters/ number from the given alternatives. Jewellery : Gold : : Furniture : ?
    A. Wood B. Paint C. Table D. Tree

Answer - Option A
Explanation :
Gold is used to make Jewellery. Similarly, wood is used to make furniture. Hence, option A is correct.
9. Select the missing numbers from the given alternatives. 7 4 5 2 3 ? 5 9 6 70 108 240
    A. 7 B. 8 C. 6 D. 9

Answer - Option B
Explanation :
7 × 2 × 5 = 70
4 × 3 × 9 = 108
5 × ? × 6 = 240
Thus ? = [latex]\frac {240}{30}[/latex] = 8
Hence Option B is correct
10. In the following question, choose the correct alternative from the given ones that will complete the series. 3, 5, 8, 13, 21, 34, 55, ?.
    A. 34 B. 55 C. 21 D. 89

Answer - Option D
Explanation :
The sum of first two consecutive terms is equal to third consecutive term.
3 + 5 = 8; 5 + 8 = 13; 13 + 21 = 34; 34 + 55 = 89
Hence ? = 89
1. Directions: In each of the following questions, select the related word/letters/ number from the given alternatives. Defy : Obey : : Rest : ?
    A. Lazy B. Idle C. Labour D. Hard

Answer - Option C
Explanation :
“Defy” is opposite in meaning to “Obey”. Similarly, “Rest” is opposite in meaning to “Labour”.
2, Direction: Choose the correct alternative from the given ones that will complete the series. A, Z, B, Y, C, X, ?, ?.
    A. EV B. WD C. DW D. YV

Answer - Option C
Explanation :
The series can be braked in to two different series.
A, B, C, ? & Z, Y, X, ?
Clearly first series is increasing by 1 in every next step and second series is decreasing by 1 in every next step
Therefore A, B, C, ? = D & Z, Y, X, ? = W
[latex]\Rightarrow[/latex] ?, ? = D, W
3, Directions: In each of the following questions one/two statements are given followed by two conclusions (I) and (II). You have to consider the two statements to be true even if they seem to be at variance from commonly known facts. You have to decide which of the given conclusions, if any, follow from the given statements.
Statements: “Bats do not lay eggs and therefore, bats are not birds.” Conclusions : (I) All birds lay eggs. (II) Only birds lay eggs.
    A. Only (I) follows B. Only (II) follows C. Both (I) and (II) follow D. Neither (I) nor (II) follows

Answer - Option A
Explanation :
It is obvious that laying eggs is an essential criterion to be a bird. Therefore, Conclusion I follows.
Reptiles (Lizards and snakes) also lay eggs so Conclusion II does not follow.
4. In each of the following questions, select the related word/letters /number from the given alternatives. Today: Day before yesterday:: January : ?
    A. December B. February C. November D. March

Answer - Option C
Explanation :
Today - Yesterday - Day before yesterday
January - December – November
Hence, option C is correct.
5. Six friends A, B, C, D, E and F are sitting in a row facing East. C is adjacent to A and E. B is just to the right of E but left of D. F is not at the right end. Who is to the left of A?
    A. e B. c C. d D. f

Answer - Option A
6. A series is given with one (or more) term missing. Choose the correct alternative from the given ones that will complete theseries. 17, 14, 15, l2, 13, ?, ?
    A. 10,11 B. 14,11 C. 11,13 D. 12,15

Answer - Option A
7. Directions:Two statements are given followed by two conclusions I and II. You have to consider the statements to be true even if they seem to be at variance from commonly known facts. You are to decide which of the given conclusions, if any, follow from the given statements. Indicate your answer. Statements : Teaching is an art. Drawing is also an art. Conclusions I. All artists are teachers II. All artists know to draw pictures.
    A. Only conclusion I follow B. Only conclusion II follows C. Neither conclusion I nor II follows D. Both conclusions I and II follow

Answer - Option C
Explanation :
Given Statements :
Teaching is an art and Drawing is also an art.
From the given statements we cannot confer that all teachers are drawing picture and all persons who draw the picture are teachers Therefore all the artist cannot be ‘teacher’ and cannot draw the pictures.
Therefore no conclusion follows from the given statements.
8, Find the odd word/letters/number pair from the given alternatives
    A. FHJL B. DGIK C. KMOQ D. SUWY

Answer - Option B
9. Which one set of letters when sequentially placed at the gaps in the given letter series shall complete it? T, R, P, N, L, _?_
    A. J, G B. J, H C. K, H D. K, I

Answer - Option B
10. In the following question, select the related word/letters from the given alternatives. Ravishanksr : Sitar :: Bismillah khan : ?
    A. Sarod B. Santoor C. Shehnai D. Flute

Answer - Option C
1. Direction: Select the related word/letters/ number from the given alternatives. DB: IC:: YE: ?
    A. FJ B. DF C. TH D. WL

Answer- Option B
Explanation :
The pattern is:
D + 5 = I
B + 1 = C
Similarly,
Y + 5 = D
E + 1 = F
2. In a certain code COURSE is written as ESRUOC. How can BREATH be written in that code?
    A. HATEBR B. HTAERB C. HTBREA D. HEATRB

Answer - Option B
Explanation :
The letters have been written in the reverse order in the code “COURSE”. Therefore “BREATH” can be written as HTAERB.
3. In the following questions, find the odd letters/ words/ number pair from the given alternatives.
    A. 48 – 68 B. 71 – 87 C. 5 – 21 D. 29 – 45

Answer - Option A
Explanation :
68 – 48 = 20
87 – 71 = 16
21 – 5 = 16
45 – 29 = 16
Thus 48 – 68 is odd term.
Hence Option A is correct
4. Direction: Select the one which is different from the other three responses
    A. River B. Ocean C. Lake D. Rain

Answer - Option D
Explanation :
5. If 'EXPANSION' is written as 248537693, in a certain code, how would 'PENSION' be written in that code ?
    A. 8236793 B. 8237639 C. 8237693 D. 8233769

Answer - Option C
6. Direction:In each of the following questions select the related word/letters/number from the given alternatives. ZA : YB :: XC : ?
    A. YZ B. BC C. OP D. WD

Answer - Option D
Explanation :
ZA : YB
Z is [latex]{1}^{st}[/latex] from last and A is [latex]{1}^{st}[/latex] from starting
Y is [latex]{2}^{nd}[/latex] from last and B is 2nd from starting
Z (-1) = Y; A(+1) = B
Similarly
XC : ?
X is [latex]{3}^{rd}[/latex] from last and C is [latex]{3}^{rd}[/latex] from starting
Hence X(-1) = W; C(+1) = D
? = WD
7. Select the correct combination of mathematical signs to replace * signs and to balance the given equation. 9 * 7 * 2 * 3 * 10
    A. - + × = B. ÷ × + = C. - + ÷ = D. + - × =

Answer - Option B
8. In the following question, select the odd letter from the given altternatives.
    A. DGLS B. MPSV C. HKPW D. KNSZ

Answer - Option B
9. In the following question, select the odd letter from the given alternatives.
    A. BEH B. CFI C. DGJ D. EHL

Answer - Option D
Explanation :
Hence, option D is correct.
10. In the following question, select the related word from the given alternatives. Tamil Nadu : Chennai : Manipur : ?
    A. Imphal B. Aizwal C. Itanagar D. Dispur

Answer - Option A
Explanation :
Chennai is capital of Tamil Nadu similarly, Imphal is capital of Manipur.
Hence, option A is the right answer.
1. In each of the following questions, select the one which is different from the other three responses:
    A. EBD B. IFH C. QNO D. YVX

Answer - Option C
Explanation :
Hence, option C is correct.
2. Direction: If JACOB can be written as QZXLY, then KENDY can be written as ___________
    A. PVWMA B. PVMWB C. PUMWB D. PVMWA

Answer - Option B
Explanation :

3. In the following questions, nd the odd number/word from the given alternatives
    A. 443 B. 633 C. 821 D. 245

Answer - Option B
Explanation :
443 = 4 + 4 + 3 = 11
633 = 6 + 3 + 3 = 12
821 = 8 + 2 + 1 = 11
245 = 2 + 4 + 5 = 11
Thus 633 is the odd one out.
Hence Option B is correct
4. Direction: In the following questions, which one of the given responses would be a meaningful order of the following? 1. Evaluation 2. Presentation 3. Recap 4. Aim announcement 5. Motivation
    A. 1, 2, 5, 3, 4 B. 5, 2, 1, 4, 3 C. 5, 4, 2, 3, 1 D. 2, 1, 3, 4, 5

Answer - Option C
Explanation : Motivation-- Aim announcement—Presentation—Recap-- Evaluation.
Hence option C is the right answer.
5. A series is given with one term missing. Select the correct alternative from the given ones that will complete the series. 2, 5, 8, 11, 14, ?
    A. 17 B. 16 C. 15 D. 18

Answer - Option A
Explanation :
6. Direction: Select the related word/letters/ number from the given alternatives. Doctor : Hospital : : ?
    A. Plumber : Wrench B. Chef : Kitchen C. Water : Reservoir D. Farmer : Village

Answer - Option B
Explanation :
This question is about worker and workstation.
As doctors work in a hospital similarly chef works on kitchen.
Doctor : Hospital : : Chef : Kitchen
Hence B is the right answer.
7. In each of the following questions, select the one which is different from the other three responses:
    A. EBD B. IFH C. QNO D. YVX

Answer - Option C
Explanation :

Hence, option C is correct.
8, Directions: In each of the following questions, select the related word/letters/ number from the given alternatives. 25 : 125 : : ? : 3125
    A. 525 B. 625 C. 725 D. 825

Answer - Option B
Explanation :
[latex]{(5)}^{2}[/latex] = 25
[latex]{(5)}^{3}[/latex] = 125
[latex]{(5)}^{4}[/latex] = 625
[latex]{(5)}^{5}[/latex] = 3125
9. Direction: Select the one which is different from the other three responses.
    A. 286 - 628 B. 397 - 739 C. 475 - 574 D. 369 - 936

Answer - Option C
Explanation :

10. Direction: Select the one which is different from the other three responses
    A. CDFE B. JKLM C. STVU D. WXZY

Answer - Option B
Explanation :
CDFE, STVU and WXZY are alike, as
C(+1) [latex]\rightarrow[/latex] D(+2) [latex]\rightarrow[/latex] F(-1) [latex]\rightarrow[/latex] E
S(+1) [latex]\rightarrow[/latex] T(+2) [latex]\rightarrow[/latex] V(-1) [latex]\rightarrow[/latex] U
W(+1) [latex]\rightarrow[/latex] X(+2) [latex]\rightarrow[/latex] Z(-1) [latex]\rightarrow[/latex] Y
But JKLM: J(+1) [latex]\rightarrow[/latex] L(+1) [latex]\rightarrow[/latex] L(+1) [latex]\rightarrow[/latex] M
Hence JKLM has different rule from others, therefore JKLM is odd from others.
1. In a class, Karthik's rank is [latex]{17}^{th}[/latex] from the top and 28th from the bottom. How many students are there in the class?
    A. 45 B. 41 C. 40 D. 44

Answer - Option D
Explanation :
Total number of students in the class = 17 + 28 – 1 = 44
2. In the following question, select the related number from the given alternatives. ACE : HJL:: LNP : ?
    A. SUW B. MOQ C. IJK D. BDF

Answer - Option A
Explanation :


Thus LNP is related to SUW.
3. In the following question, select the word which cannot be formed using the letters of the given word. SIGNATURE
    A. GAIN B. NATURE C. GATE D. SIGHT

Answer - Option D
Explanation :
In the given word there are 1S, 1I, 1G, 1N, 1A, 1T, 1U, 1R, 1E. There is no ‘H’ letter in the given word. Therefore, the word SIGHT cannot be formed. Hence, option D is the correct response.
4. Directions: In each of the following questions, select the missing number from the given responses. 16, 4, 4 81, 3, 27 ?, 25, 5
    A. 97 B. 12 C. 125 D. 30

Answer - Option C
Explanation :
Row wise,
First number = Second number x Third number
16 = 4 [latex]\times[/latex] 4
81 = 3 [latex]\times[/latex] 27
? = 25 [latex]\times[/latex] 5 = 125
Hence, 125 will come in place of question mark.
5. From the given alternatives select the word which can be formed using the letters given in the word. INTERNATIONAL
    A. ANNUAL B. LAMINATION C. TERMINATE D. INTERNAL

Answer - Option D
Explanation :
INTERNAL word can be formed with the letters INTERNATIONAL.
Annual cannot be formed because there is no 'U' letter in the INTERNATIONAL and also there is no 'M' letter in the INTERNATIONAL, so word - Terminate & Lamination also cannot be formed.
6. Find out the odd word/letters/number/number pair from the given alternatives.
    A. 431 B. 835 C. 871 D. 653

Answer - Option D
Explanation :
Here,
431 [latex]\rightarrow[/latex] 4 – 3 = 1
835 [latex]\rightarrow[/latex] 8 – 3 = 5
871 [latex]\rightarrow[/latex] 8 – 7 = 1
653 [latex]\rightarrow[/latex] 6 – 5 = 1 not 3
Thus 653 is the odd one.
7. Direction: If rectangle = 12, triangle = 15, square = 6, parallelogram = 4 and circle = 3, solve the equation using the above values and answer in figures. [latex]\frac {rectangle - square}{triangle}[/latex] = ?
    A. [latex]\frac {4}{5}[/latex] B. [latex]\frac {3}{5}[/latex] C. [latex]\frac {6}{5}[/latex] D. [latex]\frac {2}{3}[/latex]

Answer - Option C
Explanation :
Rectangle = 12
Square = 6
Triangle = 15
[latex]\frac {(12 + 6)}{15}[/latex] = [latex]\frac {18}{15}[/latex] = [latex]\frac {6}{5}[/latex]
Hence Option C is correct
8. Arrange the given words in the sequence in which they occur in the dictionary.
    i. Arise ii. Abysmal iii. Agility iv. Accrue

    A. ii, i, iv, iii B. iii, iv, ii, i C. ii, iv, i, iii D. ii, iv, iii, i

Answer - Option D
Explanation :
Alphabetical order is:
    ii. Abysmal iv. Accrue iii. Agility i. Arise

So the correct order is (ii, iv, iii, i).
9. Direction: Select the one which is different from the other three responses.
    A. Eye B. Hand C. Nose D. tongue

Answer - Option B
Explanation :
Eye, nose and tongue are the parts of face, but ‘hand’ is not a part of face. Therefore ‘hand’ will be odd from others.
10. Select the related word/letters/ number from the given alternatives. Smell : Flower :: Taste : ?
    A. Food B. Sweet C. Water D. Salt

Answer - Option A
Explanation :
Smell is the property of flower; similarly taste is the property of food.
Hence Option A is correct
1. In each of the following questions, an equation is solved on the basis of a certain system. On that basis, find out the correct answer from amongst the four alternatives for the unsolved equation in questions. 6 × 9 × 3 = 369 5 × 6 × 4 = 456 4 × 6 × 8 = ?
    A. 486 B. 846 C. 684 D. 864

Answer - Option B
Explanation :
6 × 9 × 3 = 369
5 × 6 × 4 = 456
Similarly;
4 × 6 × 8 = 846
2. The last two digits of the binary equivalent of the number 2 2 6 8 4 2 8 2 4 8 3 is
    A. 01 B. 11 C. 00 D. 10

Answer - Option B
Explanation :
The number 22684282483 has an odd digit at the unit's place. Except the unit's digit the remaining number is completely divisible by 2.
Now take binary equivalent of 3.
[latex]{(3)}_{10} = {(11)}_{2}[/latex]
3. If "S" denotes "multiplied by", "V" denotes "subtracted from", "M" denotes "added to" and "L" denotes "divided by", then 8 V 10 M 96 L 6 S 9 = ?
    A. 140 B. 142 C. 134 D. 144

Answer - Option B
Explanation :
Substituting symbols with their respective operator, the equation becomes,
8 – 10 + 96 ÷ 6 × 9
Applying BODMAS rule we get,
8 – 10 + 96 ÷ 6 × 9
= 8 – 10 + 16 × 9
= 8 – 10 + 144
= 152 - 10
= 142
4. Find the odd word/letters/number pair from the given alternatives.
    A. Publisher B. Author C. Novelist D. Poet

Answer - Option A
Explanation :
Publisher is responsible for publishing books, magazines, journals etc. Author writes books, essay, articles etc. Novelist writes novel while poet composes poetry.
5. A series is given with one term missing. Choose the correct alternative from the given ones that will complete the series. SQ, R, XV, W, KI, ?
    A. J B. L C. M D. O

Answer - Option A
Explanation :
There exists a relation between every two term of the series, the second term of the series is
the middle term of the first term in the series. if we write all the alphabets in English language
‘R’ comes in the middle of ‘S’ and ‘Q’, ‘W’ comes in the middle of ‘X’ and ‘V’ and likewise, ‘J’
Comes in the middle of ‘I’ and ‘K’.
Hence the correct option is A.
6. Direction: Select the related word/letters/ number from the given alternatives. Doctor : Hospital : : ?
    A. Plumber : Wrench B. Chef : Kitchen C. Water : Reservoir D. Farmer : Village

Answer - Option B
Explanation :
This question is about worker and workstation.
As doctors work in a hospital similarly chef works on kitchen.
Doctor : Hospital : : Chef : Kitchen
Hence B is the right answer.
7. Direction: Select the one which is different from the other three responses.
    A. Aluminum B. Iron C. Copper D. Brass

Answer - Option D
Explanation :
All others except BRASS are metals. BRASS is an alloy. Thus BRASS is the odd among the options.
Hence Option D is correct
8. Direction: Which one set of letters when sequentially placed at the gaps in the given letter series shall complete it? XGH, WIJ, VKL, UMN, ?
    A. TOP B. SOP C. HOW D. UJI

Answer - Option A
Explanation :

Hence, TOP will complete the series.
9. For the following question Find the odd word/letters/number pair from the given alternatives
    A. 81 B. 8 C. 16 D. 625

Answer - Option B
Explanation : 81 = [latex]{9}^{2}[/latex]; 16 = [latex]{4}^{2}[/latex]; 625 = [latex]{25}^{2}[/latex]
81, 16 and 625 are square of some natural numbers
But 8 = [latex]{2}^{3}[/latex], 2 is a cube of natural number.
Therefore 8 is odd from others.
10. A series is given with one term missing. Select the correct alternative from the given ones that will complete the series. VUT, ONM, GFE, ?
    A. XWV B. WVX C. XYZ D. WVU

Answer - Option A
Explanation :
V-1 = U
U-1 = T
T-4 = O
O-1 = N
N-1 = M
M-5 = G
G-1 = F
F-1 = E
E-6 = X
X-1 = W
W-1 = V
Hence, we get XWV.
1, Direction: In the following question, select the related letters/word/number from the given alternatives. 72 : 18 : : 56 : ?
    A. 24 B. 14 C. 20 D. 16

Answer - Option B
Explanation :
18 * 4 = 72
14 * 4 = 56
2, 3 daily wage workers A, Band C are distributed Rs. 178 in such a way that A gets Rs. 4 less than C. B gets. Rs. 15 more than A and C gets Rs. 11 less than B. What is the ratio of their shares ?
    A. 57 : 53 : 68 B. 50 : 51 : 52 C. 53 : 56 : 68 D. 53 : 68 : 57

Answer - Option D
Explanation :
Suppose A gets Rs. x.
C gets Rs. x + 4
B gets Rs. x + 15
Now.
x + x 4 + x + 15 = 178
3x = 178 - 19 = 159
i.e, x = 53
A [latex]\rightarrow[/latex] 53
B [latex]\rightarrow[/latex] 53 + 15 = 68
C [latex]\rightarrow[/latex] 53 + 4 = 57
Hence, option D is the right answer.
3. In the following question, which one of the given responses would be a meaningful order of the following? (1) Police (2) Punishment (3) Crime (4) Judge (5) Judgment
    A. 3, 1, 4, 5, 2 B. 3,1, 2, 4, 5 C. 1, 2, 4, 3, 5 D. 3,1, 2, 4, 5

Answer - Option A
Explanation :
Crime> Police> Judge>Judgment> Punishment
This gives the meaningful order of the responses.
4. In the following question, select the related number from the given alternatives.
    A. 8 - 15 B. 25 - 36 C. 49 -64 D. 81- 100

Answer Option A
Explanation :
25 = [latex]{5}^{2}[/latex] & 36 = [latex]{6}^{2}[/latex]
49 = [latex]{7}^{2}[/latex] % 64 = [latex]{8}^{2}[/latex]
81 = [latex]{9}^{2}[/latex] & 100 = [latex]{10}^{2}[/latex]
Numbers in each of these pair are squares of consecutive numbers.
5. A series is given with one term missing. Select the correct alternative from the given ones that will complete the series. DE, HI, LM, ?
    A. QA B. PR C. PQ D. PU

Answer - Option C
Explanation :

Thus the next term will be PQ.
6, Select the one which is different from the other three responses.
    A. 746 B. 692 C. 581 D. 737

Answer - Option C
Explanation :
7 + 4 + 6 = 17,
6 + 9 + 2 = 17,
5 + 8 + 1 = 14,
7 + 3 + 7 = 17
Addition of digits of 746, 692 and 737 is 17 except 581. Hence, option C is different from others.
7. Direction: A series is given with one term missing. Select the correct alternative from the given ones that will complete the series. 3, 6, 12, 21, ?, 48
    A. 31 B. 33 C. 34 D. 38

Answer - Option B
Explanation :

Hence, 33 will come in place of question mark.
8. Direction: Select the one which is different from the other three responses.
    A. CFH B. LOQ C. XAC D. KMO

Answer - Option D
Explanation :

Hence, option D is correct
9. Arrange the words in ascending order of their magnitude (1) Mega (2) Kilo (3) Tera (4) Giga
    A. (1), (2), (3), (4) B. (1), (3), (2), (4) C. (2), (1), (4), (3) D. (2), (4), (3), (1)

Answer - Option C
Explanation :
Given units are used of weighing.
Ascending order of their magnitude is:
Kilo – [latex]{10}^{3}[/latex]
Mega – [latex]{10}^{6}[/latex]
Giga – [latex]{10}^{9}[/latex]
Tera – [latex]{10}^{12}[/latex]
10. Arrange the given words in the sequence in which they occur in the dictionary. A- Advertise B- Admin C- Addition D- Adhesive
    A. A, B, C, D B. C, D, B, A C. C, B, D, A D. D, A, B, C

Answer - Option B
Explanation :
The given words are:
A d v e r t i s e
A d m i n
A d d i t i o n
A d h e s i v e
First two letters are common in all hence already arranged let we cut those letters;
A d v e r t i s e
A d m i n
A d d i t i o n
A d h e s i v e
In third place letters are (v, m, d, h), clearly ‘d’ comes first than ‘h’, after it ‘m’ and at last ‘v’.
Therefore arrangement of words are as follows:
A d d i t i o n (C)
A d h e s i v e (D)
A d m i n (B)
A d v e r t i s e (A)
1. Find the odd one out.
    A. 4 B. 1 C. 3 D. 3

Answer - Option C
Explanation :
Except 3 in each figure two figures are colored in Black color.
The correct option is C.
2. Two statements are given followed by two conclusions I and II. You have to consider the statements to be true even if they seem to be at variance from commonly known facts. You are to decide which of the given conclusions, if any, follow from the given statements. Indicate your answer. Statements : Teaching is an art. Drawing is also an art. Conclusions I. All artists are teachers II. All artists know to draw pictures.
    A. Only conclusion I follow B. Only conclusion II follows C. Neither conclusion I nor II follows D. Both conclusions I and II follow

Answer - Option C
Explanation :
Given Statements :
Teaching is an art and Drawing is also an art.
From the given statements we cannot confer that all teachers are drawing picture and all persons who draw the picture are teachers Therefore all the artist cannot be ‘teacher’ and cannot draw the pictures.
Therefore no conclusion follows from the given statements.
3. For the following question Find the odd word/letters/number pair from the given alternatives
    A. Moon B. Mars C. Venus D. Jupiter

Answer - Option A
Explanation :
Except Moon, all others are planets of our solar system. Moon is a satellite.
Hence, option A is correct.
4. In the following question, select the odd letter from the given alternatives.
    A. MPS B. WZC C. FHJ D. GJM

Answer - Option C
Explanation :
M (+3) [latex]\rightarrow[/latex] P (+3) ? S
W (+3) [latex]\rightarrow[/latex] Z (+3) ? C
F (+2) [latex]\rightarrow[/latex] H (+2) ? J
G (+3) [latex]\rightarrow[/latex] J (+3) ? M
Thus, FHJ is the odd one.
5. Select the related word/letters/number from the given alternatives. Railway Station : Station Master : : School : ?
    A. Chef B. Doctor C. Lieutenant D. Principal

Answer - Option D
Explanation :
As Station Master heads the Railway Station likewise, School is headed by the Principal.
Hence, the correct option is D.
6. Direction: Select the one which is different from the other three responses.
    A. CFH B. KNP C. MPR D. JLN

Answer - Option D
Explanation :
C(+3) [latex]\rightarrow[/latex] F(+2) [latex]\rightarrow[/latex] H
K(+3) [latex]\rightarrow[/latex] N(+2) [latex]\rightarrow[/latex] P
M(+3) [latex]\rightarrow[/latex] P(+2) [latex]\rightarrow[/latex] R
J(+2) [latex] \rightarrow[/latex] L(+2) [latex]\rightarrow[/latex] N
Clearly JLN has different rule then others, therefore JLN is odd.
7. Directions: In each of the following questions one/two statements are given followed by two conclusions (I) and (II). You have to consider the two statements to be true even if they seem to be at variance from commonly known facts. You have to decide which of the given conclusions, if any, follow from the given statements. Statements: “Bats do not lay eggs and therefore, bats are not birds.” Conclusions : (I) All birds lay eggs. (II) Only birds lay eggs.
    A. Only (I) follows B. Only (II) follows C. Both (I) and (II) follow D. Neither (I) nor (II) follows

Answer - Option A
Explanation :
It is obvious that laying eggs is an essential criterion to be a bird.
Therefore, Conclusion I follows.
Reptiles (Lizards and snakes) also lay eggs so Conclusion II does not follow.
8. Direction: Which one set of letters when sequentially placed at the gaps in the given letter series shall complete it? XGH, WIJ, VKL, UMN, ?
    A. TOP B. SOP C. HOW D. UJI

Answer - Option A
Explanation :

Hence, TOP will complete the series.
9. Direction: Select the one which is different from the other three responses.
    A. January B. June C. July D. August

Answer - Option B
Explanation :
Except June, all other months are of 31 days each. Hence option B is different from others.
10. Direction: Arrange the given words in the sequence in which they occur in the dictionary. i. Drama ii. Dramatical iii. Dramaticism iv. Dramality v. Dramatise
    A. i, iv, ii, iii, v B. i, iv, ii, v , iv C. iv, ii, v , iv, i D. v , iv, i, iv, ii

Answer - Option A
Explanation :
Clearly,
Drama > Dramality > Dramatical > Dramaticism> Dramatise
Therefore, I > iv > ii > iii > v
Hence, A is correct.
1. Select the missing number from the given responses:
    A. 122 B. 222 C. 212 D. 132

Answer - Option B
Explanation :
5 [latex]\times[/latex]2 + 2 = 12
12 [latex]\times[/latex] 2 + 2 = 26
26 [latex]\times[/latex] 2 + 2 = 56
54 [latex]\times[/latex] 2 + 2 = 110
110 [latex]\times[/latex] 2 + 2 = 222
2. Directions: Find the odd word/letters/number/number pair from the given alternatives.
    A. srqp B. lkji C. hgfe D. uvwx

Answer - Option D
3. If in a code language 'PUTREFY' is written as 'XPQSTRL' and 'NAVIGATE' is written as ‘GYMOWYQT’, how is ‘AVIARY’ written in that language?
    A. YOMYLS B. YMOYLS C. YMOYSL D. YOMYSL

Answer - Option C
Explanation :
'PUTREFY' is written as 'XPQSTRL'

'NAVIGATE' is written as ‘GYMOWYQT’

Hence ‘AVIARY’ can be written

AVIARY [latex] \rightarrow [/latex] YMOYSL
4. Direction: A series is given with one term missing. Select the correct alternative from the given ones that will complete the series. 4, 11, 30, 67, 128, ?
    A. 219 B. 228 C. 231 D. 237

Answer - Option A
Explanation :
4 = [latex]{1}^{3}[/latex] + 3
11 = [latex]{2}^{3}[/latex] + 3
30 = [latex]{3}^{3}[/latex] + 3
67 = [latex]{4}^{3}[/latex] + 3
128= [latex]{5}^{3}[/latex] + 3
Similarly,
? = [latex]{6}^{3}[/latex] + 3 = 216 = 3 = 219
? = 219
5. In the following question, which one of the given responses would be a meaningful order of the following words in ascending order? (1) Plant (2) Food (3) Seed (4) Leaf (5) fruit
    A. (5), (4), (3), (2), (1) B. (3), (2), (4), (5), (1) C. (3), (1). (4), (5), (2) D. (1), (3), (5), (4), (2)

Answer - Option C
Explanation :
Seed grows and convert into plant, plant contains leafs, fruits are found on leaf of plant. Foods can be found from fruits.
Therefore correct ascending order of given words are:
Seed [latex]\rightarrow[/latex] plant [latex]\rightarrow[/latex] leaf [latex]\rightarrow[/latex] fruit [latex]\rightarrow[/latex] food
6. In the following questions select the related letters/word/ number from the given alternatives. Smoke : Pollution :: War : ?
    A. Treaty B. Destruction C. Peace D. Victory

Answer - Option B
Explanation :
The second term denotes the results of the first term. Smoke causes pollution; similarly War causes Destruction.
Hence Option B is correct.
7. Direction: Select the related word/letters/ number from the given alternatives. Fodder : Cattle : : ?
    A. Pen : Ink B. Ball : Stick C. Fruit : Juice D. Grass : Horses

Answer - Option D
Explanation :
First is the food, eaten by the second.
Fodder : Cattle Fodder is eaten by cattle,
Similarly grass is eaten by horse.
Hence: Grass : Horses
Fodder : Cattle : : Grass : Horses
8. Direction :Select the missing numbers from the given alternatives.
    A. 1 B. 0 C. 2 D. 4

Answer - Option B
Explanation :
7 = 1 + 2 × 3 = 1 + 6
23 = 3 + 5 × 4 = 3 + 20
21 = 0 + 7 × 3 = 0 + 21
Hence option B is the right answer.
9. Arrange the given words in the sequence in which they occur in the dictionary. i. Arise ii. Abysmal iii. Agility iv. Accrue
    A. ii, i, iv, iii B. iii, iv, ii, i C. ii, iv, i, iii D. ii, iv, iii, i

Answer - Option D
Explanation :
Alphabetical order is:
ii. Abysmal
iv. Accrue
iii. Agility
i. Arise
So the correct order is (ii, iv, iii, i).
10. In the following question, select the odd word from the given alternatives.
    A. Islamabad B. Kabul C. Canberra D. Sydney

Answer - Option D
Explanation :
All are capital city of some country except ‘Sydney’.
Islamabad [latex] \rightarrow [/latex] Pakistan
Kabul [latex] \rightarrow [/latex] Afghanistan
Canberra [latex] \rightarrow [/latex] Australia
Hence, the correct option is D.
1. Select the one which is different from the other three responses :
    A. Cholera B. AIDS C. Cancer D. Health

Answer - Option D
Explanation :
Cholera, AIDS and Cancer are diseases but health is not a disease hence ‘Health’ will be odd from others.
2. Select the odd word/letters/number/ number pair from the given alternatives.
    A. Truck B. Car C. Motorcycle D. Airplane

Answer - Option D
Explanation :
Truck, Car and Motorcycle all run on roads whereas Airplane fly in air.
3. In the following questions select the related letters/word/ number from the given alternatives. Smoke : Pollution :: War : ?
    A. Treaty B. Destruction C. Peace D. Victory

Answer - Option B
Explanation :
The second term denotes the results of the first term. Smoke causes pollution; similarly War causes Destruction.
Hence Option B is correct.
4. In each of the following question, select the one which is different from the other three responses.
    A. XRMKL B. XRMIF C. XRNLJ D. XSNLJ

Answer - Option B
Explanation :
There is a Vowel (I) in the letter group XRMIF while in other letter groups, all letters are consonants.
5. Direction: In the following questions, a series is given, with one term missing. Choose the correct alternative from the given ones that will complete the series. 3, 6, 18, 21, 63, 66, ___?__.
    A. 181 B. 160 C. 147 D. 198

Answer - Option D
Explanation :
3 + 3 = 6
6 × 3 = 18
18 + 3 = 21
21 × 3 = 63
63 + 3 = 66
66 × 3 = 198
Hence Option D is correct
6. Direction: Find the missing number in the following series: 1000, 200, 40, ?
    A. 10 B. 20 C. 15 D. 8

Answer - Option D
Explanation :
[latex]\frac {1000}{5}[/latex] = 200
[latex]\frac {200}{5}[/latex] = 40
[latex]\frac {40}{5}[/latex] = 8
Thus 8 is the required number.
Hence Option D is correct
7. https://gradeup.co/rrb-je-2019-day-81-ga-+-reasoning-revision-quiz-21-i-3431b160-4ee5-11e9-8bf6-605c269f44b0
    A. EAUTYB B. AUTYBE C. YTUAEB D. UTYBEA

Answer - Option C
Explanation :
This question is for reverse sequence of alphabets in a word.
As COMPREHENSION [latex] \rightarrow [/latex] NOISNEHERPMOC (reverse order of alphabets of COMPREHENSION)
Similarly reverse order of alphabets in BEAUTY is YTUAEB
Hence BEAUTY [latex] \rightarrow [/latex] YTUAEB
Hence COMPREHENSION : NOISNEHERPMOC : : BEAUTY : YTUAEB
8. A series is given with one term missing. Select the correct alternative from the given ones that will complete the series. 15, 23, 31, 39, ?, 55, 63
    A. 47 B. 46 C. 44 D. 45

Answer - Option A
Explanation :
15 + 8 = 23
23 + 8 = 31
31 + 8 = 39
39 + 8 = 47
47 + 8 = 55
55 + 8 = 63
Hence, option A is the correct response.
9. If 'EXPANSION' is written as 248537693, in a certain code, how would 'PENSION' be written in that code ?
    A. 8236793 B. 8237639 C. 8237693 D. 8233769

Answer - Option C
Explanation :
EXPANSION' is written as 248537693

Hence 'PENSION' be written as:

Therefore PENSION ? 8237693
10. Find the odd word/letters/number pair from the given alternatives.
    A. Publisher B. Author C. Novelist D. Poet

Answer - Option A
Explanation :
Publisher is responsible for publishing books, magazines, journals etc. Author writes books, essay, articles etc. Novelist writes novel while poet composes poetry.
1. In the following question, select the related number group from the given alternatives. 9 : 27 : : 64 : ?
    A. 225 B. 512 C. 216 D. 324

Answer - Option B
Explanation :
As we can observe in the first set: [latex]{3}^{2}[/latex] = 9 and [latex]{3}^{3}[/latex] = 27
Likewise, [latex]{8}^{2}[/latex] = 64 and [latex]{8}^{3}[/latex] = 512.
Hence, the correct option is B.
2. A series is given with one term missing. Select the correct alternative from the given ones that will complete the series. BD, HJ, NP, ?
    A. TU B. ST C. TV D. SU

Answer - Option C
3. Directions: In each of the following questions, a series is given, with one number missing. Choose the correct alternative from the given ones that will complete the series. 62, 61, 63, 62, 64, 63, _________ .
    A. 66 B. 62 C. 64 D. 65

Answer - Option D
Explanation :
There is an increment of one unit after every one place i.e.
62 + 1 = 63
63 + 1 = 64
64 + 1 = 65
4. In the following question, select the odd word from the given alternatives.
    A. Iron B. Plastic C. Brass D. Copper

Answer - Option B
Explanation :
Iron, Brass and Copper are hard material but Plastic is flexible material. Hence, option B is the right answer.
5. If ‘+’ stands for ‘÷’, ‘×’ stands ‘+’, ‘-’ stands for ‘If ‘+’ stands for ‘÷’, ‘×’ stands ‘+’, ‘-’ stands for ‘×’, and ‘÷’ stands for ‘-’, then which of the following statements is correct?’, and ‘÷’ stands for ‘-’, then which of the following statements is correct?
    A. 36 × 6 + 7 ÷ 2 - 6 = 20 B. 1 × 6 + 7 - 1 = 6 C. 1 × 6 + 7 - 3 = 5 D. 36 - 6 + 3 × 5 ÷ 3 = 74

Answer - Option D
6. Directions: In the following question, which one of the given responses would be a meaningful order of the following words in ascending order? 1) Point 2) Triangle 3) Square 4) Angle 5) Line
    A. 4, 1, 5, 2, 3 B. 3, 2, 1, 5, 4 C. 2, 1, 4, 5, 3 D. 1, 5, 4, 2, 3

Answer - Option D
Explanation :
Ascending order of words:
Point (1) > Line (5) > Angle (4) > Triangle (2) > Square (3)
Hence, Option (D) is correct.
7. Find the odd word/letters/number from the given alternatives.
    A. Bus B. Scooter C. Cycle D. Boat

Answer - Option D
Explanation :
Bus, Scooter and Cycle are the vehicle that goes on Road but Boat runs onto the water.
8. Direction: Select the one which is different from the other three responses.
    A. 35 B. 37 C. 23 D. 53

Answer - Option A
Explanation :
Except the number 35, all others are prime numbers.
9. If FAITH is coded as 82731, HABIT is coded as 12573 and HEALTH is coded as 192431, how can BELIEF be coded as?
    A. 594598 B. 594789 C. 594978 D. 594798

Answer Option D
10. Six friends A, B, C, D, E and F are sitting in a row facing East. C is adjacent to A and E. B is just to the right of E but left of D. F is not at the right end. Who is to the left of A?
    A. E B. C C. D D. F

Answer - Option D
1. If B = 2, BAG = 10, then BOX = ?
    A. 41 B. 52 C. 36 D. 39

Answer - Option A
Explanation :
B = 2; Position number in the English alphabet.
BAG = 2 + 1 + 7 = 10
Similarly;
BOX = 2 + 15 + 24 = 41
Hence Option A is correct
2. A series is given with one term missing. Select the correct alternative from the given ones that will complete the series. 200, 100, 50, 25, 12.5, 6.25, ?
    A. 2, 125 B. 3,025 C. 3,125 D. 2,025

Answer - Option C
Explanation :
200 ÷ 2 = 100
100 ÷ 2 = 50
50 ÷ 2 = 25
25 ÷ 2 = 12.5
12.5 ÷ 2 = 6.25
6.25 ÷ 2 = 3.125
Thus, the next number in the series will be 3.125.
3. Select the related word/letters/number from the given alternatives. 76 : 42 : : 46 : ?
    A. 52 B. 24 C. 36 D. 64

Answer - Option B
Explanation :
Since ‘42’ is obtained by the multiplication of the digits in ‘76’ i.e.
42 = 7 × 6
Likewise, missing number will be given by: 4 × 6 = 24
Hence, the correct option is B.
4. Direction: In the following questions, a series is given, with one term missing. Choose the correct alternative from the given ones that will complete the series. 3, 6, 18, 21, 63, 66, ___?__.
    A. 181 B. 160 C. 147 D. 198

Answer - Option D
Explanation :
3 + 3 = 6
6 × 3 = 18
18 + 3 = 21
21 × 3 = 63
63 + 3 = 66
66 × 3 = 198
Hence Option D is correct
5. Find the odd word/letters/number from the given alternatives.
    A. Bus B. Scooter C. Cycle D. Boat

Answer - Option D
Explanation :
Bus, Scooter and Cycle are the vehicle that goes on Road but Boat runs onto the water.
6. Direction: Sohan ranks seventh from the top and twenty-sixth from the bottom in a class. How many students are there in the class ?
    A. 34 B. 31 C. 32 D. 33

Answer - Option C
Explanation :
Total number of students in the class
= 7 + 26 - 1 = 32
7. If ‘÷’ means ‘-’, ‘-’ means ‘×’ ‘×’ means ‘+’ and ‘+’ means ‘÷’, then 20 × 60 ÷ 40 - 20 + 10 = ?
    A. 0 B. 40 C. 60 D. 80

Answer - Option A
Explanation :
Given ÷ ? - ; - ? ×; × ? +; + ? ÷
Hence 20 × 60 ÷ 40 - 20 + 10
? 20 + 60 - 40 × 20 ÷ 10
= 80 - 40 × 2 = 80 - 80 = 0
8. A series is given with one term missing. Choose the correct alternative from the given ones that will complete the series. 7, 13, 21, 31, 43, 57, ?
    A. 73 B. 83 C. 78 D. 63

Answer - Option A
Explanation :
Since there is an incremental difference of ‘2’ starting from ‘6’:
13 – 7 = 6
21 – 13 = 8
31 – 21 = 10
43 – 31 = 12
57 – 43 = 14
Therefore, the next term will be: 57 + 16 = 73
Hence, the correct option is A.
9. Direction: Find the missing number in the following series: 32, 58, 92, 134, ___?__.
    A. 184 B. 194 C. 156 D. 169

Answer - Option A
Explanation :

10. A series is given with one term missing. Select the correct alternative from the given ones that will complete the series. 30, 24, 19, 15, 12,?
    A. 6 B. 8 C. 10 D. 11

Answer - Option C
1. Select the missing numbers from the given alternatives
    A. 24 B. 26 C. 28 D. 36

Answer - Option B
Explanation :
Changes in clockwise direction are given as:
6 + 2 = 8; 8 + 3 = 11; 11 + 4 = 15; 15 + 5 = 20
In every next stage difference increases by 1.
Hence ? = 20 + 6 = 26
? = 26
2. If MADRAS is written as DAMSAR, how can MUMBAI be written in that code?
    A. IABMUM B. MBIAUM C. BAIUMM D. MUMIAB

Answer - Option D
3. Find the odd one
    A. KJML B. GFIH C. TSVU D. ZABY

Answer - Option D
4. In the following question, select the related word/letters/number from the given alternatives. VXZ : JLN : : GIK : ?
    A. QSO B. QRS C. OQS D. LMN

Answer - Option C
Explanation :

5. In a certain code ‘KINDLE’ is written as ‘ELDNIK’ how ‘EXOTIC’ can be written in that code?
    A. EXIOTC B. COXITE C. CSOTIE D. CITOXE

Answer - Option D
6. In the following question, select the related letters from the given alternatives. RARE : ERAR : ROCKET : ?
    A. TKEROC B. TEKRCO C. TEKROC D. TEKCOR

Answer - Option D
Explanation :
If RARE is written in reverse order, we get ERAR.
Similarly if ROCKET is written in reverse order, we get TEKCOR.
7. A series is given with one term missing. Choose the correct alternative from the given ones that will complete the series. 30, 62, 189, 760, ?
    A. 3306 B. 1157 C. 2185 D. 3805

Answer - Option D
Explanation :
30 * 2 + 2 = 62
62 * 3 + 3 = 189
189 * 4 + 4 = 760
760 * 5 + 5 = 3805
Hence D
8. Select the missing numbers from the given alternatives.
    A. 34 B. 12 C. 99 D. 104

Answer - Option D
Explanation :
Row-wise
First Row, [latex]{5}^{2}[/latex] +[latex]{4}^{2}[/latex] = 25 + 16 = 41
Second Row, [latex]{7}^{2}[/latex] + [latex]{3}^{2}[/latex] = 49 + 9 = 58
Third Row, [latex]{10}^{2}[/latex] + [latex]{2}^{2}[/latex] = 100 + 4 = 104
104 will come in place of question mark.
Hence option D is the right choice.
9. Select the missing number from the given responses.
    A. 10 B. 12 C. 13 D. 15

Answer - Option C
Explanation :
1 + 2 = 3
2 + 3 = 5
3 + 5 = 8
5 + 8 = [13]
10. In the following question, select the related letters from the given alternatives. EFI : IJM :: MNQ : ?
    A. OPQ B. PQS C. QRU D. OPW

Answer - Option C
Explanation :
EFI : IJM : : MNQ : ?
E + 1 - F
F + 3 - I
I +1 - J
J + 3 - M
M + 1 - N
N + 3 - Q
Hence,
Q + 1 - R
R + 3 = U
So, correct option is C. QRU
1. Directions: Two statements are given followed by four conclusions I, II, III and IV. You have to consider the statements to be true even if they seem to be at variance from commonly known facts. You are to decide which of the given conclusions, if any, follow from the given statements. Indicate you’re answer. Statements : 1. All goats are tigers 2. All tigers are lions. Conclusions : I. All tigers are goats. II. All lions are tigers III. No goat is a lion. IV. No lion is a goat
    A. Either II or III follows B. Either II or IV follows C. Either I or III follows D. None of the conclusions follow

Answer - Option C
2. DirectionIn the following questions, some statements are given followed by two conclusions I and II. You . have to consider the statements to be true even lf they seem to be at variance from commonly, known facts. You are to decide which of the given conclusions. If any, follow from the given statements. Indicate your answer. Statements : All chillies are garlics. Some garlics are onions. All onions are potatoes. No potato is ginger. Conclusions : I. Some chillies are potatoes. II. No onion is ginger.
    A. Only conclusion I follows B. Only conclusion II follows C. Both the conclusions I and II follow D. None of the conclusions I or II follows

Answer - Option B
3. In each of the following questions, a series is given, with one term/number/letter missing. Choose the correct alternative from the given ones that will complete the series. 14, 28, 20, 40, 32, 64, ?
    A. 52 B. 56 C. 128 D. 48

Answer - Option B
Explanation :
14 × 2 = 28
28 – 8 = 20
20 × 2 = 40
40 – 8 = 32
32 × 2 = 64
64 – 8 = [56]
4. Directions: A word given in Capital Letters is followed by four answer words. Out of these only one cannot be formed by using the letters of the given words. Find out that word. FRAMEWORK
    A. MARK B. FAME C. FOUR D. MORE

Answer - Option C
Explanation :
The word FOUR cannot be formed by using the letters of the word FRAMEWORK as U is not there.
Hence Option C is correct
5. Direction: From the given alternative words, select the word which cannot be formed using the letters of the given word: ARCHITECTURE
    A. TACT B. CHAT C. TORCH D. RICH

Answer - Option C
Explanation :
O is not there in the original word.
Hence correct option is C
6. Prakash travelled 6 km north ward, then turned left and travelled 4 km, then turned left and travelled 6 km. How far was Prakash from the starting point?
    A. 6 cm B. 4 km C. 10 km D. 8 km

Answer - Option B
7. A series is given with one term missing. Select the correct alternative from the given ones that will complete the series. ASY, BRX, CQW, ?
    A. DVP B. DPV C. PDV D. PQD

Answer - Option B
8. I went 15 m to the north, then I turned west and covered 10 m, then I turned south and covered 5 m and then turned east and covered 10 m. In which direction am I from the starting point?
    A. East B. West C. North D. South

Answer - Option C
9. If you are [latex]{9}^{th}[/latex] person in a queue starting from one end and [latex]{11}^{th}[/latex] from another end, what is the number of persons in the queue?
    A. 20 B. 19 C. 21 D. 18

Answer - Option B
Explanation :
Number of persons in the queue = 9 + 11 – 1 = 19
10. Shyam was facing East. He walked 5 km forward and then after turning to his right walked 3 km. Again he tuned to his right and walked 4 kms. Which direction was he facing at that time?
    A. East B. West C. North D. South

Answer - Option B
Explanation :
Therefore, he will be in West direction from the starting point.
1. A student walked out from the Classroom towards the Library. She went first to the Canteen on the left side, 24 ft away. After a cup of tea, took a right turn and went to the Laboratory 13 ft away. She then went to the Physics Block, 15 ft to the left. She talked to a friend, in the garden 3 ft. on the left and continued walking in the same direction to the Library, 10 ft more. What was the actual distance between the Library and the Classroom?
    A. 42 ft B. 66 ft C. 39 ft D. 34 ft

Answer - Option C
2. In the following question, select the odd letter group from the given alternatives.
    A. MKO B. QSO C. VTX D. QWE

Answer - Option B
3. A series is given with one term missing. Select the correct alternative from the given ones that will complete the series. EAC, GCE, IEG, ?
    A. JHI B. KGI C. JGI D. KIJ

Answer - Option B
4. Sunita is the 11th from either end of a row of girls. How many girls are there in that row?
    A. 21 B. 22 C. 19 D. 20

Answer - Option A
Explanation :
Total number of girls in the row = 11 + 11 - 1 = 21
Hence Option A is correct
5. In the following questions, a series is given, with one term missing. Choose the correct alternative from the given ones that will complete the series. 7, 8, 11, 16, 23, ?
    A. 37 B. 40 C. 31 D. 32

Answer - Option D
Explanation :
7 + 1 = 8
8 + 3 = 11
11 + 5 = 16
16 + 7 = 23
23 + 9 = 32
Hence Option D is correct
6. In each of the following questions, from the given alternatives select the word which cannot be formed using the letters of the given word. ‘REMEMBRANCE’
    A. NUMBER B. EMBRACE C. REMEMBER D. MEMBRANE

Answer - Option A
Explanation :
There is no “U” letter in the word ‘REMEMBRANCE’ so word “NUMBER” cannot be formed.
7. If in certain code PATTERN is written as NRETTAP, then how MENTION can be written in that code?
    A. NOITMEN B. NMOEINT C. NOITNEM D. NOTIMEN

Answer - Option C
Explanation :
Each letter of the code is in reverse order of the original word.
8. Find the odd one
    A. Iron B. Aluminium C. Wood D. Copper

Answer - Option C
Explanation :
All rest are metals except wood
9. Find the missing term 5 3 7 1 7 5 9 3 4 4 4 4 3 2 ? 1
    A. 6 B. 5 C. 4 D. 2

Answer - Option C
Explanation :
First Row
5 + 7 = 4 * 3
Second Row
3 + 5 = 4 * 2
Third Column
7 + 9 = 4 * ?
? = [latex]\frac {16}{4}[/latex] = 4
Fourth Column
1 + 3 = 4 * 1
10. Find the missing term 27 9 3 4 16 64 512 ? 8
    A. 64 B. 2 C. 16 D. 8

Answer - Option A
Explanation :
First Row
27 = [latex]{(3)}^{3}[/latex]; 9 = [latex]{(3)}^{2}[/latex]
Second Row
16 = [latex]{(4)}^{2}[/latex]; 64 = [latex]{(4)}^{3}[/latex]
Third Column
512 = [latex]{(8)}^{3}[/latex]; 64 = [latex]{(8)}^{2}[/latex]
1. If TIMBER is written is BERMIT in a certain code, how would BANTER be written in that code?
    A. RETNAB B. TERNAB C. TENBAR D. TABNER

Answer - Option B
2. Select the one which is different from the other three responses:
    A. 8 – 64 B. 6 – 36 C. 9 – 81 D. 7 – 50

Answer - Option D
Explanation :
Except in the number pair 7 – 15, in all othe numbe pairs, the second number is
8 * 8 = 64; 6 * 6 = 36;
9 * 9 = 81;
50 = 7 * 7 + 1
3. Direction: Select the one which is different from the other three responses.
    A. GLOV B. CFKR C. ILQX D. ADIP

Answer - Option A
4. A series is given with one term missing. Select the correct alternative from the given ones that will complete the series. ALN, DNP, GPR, ?
    A. KLM B. JRT C. RNU D. RNV

Answer - Option B
5. A series is given with one term missing. Select the correct alternative from the given ones that will complete the series. CAE, HFJ, MKO, RPT, ?
    A. WUY B. UVY C. VUZ D. WUZ

Answer- Option A
6. If SUNDAY = 18, MONSOON = 21, YEAR = 12, then THURSDAY = ?
    A. 26 B. 42 C. 28 D. 24

Answer - Option D
Explanation :
Clearly,
Number of letters in word ‘SUNDAY’ = 6
That is SUNDAY’ = 6 × 3 = 18
Similarly, Number of letters in word ‘MONSOON’ = 7
That is MONSOON = 7 × 3 = 21
And, Number of letters in word ‘YEAR’ = 4
That is YEAR = 4 × 3 = 12
Therefore, Number of letters in word ‘THURSDAY’ = 8
That is THURSDAY = 8 × 3 = 24
Hence, D is correct.
7. In a certain code language, "VENUS" is written as "TVOFW". How is "FAMILY" written in that code language?
    A. ZMJNBG B. GBNJMZ C. EZLQKX D. XKHLZQ

Answer - Option A
8. Arrange the given words in the sequence in which they occur in the dictionary. i. Amoeba ii. Anok iii. Amiable iv. Amalgamate
    A. ii, i, iv, iii B. iii, iv, ii, i C. iv, iii, i, ii D. ii, iv, iii, i

Answer - Option C
Explanation :
Alphabetical order is:
iv. Amalgamate
iii. Amiable
i. Amoeba
ii. Amok
So the correct order is (iv, iii, i, ii).
9. Arrange the given words in the sequence in which they occur in the dictionary. i. Preview ii. Preventive iii. Prefer iv. Preformation
    A. iii,ii,i,iv B. iv,iii,i,ii C. iii,iv,ii,i D. iii,i,ii,iv

Answer - Option C
Explanation :
Arranging in the order they appear in the dictionary,
iii. Prefer
iv. Preformation
ii. Preventive
i. Preview
Hence, the correct option is C.
10. In the following question, select the related word from the given alternatives. Elephant : Tusk :: Parrot : ?
    A. Quill B. Feather C. Beak D. Spine

Answer - Option C
Explanation :
Tusk are the teeth outside the mouth of elephant, similarly beak is a curved part of bird’s mouth. So, ‘beak’ is related to parrot.
Hence, option C is the correct response.
1. Direction: Which one of the alternatives will replace the question mark? 24, 29, 39, ?, 74
    A. 49 B. 54 C. 44 D. 50

Answer - Option B
Explanation :
29 – 24 = 5; 39 – 29 = 10
Difference between two consecutive terms increased by 5 in every next step.
Hence; ? = 39 + 15 = 54
74 – 54 = 20
2. In the following question, select the related number from the given alternatives. 18 : 52 : : 12 : ?
    A. 34 B. 48 C. 60 D. 72

Answer - Option A
Explanation :
Second term is two less than the thrice of first.
18 : 52
52 = 18 × 3 – 2 = 54 – 2
Similarly
12 : ?
[latex]\rightarrow[/latex] = 12 × 3 – 2 = 36 – 2 = 34
[latex]\rightarrow[/latex] ? = 34
18 : 52 : : 12 : 34
Hence, option A is the correct response.
3. Find the odd word/letters/number pair from the given alternatives
    A. 66- 56 B. 101-90 C. 41- 30 D. 33 - 22

Answer - Option A
Explanation :
66 – 56 = 10
101 – 90 = 11
41 – 30 = 11
33 – 22 = 11
4. Find the odd word/letters/number pair from the given alternatives
    A. Reward B. Praise C. Encourage D. Punishment

Answer - Option D
Explanation :
Punishment means the action of punishing somebody or of being punished.
Reward means a thing that is given or received in return for doing something good.
Praise means the expression of approval or admiration.
Encourage means to give support, confidence or hope.
Therefore, Punishment is different from the other three words.
5. Two statements are given, each followed by two conclusion/assumption, I and II. You have to consider the statement to be true even if they seem to be at variance from commonly known facts. You have to decide which of the given conclusion/ assumptions, if any, follows from the given statements. Statements: I. Some papers are pens. II. All the pencils are pens. Conclusions: I. Some pens are pencils. II. Some pens are papers.
    A. Either I or II follows. B. Both I and II follows. C. Only I conclusion follows D. Only II conclusion follows

Answer - Option B
Explanation :
Conclusions:
I. Some pens are pencils. - True- (It is clear from the given diagram)
II. Some pens are papers. - True- (It is clear from the given diagram) Hence, option B is the correct response.
6. If ‘-‘ stands for addition, ‘+’ for multiplication, ‘÷’ for subtraction and ‘×’ for division, which one of the following equations is correct?
    A. 5 + 2 - 12 * 6 + 2 = 10 B. 5 + 2 + 12 * 6 - 2 = 4 C. 5 - 2 + 12 * 6 + 2 = 27 D. 5 + 2 - 12 + 6 * 2 = 13

Answer - Option A
Explanation :
Clearly, If ‘-‘ stands for addition, ‘+’ for multiplication,
‘÷’ for subtraction and ‘×’ for division, then
5 + 2 -12 * 6 - 2 = 10
After changing the signs,
5 x 2 + 12 ÷ 6 - 2
10 + 2 - 2
= 10
Hence, A is correct.
7. In the following question, select the related letters from the given alternatives. ABCD : MOQS : : FGHI : ?
    A. RTXV B. STTV C. RTVX D. RVTW

Answer - Option C
Explanation :

Similarly,

8. In the following question, select the missing number from the given series. 36 44 32 72 88 64 54 66 ?
    A. 44 B. 48 C. 50 D. 64

Answer - Option B
Explanation :
Here the logic is,
Difference between first and third row = difference between second and third row.
54 – 36 = 18 and 72 – 54 = 18
66 – 44 = 22 and 88 – 66 = 22
Similarly,
? – 32 = 64 - ?
Therefore ? = [latex]\frac {(64 + 32)}{2}[/latex]
= 48
9. A series is given with one term missing. Select the correct alternative from the given ones that will complete the series. RS, ZA, HI, ?
    A. LM B. KL C. PQ D. PR

Answer - Option C
10. Arrange the given words in the sequence in which they occur in the dictionary. i. Application ii. Approve iii. Appeal iv. Astonishing
    A. ii, iii, iv, i B. iii, ii, i, iv C. iii, i, ii, iv D. i, ii, ii, iv

Answer - Option C
Explanation :
Alphabetical order is:
iii. Appeal
i. Application
ii. Approve
iv. Astonishing
Hence the order is (iii, i, ii, iv)
1. In the following question, select the missing number from the given series. 6 10 9 7 2 1 3 4 ?
    A. 9 B. 8 C. 6 D. 7

Answer - Option C
Explanation :
Column I: 6 + 7 + 3 = 16
Column II: 10 + 2 + 4 = 16
Similarly,
Column III: 9 + 1 + ? = 16
? = 16 – 9 – 1 = 6
Thus the missing number is 6.
2. In the following question, select the missing number from the given series. 49 169 196 17 28 ? 100 225 144
    A. 27 B. 28 C. 26 D. 25

Answer - Option C
Explanation :
In column I: v49 = 7 and v100 = 10
7 + 10 = 17
In column II: v169 = 13 and v225 = 15
13 + 15 = 28
Similarly, in column III: v196 = 14 and v144 = 12
14 + 12 = 26
Thus the missing number is 26.
3. Arrange the given words in the sequence in which they occur in the dictionary. i. Manufacture ii. Manualism iii. Manumission iv. Manual
    A. iv, ii, i, iii B. iii, ii, i, iv C. ii, iv, i, iii D. i, ii, iii, iv

Answer - Option A
Explanation :
MANU is common to all. Hence we will look at the next letters.
Out of ‘f’ ‘a’ ‘m’ and ‘a’, we can see that again ‘a’ is common. So let’s first compare the next letters after ‘a’.
We have ‘l’ and ‘l’ again common. So jump for next. Now
Manual is over and hence it is an indication that it will come first.
So 4.
Then 2.
Then out of ‘f’ & ‘m’, ‘f’ will come first and then ‘m’.
So, 1
And at last 3
Hence the sequence is 4,2,1,3
Hence A
4. Select the related word/letters/number from the given alternatives. Subhas Chandra Bose : Orissa :: Mahatma Gandhi : ?
    A. Bihar B. Jammu and Kashmir C. Gujarat D. Delhi

Answer - C
Explanation :
Subhash Chandra bose was born in Cuttak, Orissa and hence we need to match mahatma Gandhi to his birth place. Out of all the options present, as we all know Gandhi was born in Porbandar which is in present day Gujarat.
Hence C
5. A series is given with one term missing. Choose the correct alternative from the given ones that will complete the series. JN, OR, UW, BC, ?
    A. KM B. JJ C. JK D. KJ

Answer - Option B
Explanation :
J + 5 = O
O + 6 = U
U + 7 = B
B + 8 = J
N + 4 = R
R + 5 = W
W + 6 = C
C + 7 = J
Hence JJ i.e. B
6. Select the odd word/letters/number /number pair from the given alternatives.
    A. IJKRQP B. FGHUTS C. BCDWXY D. KLMPON

Answer - Option C
Explanation :
All except ‘BCDWXY’ are written in two part where 1st part is in normal order and 2nd part is
written in reverse order of their occurrence in English language.
Hence, the correct option is C.
7. Ankita travels 35 km towards the west, takes a right turn and travels 50 km more. Next, she takes another right turn and travels 35 km in that direction. How far is she now from her original position?
    A. 75 km B. 25 km C. 50 km D. 20 km

Answer - Option C
8. The age of Ravi is twice the age of Prakash and three times the age of Kumar. If the age of Prakash is 24 years, then what will be the age of Kumar after 4 years ?
    A. 16 years B. 20 years C. 24 years D. 22 years

Answer - Option B
Explanation :
Going by the conditions given:
Ravi’s age = 2 × 24 ( Prakash’s age) 48
Kumar’s age = 48 (Ravi’s age) ÷ 3 16
After 4 years, Kumar will be: 16 + 4 = 20
Hence, the correct option is B.
9. In the following question, select the missing number from the given series? 81 3 90 60 4 76 49 6 ?
    A. 78 B. 80 C. 85 D. 75

Answer - Option C
Explanation :
The elements in the 3rd column is equals to the sum of elements in 1st column plus the square of elements in 2nd column :
90 = 81 + [latex]{3}^{2}[/latex]
76 = 60 + [latex]{4}^{2}[/latex]
? = 49 + [latex]{6}^{2}[/latex] = 85
Hence, the correct option is C.
10. A series is given with one term missing. Choose the correct alternative from the given ones that will complete the series. 64, 100, 169, 289, ?
    A. 484 B. 444 C. 404 D. 468

Answer - Option A
Explanation :
64 = 8 * 8
And 8 + 2 = 10
10 * 10 = 100
And 10 + 3 = 13
13 * 13 = 169
And 13 + 4 = 17
17 * 1 = 289
And 17 + 5 = 22
22 * 22 = 484
Hence A